UWorld Flashcards

1
Q

An elderly pt presenting with progressive memory impairment along with impaired vibration sense, positive romberg sign, spastic paresis and hyperreflexia most likely suffers from ..

A

Vitamin B deficiency

dementia with subacute combined degeneration

How well did you know this?
1
Not at all
2
3
4
5
Perfectly
2
Q

A pt with vitamin B 12 deficiency presenting with elevated lactate dehydrogenase, low haptoglobin, and indirect hyperbilirubinemia most likely suffer from …

A

Hemolytic anemia secondary to ineffective erythropoesis

defective DNA synthesis in new RBCs results in hemolysis when pt has vitamin B deficiency

How well did you know this?
1
Not at all
2
3
4
5
Perfectly
3
Q

An elderly pt presenting with dementia, cognitive fluctuations, visual hallucinations, parkinsonism (bradykinesia, rigidity, gait abnormalities), vivid dreams, and/or neuroleptic hypersensitivity most likely suffers from …

A

Lewy Body Dementia

How well did you know this?
1
Not at all
2
3
4
5
Perfectly
4
Q

What treatment for Lewy Body Dementia can exacerbate the pt’s visual hallucination symptoms?

A

dopamine agonist

lewy body dementia treated with cholinesterase inhibitors and anti-parkinson medications

How well did you know this?
1
Not at all
2
3
4
5
Perfectly
5
Q

A pt presenting with a unilateral headache, ipsilateral miosis (small pupil), and ipsilateral ptosis (eyelid drooping) most likely suffers from …

A

Carotid Artery Dissection

(in setting of trauma/ connective tissue disease/ smoking/ neck manipulation/ hypertension/ 3-point restraint belts in car accidents)

How well did you know this?
1
Not at all
2
3
4
5
Perfectly
6
Q

What is the most appropriate next step in management of a pt presenting with unilateral headache, ipsilateral miosis (small pupil), and ipsilateral ptosis (eyelid drooping)?

A

Head and Neck CT angiography

detect carotid artery dissection

How well did you know this?
1
Not at all
2
3
4
5
Perfectly
7
Q

An alcoholic pt presenting with confusion, ataxia and nystagmus (leading to ophthalmoplegia) most likely suffers from …

A

Wernicke’s Encephalopathy

due to thiamine deficiency; tx with IV/ IM thiamine

How well did you know this?
1
Not at all
2
3
4
5
Perfectly
8
Q

An alcoholic pt presenting with amnesia (anterograde and retrograde) and confabulation most likely suffers from …

A

Korsakoff’s syndrome

due to thiamine deficiency

How well did you know this?
1
Not at all
2
3
4
5
Perfectly
9
Q

Administration of what substance prior to thiamine in an alcoholic with Wernicke’s encephalopathy can precipitate progression into Korsakoff’s syndrome?

A

Glucose

do not give glucose before thiamine

How well did you know this?
1
Not at all
2
3
4
5
Perfectly
10
Q

What part of the brain is affected in alcoholic patients with Wernicke-Korsakoff syndrome?

A

mammillary bodies

How well did you know this?
1
Not at all
2
3
4
5
Perfectly
11
Q

What are three major reversible causes of dementia that should be ruled out before diagnosing a pt with Alzheimer’s?

A
  1. hypothyroidism (perform thyroid function test)
  2. Vitamin B12 deficiency
  3. depression

(consider vitamin B1 in alcoholic and severely malnourished)

How well did you know this?
1
Not at all
2
3
4
5
Perfectly
12
Q

A pt presenting with rapidly progressive dementia, periodic sharp wave complexes on EEG, and presence of 14-3-3 protein in CSF most likely suffers from …

A

Creutzfeldt-Jakob disease

How well did you know this?
1
Not at all
2
3
4
5
Perfectly
13
Q

What is the best initial treatment for Alzheimer’s disease?

A

acetylcholinesterase inhibitors

donepezil better than tacrine b/c once daily and more improvement in behavioral and cognitive domains

How well did you know this?
1
Not at all
2
3
4
5
Perfectly
14
Q

What part of the brain has been associated with the decreased levels of acetylcholine in Alzheimer’s?

A

degeneration of basal nucleus of Meynert (along with diffuse deficiency of choline acetyltransferase)

How well did you know this?
1
Not at all
2
3
4
5
Perfectly
15
Q

A pt presenting with unilateral onset of symptoms consisting of bradykinesia, resting tremor, and rigidity with a persistant asymmetry in the symptoms, and develops postural instability most likely suffers from ..

A

Parkinson’s Disease

dx: clinical

How well did you know this?
1
Not at all
2
3
4
5
Perfectly
16
Q

Other than physical exam, what diagnostic test can be used to diagnosis Parkinson’s disease in a pt whose symptoms are suggestive but pt not responding to initial therapy?

A

Striatal Dopamine Transporter Scan

low uptake of ioflupane-123 in striatal region

How well did you know this?
1
Not at all
2
3
4
5
Perfectly
17
Q

What is the best initial treatment for Parkinson’s Disease?

A

if > 65 years and severe: levodopa
if less than 65 years and mild: dopamine agonist

(prevent use of levodopa as long as can)

How well did you know this?
1
Not at all
2
3
4
5
Perfectly
18
Q

What is the clinical criteria for brain death? (3)

A
  1. imaging evidence of devastating cause
  2. absence of confounding factors (endocrine, electrolyte abnormalities, drug intoxication)
  3. hemodynamically stable (temp > 36; sbp > 100)
How well did you know this?
1
Not at all
2
3
4
5
Perfectly
19
Q

An elderly pt presenting with dementia along with hyponatremia, macrocytosis (high MCV), and bradypsychia (slowed thought process)/ cognitive slowing most likely suffers from..

A

Hypothryoidism

obtain thryoid function testing

How well did you know this?
1
Not at all
2
3
4
5
Perfectly
20
Q

A pt presenting with progressive flaccid ascending paralysis (progressive lower extremity weakness), areflexia (absent deep tendon reflexes), and possibly difficulty breathing weeks after an upper respiratory or gastrointestinal infection most likely suffers from ..

A

Guillian Barre Syndrome

intubate if signs of respiratory compromise

How well did you know this?
1
Not at all
2
3
4
5
Perfectly
21
Q

What diagnostic test is most useful in determining the degree of respiratory impairment in a pt with Guillian Barre Syndrome?

A

bedside vital capacity measurements

How well did you know this?
1
Not at all
2
3
4
5
Perfectly
22
Q

What is the mainstay of therapy for Guillian Barre Syndrome (after triaging the severity and providing appropriate respiratory support)?

A

plasmapheresis and/or IVIG (IV immunoglobulin)

How well did you know this?
1
Not at all
2
3
4
5
Perfectly
23
Q

A pt presenting with a tremor (usually in distal upper extremities) that becomes more pronounced with outstretching of the arm/ end of an activity or movement in the setting of a family member having had similar symptom and alcohol improves symptoms most likely suffers from ..

A

Benign Essential Tremor (Familial Tremor)

normal life expectancy, no significant disabilities/ neurologic problems

How well did you know this?
1
Not at all
2
3
4
5
Perfectly
24
Q

What is the best initial treatment for benign essential tremor (familial tremor) that is significantly impacting a pt’s life?

A

beta blockers (propanolol)

How well did you know this?
1
Not at all
2
3
4
5
Perfectly
25
Q

What is the next best step in management of a pt who meets all clinical criteria of brain death (clinical/imaging evidence of devastating cause, absence of confounding factors, hemodynamic stability)?

A

Neurologic examination

(cerebral and brain stem reflexes including motor response to pain, pupillary, oculocephalic, oculovestibular/ caloric, cough reflex with tracheal suctioning, corneal, gag, suckling, swallowing, extensor posturing)

How well did you know this?
1
Not at all
2
3
4
5
Perfectly
26
Q

What is the next best step in management of a pt who meets all clinical criteria of brain death (clinical/imaging evidence of devastating cause, absence of confounding factors, hemodynamic stability) and has abnormal neurologic findings (coma, absent motor response to pain, absent pupillary light reflex, absent corneal and oculocephalic reflexes, absent cough reflex with tracheal suctioning)?

A

Apnea Testing

(pre-oxygenated and then disconnect ventilator)

(absence of respiratory response for 8-10 minutes with PaCO2 > 60 or > 20 from baseline, and final arterial pH of less than 7.28

How well did you know this?
1
Not at all
2
3
4
5
Perfectly
27
Q

What are the three conditions in which ancillary testing (EEG showing electrocerebral silence or absence of somatosensory/ brainstem-evoked potentials; absent intracranial blood flow on brain imaging) for suspected brain death should be performed?

A
  1. don’t meet all clinical criteria
  2. difficult to evaluate neuro exam or pt paralyzed
  3. difficult to interpret or inconclusive apnea test
How well did you know this?
1
Not at all
2
3
4
5
Perfectly
28
Q

A pt presenting with loss of pupillary reaction bilaterally, vertical gaze paralysis (impaired upward gaze), loss of optokinetic nystagmus and ataxia in the setting of progressively worsening headache most likely suffers from …

A

Parinaud’s syndrome

pineal tumor that can secrete HCG

How well did you know this?
1
Not at all
2
3
4
5
Perfectly
29
Q

A male pt presenting with headaches, pulsating rhythmic sound in ears when bent over, papilledema, and progressive optic atropy leading to vision problems (possibly blindness) while on retinoic acid derivative treatment for acne most likely suffers from …

A

Pseudotumor Cerebri

Idiopathic intracranial hypertension

How well did you know this?
1
Not at all
2
3
4
5
Perfectly
30
Q

What spinal cord level is responsible for the cremasteric reflex?

A

L1-L2

can be absent if pt suffering from diabetic neuropathy

How well did you know this?
1
Not at all
2
3
4
5
Perfectly
31
Q

An adolescent presenting with generalized tonic clonic seizure and an EEG showing bilateral polyspike and slow discharge in the setting of a history of myoclonic jerks of the upper extremity within the first hour of waking and history of anxiety most likely suffers from …

A

Juvenile Myoclonic Epilepsy

How well did you know this?
1
Not at all
2
3
4
5
Perfectly
32
Q

What is the best initial treatment for Juvenile Myoclonic Epilepsy?

A

Valproic Acid (Depakote)

How well did you know this?
1
Not at all
2
3
4
5
Perfectly
33
Q

A pt presents with a history of unsteady gait rapidly progressing to ascending paralysis within hours to days of spending time in a wooded area along with loss of deep tendon reflexes, normal CSF findings, and absence of fever/ pupillary abnormalities most likely suffers from …

A

Tick Paralysis

dx: skin examination to assess for tick attachment

How well did you know this?
1
Not at all
2
3
4
5
Perfectly
34
Q

What is the best initial treatment for pt presenting with Tick Paralysis?

A

remove tick carefully (improvement immediately, may worsen initially)

How well did you know this?
1
Not at all
2
3
4
5
Perfectly
35
Q

What is the best initial step in management for a pt with suspected epidural spinal cord compression (i.e. pt with cancer history presenting with thoracic radicular pain wrapping around abdomen with associated weakness and sensory changes in lower extremities)?

A

High Dose corticosteroids (reduce inflammation and swelling) followed by spinal MRI (to confirm diagnosis and assess extent)

How well did you know this?
1
Not at all
2
3
4
5
Perfectly
36
Q

What is the definitive treatment for epidural spinal cord compression secondary to tumor metastasis?

A

radiation: if stable spine, minimal neuro symptoms or highly radiosensitive tumor
surgical decompression: if radioresistant or unstable spine

How well did you know this?
1
Not at all
2
3
4
5
Perfectly
37
Q

What diagnostic tests are included in a routine dementia work-up before diagnosing a pt with Alzheimer’s? (6)

A
  1. CBC (rule out anemia)
  2. Serum glucose (rule out hypoglycemia)
  3. electrolytes (rule out electrolyte disturbance)
  4. calcium level (rule out hypo/ hypercalemia)
  5. creatinine (rule out renal failure)
  6. TSH (rule out hypothryoidism)
How well did you know this?
1
Not at all
2
3
4
5
Perfectly
38
Q

What Mini-Mental Status Exam score indicates dementia? What score indicates benign forgetfulness of senility?

A

dementia: less than 20
benign: greater than 25

How well did you know this?
1
Not at all
2
3
4
5
Perfectly
39
Q

What is the prognosis of Alzheimer’s disease?

A

irreversible, medication slows progression but there is no cure

How well did you know this?
1
Not at all
2
3
4
5
Perfectly
40
Q

A pt who presents to the ER unconscious, unresponsive to verbal and tactile stimuli, has symmetric decrease in muscular tone, normal deep tendon reflexes, and a normal response to the caloric stimulation (transient conjugate slow deviation of gaze to side of stimulus followed by saccadic correction to midline after irrigation of the external auditory canal with cold water) most likely suffers from …

A

Psychogenic Coma

How well did you know this?
1
Not at all
2
3
4
5
Perfectly
41
Q

What is the net clinical benefit of a medication? (in reference to studies)

A

measure of possible benefit minus its possible harm

How well did you know this?
1
Not at all
2
3
4
5
Perfectly
42
Q

… is when participants in trials are analyzed in the groups to which they were randomized, regardless of whether they received or adhered to the allocated intervention and regardless of whether they withdrew from treatment

A

Intention-to-treat analysis

purpose is to maintain randomization

How well did you know this?
1
Not at all
2
3
4
5
Perfectly
43
Q

…. is the practice of monitoring the safety of medication or devices after they have been released on the market due to the fact that clinical trials have ….

A

Postmarketing surveillance; inadequate power (short follow-up times, underrepresented populations, too few participants to detect rare and serious side effects)

How well did you know this?
1
Not at all
2
3
4
5
Perfectly
44
Q

A pt presenting with sticking sensation in the throat (dysphagia) accompanied by heartburn and esophageal manometry showing absence of peristaltic waves in the lower two-thirds of the esophagus and significant decrease in lower esophageal sphincter tone most likely suffers from …

A

Scleroderma

results in esophageal dysmotility

How well did you know this?
1
Not at all
2
3
4
5
Perfectly
45
Q

What are the esophageal manometric findings in Achalasia?

A

significant decrease/ absence of peristaltic waves and increased lower esophageal sphincter tone

How well did you know this?
1
Not at all
2
3
4
5
Perfectly
46
Q

A pt with history of treated cancer (especially Hodgkin lymphoma) presenting with myocardial ischemia, restrictive cardiomyopathy with diastolic dysfunction, sclerotic and calcified valves with associated stenosis/ regurgitation, and ostial narrowing of the coronary arteries most likely suffers from …

A

Radiation Induced Cardiotoxicity

How well did you know this?
1
Not at all
2
3
4
5
Perfectly
47
Q

What is the next best step in management of a pt with suspected active TB (travel to endemic area, cough for more than 2-3 weeks, fever, night sweats, weight loss) after chest x-ray is performed (showing infiltrate or cavitary lesion)?

A

3 sputum specimens for acid fast bacillus smears and culture (within 8-24 hours intervals, and at least 1 early morning)

How well did you know this?
1
Not at all
2
3
4
5
Perfectly
48
Q

A pt with active TB (travel to endemic area, cough for more than 2-3 weeks, fever, night sweats, weight loss, imaging showing infiltrate or cavitary lesion) with 3 negative sputum specimens is considered …

A

noninfectious

How well did you know this?
1
Not at all
2
3
4
5
Perfectly
49
Q

What is the formula for the NNT (number needed to treat; number of patients who need to be treated in order to prevent on additional bad outcome)?

A

NNT = 1/ (control group rate- experimental group rate)

NNT = 1/ (absolute risk reduction)

How well did you know this?
1
Not at all
2
3
4
5
Perfectly
50
Q

What are the two most common causes of diaphragmatic paralysis (decreased movement of rib cage and abdominal flattening)?

A
  1. birth injury (Erb’s palsy secondary to shoulder traction)

2. cardiothoracic injury

How well did you know this?
1
Not at all
2
3
4
5
Perfectly
51
Q

A pt presenting with easy fatigability, pallor of conjunctiva, pallor of oral mucosa, low hemoglobin, microcytosis (low MCV), anisocytosis on blood smear and serum ferritin level ( less than 30) most likely suffers from …

A

Iron Deficiency Anemia

likely due to GI bleed in males so perform fecal occult blood test

How well did you know this?
1
Not at all
2
3
4
5
Perfectly
52
Q

What is the typical prognosis for an infant suffering from hypertrophic cardiomyopathy (i.e. hypertrophic interventricular septum) as a result of being born to a diabetic mother with poor glycemic control during gestation?

A

does well with conservative medical therapy and the abnormality corrects itself (resolves spontaneously b/c no longer exposed to maternal hyperglycemia leading to excessive glycogen within myocardium)

How well did you know this?
1
Not at all
2
3
4
5
Perfectly
53
Q

What is the likely cause of isolated gastric varices in a pt with a history of chronic recurrent pancreatitis?

A

splenic vein thrombosis
(runs along posterior surface of pancreas and gets directly inflamed and thrombosed due to recurrent pancreatic inflammation)

How well did you know this?
1
Not at all
2
3
4
5
Perfectly
54
Q

What is the likely cause of gastric varices with esophageal varices in a pt with a history of chronic recurrent pancreatitis?

A

portal vein thrombosis

leads to prehepatic/ noncirrhotic portal hypertension

How well did you know this?
1
Not at all
2
3
4
5
Perfectly
55
Q

What is the benefit of smoking cessation in a pt who is age 60 or older?

A

mortality risk will fall below current smokers within 5 years

(can result in initial temporary increase in cough during first few weeks after cessation, followed by improvement)
(reduces risk of COPD exacerbation and developing lung cancer)

How well did you know this?
1
Not at all
2
3
4
5
Perfectly
56
Q

What is the most reliable physical exam finding for excluding severe aortic stenosis?

A

normal splitting of second heart sound (increased splitting of aortic and pulmonic components during inspiration)

How well did you know this?
1
Not at all
2
3
4
5
Perfectly
57
Q

What are three physical exam findings that indicate severe aortic stenosis?

A
  1. soft, single S2 (no normal splitting)
  2. delayed and diminshed carotid pulse (parvus and tardus)
  3. loud and late-peaking systolic murmur
How well did you know this?
1
Not at all
2
3
4
5
Perfectly
58
Q

What is the most immediate step in management for an infant presenting with bilateral flame shaped retinal hemorrhages suggestive of shaken baby syndrome?

A

CT scan of head

ultrasound can be used if fetus or infant with open fontanelles but not as accurate

How well did you know this?
1
Not at all
2
3
4
5
Perfectly
59
Q

What are the four indications for head CT after a head trauma in a kid?

A
  1. glascow coma score equal to/ less than 14
  2. signs of basilar skull fracture (battle sign- mastoid ecchymoses, raccoon eyes- orbital ecchymoses, CSF rhinorrhea or otorrhea, hemotympanum, cranial nerve deficits)
  3. altered mental status
  4. symptomatic pt with bleeding disorder
How well did you know this?
1
Not at all
2
3
4
5
Perfectly
60
Q

A type 1 diabetic who presents with recurrent hypoglycemic episodes evenly distributed throughout the day while on stable insulin dose (signs of decreasing insulin requirement) in setting of weight loss, weakness, eosinophilia, hyperkalemia, mild anion gap acidosis, prerenal azotemia, and low blood glucose most likely suffers from …

A

Adrenal Failure (secondary to autoimmune disease)

if pt has one autoimmune disease, predisposed to get another

How well did you know this?
1
Not at all
2
3
4
5
Perfectly
61
Q

What is the best initial screening test when a pt is suspected of having pheochromocytoma (episodic headaches, diaphoresis and tachycardia, early onset/ refractory hypertension, paroxysmal hypertension)?

A

24 hour fractionated urinary metanephrines and catecholamines or plasma free metanephrine levels

How well did you know this?
1
Not at all
2
3
4
5
Perfectly
62
Q

What is the next best step in management for a pt with symptoms and biochemical confirmation of pheochromocytoma?

A

Abdominal CT or MRI

How well did you know this?
1
Not at all
2
3
4
5
Perfectly
63
Q

What is the next best step in management for a pt who develops hypotension during surgical removal of adrenal tumor responsible for pheochromocytoma?

A

normal saline bolus followed by continous normal saline infusion

(pressors if needed)

How well did you know this?
1
Not at all
2
3
4
5
Perfectly
64
Q

What is the next best step in management for a pt who develops hypoglycemia during surgical removal of adrenal tumor responsible for pheochromocytoma?

A

IV dextrose infusion

due to decrease catecholamine supression of insulin secretion leading to hyperinsulinemia

How well did you know this?
1
Not at all
2
3
4
5
Perfectly
65
Q

What is the next best step in management for a pt who develops cardiac tachyarrhythmias during surgical removal pheochromocytoma?

A

IV lidocaine or esmolol

due to increase catecholamine release from adrenal gland handling

How well did you know this?
1
Not at all
2
3
4
5
Perfectly
66
Q

What is the next best step in management for pt who develops acute severe hypertension during surgical removal of pheochromocytoma?

A

IV phentolamine/ nitroprusside/ nicardipine

due to increase catecholamine release from intubation and adrenal gland manipulation

How well did you know this?
1
Not at all
2
3
4
5
Perfectly
67
Q

What is the best next step in management for a pt with an ECHO confirmed bicuspid valve?

A

ECHO screening in first degree relatives (autosomal dominant with incomplete penetrance or sporadic)

How well did you know this?
1
Not at all
2
3
4
5
Perfectly
68
Q

What is the next best step in management for a pt with suspected osteomyelitis (foot pain, swelling and erythema along with elevated WBC and ESR) but negative foot X-rays (lack periosteal reaction and osseous lucency)?

A

MRI of foot

non-invasive modality with highest accuracy for diagnosing osteomyelitis

How well did you know this?
1
Not at all
2
3
4
5
Perfectly
69
Q

A pt presenting with hyperthyroid symptoms, nontender thryoid goiter, elevated T3 and T4, low TSH, positive TPO antibody and diffusely decreased radioiodine uptake study in the setting of immunotherapy treatment (interferon for hep C) or lithium treatment most likely suffers from..

A

Painless Thyroiditis/ Silent Thyroiditis/ Subacute lymphocytic thyroiditis

(autoimmune induced proteolysis of thyroglobulin leading to thyroid hormone release until stores depleted)

How well did you know this?
1
Not at all
2
3
4
5
Perfectly
70
Q

What treatment can be used in painless/ silent/ subacute lymphocytic thyroiditis during the hyperthyroid state?

A

propanolol (beta blockers)

How well did you know this?
1
Not at all
2
3
4
5
Perfectly
71
Q

What is the best initial step in management prior to treating a sexually active female of reproductive age for a UTI?

A

urine beta-hCG (pregnancy test)

How well did you know this?
1
Not at all
2
3
4
5
Perfectly
72
Q

What is the best initial treatment for UTI’s in a pregnant patient?

A

nitrofurantoin (and first generation cephalosporin)

How well did you know this?
1
Not at all
2
3
4
5
Perfectly
73
Q

What is the influenza vaccination protocol in a pt with egg allergies in which the reaction is limited to hives?

A

administer inactivated influenza vaccine and observe for at least 30 minutes

How well did you know this?
1
Not at all
2
3
4
5
Perfectly
74
Q

…. is a measure of the likelihood that an outcome will occur in the presence of a particular exposure compared to the likelihood that the outcome will occur in the absence of that exposure

A

Odds Ratio

OR = ad/ bc

(if greater than 1: exposure associated with high odds of outcome)
(if less than 1: exposure associated with lower odds of outcome)

How well did you know this?
1
Not at all
2
3
4
5
Perfectly
75
Q

An asthmatic pt presenting with history of recurrent fever, malaise, cough with brownish mucoid expectoration, wheezing, symptoms of bronchial obstruction, eosinophilia and bilateral upper lobe parenchymal infiltrates on CXR most likely suffers from …

A

Allergic Bronchopulmonary Aspergillosis (ABPA)

How well did you know this?
1
Not at all
2
3
4
5
Perfectly
76
Q

What is the initial screening test for suspected allergic bronchopulmonary aspergillosis in an asthmatic pt?

A

skin prick test for aspergillus
(if negative, unlikely diagnosis)
(if postive, perform serum total IgE level and precipitating serum antibodies to Aspergillus fumigatus)

How well did you know this?
1
Not at all
2
3
4
5
Perfectly
77
Q

What is the best initial treatment for allergic bronchopulmonary aspergillosis (ABPA)?

A

oral prednisone (corticosteroids)

How well did you know this?
1
Not at all
2
3
4
5
Perfectly
78
Q

A pt presenting with fragmented sleep, hypnagogic hallucinations (seeing/hearing things as fall asleep), recurrent lapses into sleep or napping multiple times within the same day occuring at least 3 times weekly for 3 months along with at least one of the following symptoms (cataplexy-conscious brief episodes of sudden bilateral muscle tone loss precipitated by emotions or spontaneous abnormal facial movements without emotional triggers; hypocretin 1 deficiency by CSF analysis; rapid eye movement sleep latency less than 15 minutes) most likely suffers from ..

A

Narcolepsy

How well did you know this?
1
Not at all
2
3
4
5
Perfectly
79
Q

What is the diagnostic test of choice for suspected narcolepsy?

A

Polysomnography (Sleep Study)

shows multiple spontaneous awakenings, reduced sleep efficiency, reduced latency of rapid eye movement sleep

How well did you know this?
1
Not at all
2
3
4
5
Perfectly
80
Q

What is the best initial treatment for narcolepsy?

A

Modafinil (stimulant) along with behavioral intervention (good sleep hygiene, scheduled naps)

How well did you know this?
1
Not at all
2
3
4
5
Perfectly
81
Q

What is the best treatment for cataplexy (sudden onset of muscle weakness triggered by intense emotional episodes) associated with narcolepsy?

A

serotonin-norepinephrine reuptake inhibitor (venlafaxine)

can also use SSRI, TCAs, sodium oxybate- restricted due to abuse potential

How well did you know this?
1
Not at all
2
3
4
5
Perfectly
82
Q

A pt presents with respiratory difficulty, cough, sputum, fever, chills, and infiltrate on CXR 1-5 days after choking most likely suffers from …

A

Aspiration Pneumonia

How well did you know this?
1
Not at all
2
3
4
5
Perfectly
83
Q

What is the best initial treatment for aspiration pneumonia (gram positive cocci, gram negative rods, anaerobes)?

A

Clindamycin (or beta lactam and beta-lactamase inhibitor combination)

How well did you know this?
1
Not at all
2
3
4
5
Perfectly
84
Q

What is the best step in management in a hospitalize pt who develops acute renal failure to distinguish between prerenal azotemia and acute tubular necrosis?

A

fluid challenge
(if improved urine output, then prerenal azotemia)
(if no improvement in urine output, then acute tubular necrosis)

How well did you know this?
1
Not at all
2
3
4
5
Perfectly
85
Q

A pt who develops oliguria after hypotension with an elevated BUN and creatinine but a normal BUN to creatinine ratio (10-15), anion gap acidosis, and oliguria does not improve with fluid challenge most likely suffers from …

A

Acute Tubular Necrosis

muddy brown casts, high fractional excretion of sodium > 2%

How well did you know this?
1
Not at all
2
3
4
5
Perfectly
86
Q

What is an important diagnostic test that should be performed in all high-risk pts being treated for aspiration pneumonia?

A

speech and swallow evaluation by speech therapist (to assess for need for diet modification)

How well did you know this?
1
Not at all
2
3
4
5
Perfectly
87
Q

What is the best initial treatment for insomnia secondary to bereavement in the elderly patients?

A

behavioral therapy and relaxation techniques

(elderly at increased risk of falls and bone fractures at night and reduced daytime alertness if use medications for insomnia)

How well did you know this?
1
Not at all
2
3
4
5
Perfectly
88
Q

What are the indications for adding antibiotic therapy to incision and drainage for a skin abscess in kid? (6)

A
  1. diameter > 5 cm
  2. multiple abscesses
  3. extensive surrounding cellulitis
  4. systemic signs of infection
  5. immunocompromised
  6. age less than 6 months
How well did you know this?
1
Not at all
2
3
4
5
Perfectly
89
Q

What is the best step in management for a pt with a septate uterus (septum within the uterus) and non-obstructing fallopian tubes who experiences recurrent miscarriages?

A

metroplasty via hysteroscopy (surgery to decrease risk of future miscarriage)

How well did you know this?
1
Not at all
2
3
4
5
Perfectly
90
Q

A pt who develops hyperactive deep tendon reflexes, muscle cramps and possibly convulsions during or shortly after surgery (especially if extensive transfusions were required) most likely suffers from …

A

Hypocalcemia

due to volume expansion and hypoalbuminemia

How well did you know this?
1
Not at all
2
3
4
5
Perfectly
91
Q

A pt with sarcoidosis presenting with acute onset red eye, photophobia, blurring of vision, and mild to moderate pain most likely suffers from ..

A

Anterior Uveitis

most common ocular manifestation of sarcoidosis

How well did you know this?
1
Not at all
2
3
4
5
Perfectly
92
Q

A pt presents with a papular lesion that slowly grows in size and eventually ulcerates with non-purulent discharge along with similar lesions along the lymphatic channels proximal to the original lesions after an outdoor activity/ occupation most likely suffers from ..

A

Sporotrichosis

Sporothrix schenckii inoculated in skin or subcutaneous tissue

How well did you know this?
1
Not at all
2
3
4
5
Perfectly
93
Q

What is the treatment for lymphocutaneous sporotrichosis?

A

itraconazole for 3-6 months

How well did you know this?
1
Not at all
2
3
4
5
Perfectly
94
Q

A pt who develops intense pain which increases with passive stretch accompanied by paresthesia (ant crawling sensation, burning sensation), tightness of extremity, weakness, and rapidly increasing and tense swelling after extremity thromboectomy most likely suffers from …

A

postischemic Compartment Syndrome

due to interstitial edema and intracelluar swelling following tissue ischemia and subsequent reperfusion

How well did you know this?
1
Not at all
2
3
4
5
Perfectly
95
Q

What is a diagnostic test used to confirm compartment syndrome?

A

measuring tissue pressures
(compartment pressure more then 30 mmHg or delta pressure (diastolic bp - compartment pressure) less than 20-30 mmHg)

(time to surgical intervention is most important factor of prognosis; fasciotomy is tx)

How well did you know this?
1
Not at all
2
3
4
5
Perfectly
96
Q

What is the management step of choice to prevent deep venous thrombosis complication in a pt with a hip fracture?

A

low-molecular weight heparin (start at time of fracture, hold at time of surgery, then continue for at least 10 days post-op)

How well did you know this?
1
Not at all
2
3
4
5
Perfectly
97
Q

What is the initial and subsequent treatment options for mild- moderate functional constipation in children?

A

initial: dietary modification (increase fiber, decrease dairy)
subequent: mild laxative (magnesium hydroxide aka milk of magnesia)

How well did you know this?
1
Not at all
2
3
4
5
Perfectly
98
Q

What is the recurrence risk after having an infant with Turner’s syndrome (dorsal feet and hands edema, short webbed neck, cardiac murmur, wide spaced nipples)?

A

no increased recurrence risk (same as general population)

How well did you know this?
1
Not at all
2
3
4
5
Perfectly
99
Q

A pt presenting with a pervasive, lifelong pattern of suspiciousness and distrust in home and work relationships, hypervigilience, hostility, interpreting others’ motives as malevolent, controlling in personal relationship, and pathologically jealous most likely suffers from …

A

Paranoid Personality Disorder

How well did you know this?
1
Not at all
2
3
4
5
Perfectly
100
Q

A pt presenting with pattern of excessive emotionality, attention seeking behavior, inappropriate and sexually seductive/ provocative behavior, shallow/shifting/ dramatic emotions, easily influenced behavior, impressionistic/ vague speech and considers relationships more intimate than they really are most likely suffers from …

A

Histrionic Personality Disorder

often attempt to manipulate doctor-patient relationship through seduction and dramatic descriptions of new symptoms

How well did you know this?
1
Not at all
2
3
4
5
Perfectly
101
Q

What is the next best step in management of a pt who decides to take herbal medications against medical advice?

A

counsel pt on side effects, document discussion and refusal to discontinue, continue to follow-up pt’s health

How well did you know this?
1
Not at all
2
3
4
5
Perfectly
102
Q

A pt presenting with recurrent episodes of wheezing, dyspnea, cough and chest tightness that occurs especially at night and early morning hours and cough is induced when the pt is asked to do a forced expiration most likely suffers from ..

A

Asthma

How well did you know this?
1
Not at all
2
3
4
5
Perfectly
103
Q

What is the best test to diagnosis a pt with asthma who is currently asymptomatic?

A

Pulmonary function testing with methacholine challenge

obstructive pattern/ FEV1 decreased by more than 20% with methacholine challenge

How well did you know this?
1
Not at all
2
3
4
5
Perfectly
104
Q

What are the arthritic swellings of the distal interphalangeal joints in a pt with osteoarthritis called?

A

Heberden’s nodules

How well did you know this?
1
Not at all
2
3
4
5
Perfectly
105
Q

What are the arthritic swellings of the proximal interphalangeal joints in a pt with osteoarthritis called?

A

Bouchard’s nodules

How well did you know this?
1
Not at all
2
3
4
5
Perfectly
106
Q

What is the next best step in management for a pt diagnosed with diphtheria (pharyngitis, cervical lymphadenopathy, fever, pharyngeal erythema with exudate and membrane formation)?

A

Diphtheria Antitoxin

have epinephrine available in case of anaphylaxis or serum sickness because made with horse serum

How well did you know this?
1
Not at all
2
3
4
5
Perfectly
107
Q

What is the best initial test for diagnosing acute cholecystitis (gallbladder inflammation presents with ill appearance, right upper quadrant tenderness with guarding, positive murphy sign)?

A

Ultrasound

shows gallstones, gallbladder wall thickening/ edema, sonographic murphy’s sign

How well did you know this?
1
Not at all
2
3
4
5
Perfectly
108
Q

What is the next best step in management for suspected acute cholecystitis (gallbladder inflammation presents with ill appearance, right upper quadrant tenderness with guarding, positive murphy sign) when ultrasound findings are unclear (i.e. small stones without gallbladder edema/ ultrasonographic murphy’s sign)?

A

HIDA scan

positive if no visualization of gallbladder

How well did you know this?
1
Not at all
2
3
4
5
Perfectly
109
Q

What is the best initial treatment for a well-differentiated adenocarcinoma in the head of a pedunculated colonic polyp without involvement of the stalk or resection margins?

A

endoscopic removal

How well did you know this?
1
Not at all
2
3
4
5
Perfectly
110
Q

What is the next best step in management after endoscopic removal of a well-differentiated adenocarcinoma in the head of a pedunculated colonic polyp without involvement of the stalk or resection margins?

A

follow-up colonoscopy 2-3 months later (followed by 1 year, 4 years and 9 years later)

How well did you know this?
1
Not at all
2
3
4
5
Perfectly
111
Q

A pt with a history of an autoimmune disorder (type 1 diabetes, hypothyroidism) presenting with macrocytic anemia, loss of proprioception and vibration sense, ataxia, spasticity, weakness and loss of reflexes most likely suffers from …

A

Pernicious Anemia

(due to autoimmune destruction of parietal cell leading to decreased intrinsic factor leading to decreased vitamin B12 absorption)

How well did you know this?
1
Not at all
2
3
4
5
Perfectly
112
Q

What should be monitored during the first few days of treatment with vitamin B12 for vitamin B 12 deficiency associated megaloblastic anemia?

A

potassium level

formation of new RBCs can causes hypokalemia

How well did you know this?
1
Not at all
2
3
4
5
Perfectly
113
Q

What is the first line, second line and third line treatment for a patient with significant obesity (BMI greater than 35) with comorbid conditions?

A
  1. dietary changes, behavioral therapy, exercise
  2. Orilstat (medical therapy)
  3. Bariatric surgery
How well did you know this?
1
Not at all
2
3
4
5
Perfectly
114
Q

A pt presenting with fever, otalgia (ear pain), tympanic membrane erythema, bulging of tympanic membrane (TM inflammation) and limited tympanic membrane motility on insufflation (middle ear effusion) most likely suffers from …. and the likely causes are…. (3)

A

Acute Otitis Media

  1. Strep pneumoniae
  2. nontypeable H. influenzae
  3. Moraxella pneumoniae
How well did you know this?
1
Not at all
2
3
4
5
Perfectly
115
Q

What is the most likely cause of concurrent otitis media and purulent conjunctivitis?

A

Nontypeable H. influenzae

How well did you know this?
1
Not at all
2
3
4
5
Perfectly
116
Q

What is the first line treatment for acute otitis media? What is the best treatment for repeat infection within one month of initial treatment?

A

first: high dose amoxicillin
repeat: amoxicillin-clavulanic acid (due to beta-lactamase resistance)

How well did you know this?
1
Not at all
2
3
4
5
Perfectly
117
Q

What is the best step in management to maximize the viability of donor organs in a brain dead patient? (2)

A
  1. achieve hemodynamic stability

2. maintain normothermia with blankets (using warm air blankets and warm IV if body temp less than 35 C)

How well did you know this?
1
Not at all
2
3
4
5
Perfectly
118
Q

What is the inheritance status of a daughter born to an affected father and normal mother in the case of an X-linked recessive disorder (i.e. hemophila)?

A

carrier (received 1 affected X from dad and 1 normal X from mom)

How well did you know this?
1
Not at all
2
3
4
5
Perfectly
119
Q

What is the hemoglobin threshold level for red blood cell transfusion in a hemodynamically stable pt with an upper GI bleed and no comorbidities?

A

hemoglobin less than 7 g/dl

How well did you know this?
1
Not at all
2
3
4
5
Perfectly
120
Q

What is the hemoglobin threshold level for red blood cell transfusion in a hemodynamically stable pt with an upper GI bleed and a risk for morbidity in setting of severe anemia such as unstable coronary artery disease?

A

hemoglobin equal to or less than 9 g/dl

How well did you know this?
1
Not at all
2
3
4
5
Perfectly
121
Q

A pt presenting with chronic intense rash with excoriated lichenified plaques and a possible history of asthma/ allergic rhinitis most likely suffers from …

A

Atopic Dermatitis (Eczema)

How well did you know this?
1
Not at all
2
3
4
5
Perfectly
122
Q

What are treatments for atopic dermatitis (eczema)? (5)

A
  1. avoid excessively hot/ dry environments
  2. avoid harsh soaps and detergents
  3. oral antihistamines
  4. regular use of emollients (to maintain hydration)
  5. steroid cream if severe (short term)
How well did you know this?
1
Not at all
2
3
4
5
Perfectly
123
Q

What is the best treatment for atopic dermatitis involving the face, eyelids, flexural areas if symptoms persist after avoiding irritants and using emollients?

A

topical calcineurin inhibitors (tacrolimus)

How well did you know this?
1
Not at all
2
3
4
5
Perfectly
124
Q

A pt presenting with wrist pain radiating to palmar surface of thumb and first 2 fingers, associated with numbness and tingling sensation in same distribution, worse at night and can awaken from sleep, provoked by flexing/ extending wrist or raising arms (typing, driving), improved by flicking wrist (flick sign) or warm running water, symptoms reproduced with full flexion at wrist with elbow in full extension or by tapping on wrist most likely suffers from …

A

Carpal Tunnel Syndrome

(risk factors: diabetes, rheumatoid arthritis, hypothryoidism, wrist trauma, obesity, end stage renal disease, pregnancy)

(due to median nerve entrapment in the carpal tunnel)

How well did you know this?
1
Not at all
2
3
4
5
Perfectly
125
Q

What is the best initial treatment for carpal tunnel syndrome?

A

nocturnal wrist splinting

followed by steroid injection followed by surgery
(also surgery if motor weakness or atrophy of thenar eminence)

How well did you know this?
1
Not at all
2
3
4
5
Perfectly
126
Q

What medications should a pt with a recent myocardial infarction be taking? (5)

A
  1. antiplatelet (aspirin and clopidogrel/ prasugrel/ ticagrelor)
  2. beta blocker (metoprolol/ atenolol/ bisoprolol except if severe asthma/ COPD/ CHF/ hypotension/ bradycardia)
  3. statin (lipid lowering)
  4. ACE inhibitor/ ARBs
  5. aldosterone antagonist (eplerenone if LV ejection fraction less than 40%)
How well did you know this?
1
Not at all
2
3
4
5
Perfectly
127
Q

What is the next best step in management of an intoxicated pt who was brought in for threatening suicide attempt or suicidal ideation but is dismissing attempt as due to intoxication?

A

keep for observation in ER and perform suicide assessment when pt is completely sober

How well did you know this?
1
Not at all
2
3
4
5
Perfectly
128
Q

What is the strongest predictive risk factor for suicide?

A

prior history of attempted suicide

How well did you know this?
1
Not at all
2
3
4
5
Perfectly
129
Q

What are the four protective factors for suicide?

A
  1. social support/ family consciousness
  2. pregnancy
  3. parenthood
  4. religion and participation in religious activities
How well did you know this?
1
Not at all
2
3
4
5
Perfectly
130
Q

What is the best anticoagulation regimen for a pregnant patient requiring anticoagulation throughout pregnancy (i.e. for mechanical valve presence)?

A

replace warfarin with unfractionated heparin or low molecular weight heparin during the first trimester (warfarin teratogenic at 6-12 weeks gestation) and prior to delivery (heparin easily reversible in cases of bleeding; otherwise on warfarin (for 2nd and 3rd trimester)

How well did you know this?
1
Not at all
2
3
4
5
Perfectly
131
Q

What is the most important factor in perioperative adverse drug reaction in the elderly population?

A

multiple medication use

How well did you know this?
1
Not at all
2
3
4
5
Perfectly
132
Q

A child presenting with hypopigmented skin lesions, seizure like activity, developmental delay, and possible autistic like behavior most likely suffers from …

A

Tuberous Sclerosis

benign tumors throughout body: skin, CNS, heart, kidney

How well did you know this?
1
Not at all
2
3
4
5
Perfectly
133
Q

What diagnostic tests should be performed in a child with suspected tuberous sclerosis? (5)

A
  1. thorough skin exam (ash leaf spots, angiofibromas of malar region, shagreen patches)
  2. fundoscopy
  3. brain MRI (glioneuronal hamartomas)
  4. EEG (for baseline/ seizure activity)
  5. abdominal imaging
How well did you know this?
1
Not at all
2
3
4
5
Perfectly
134
Q

What is the predominant cause of death in patients with tuberous sclerosis?

A

Progressive neurologic impairment (uncontrollable seizures, aspiration pneumonia, obstructive hydrocephalus)

How well did you know this?
1
Not at all
2
3
4
5
Perfectly
135
Q

When can low-risk pts (few CVD risk factors, controlled HTN, asymptomatic LV dysfunction, successful revascularization of clinically significant lesions > 50-60%) initiate/ resume sexual activity after a myocardial infarction?

A

once they can perform light intensity exercise without symptoms (within weeks of MI)

How well did you know this?
1
Not at all
2
3
4
5
Perfectly
136
Q

A pt presenting with nightmares, flashback of an event, avoids distressing thoughts/ feelings/ external reminders of event, have persistent feeling of horror/ anger/ guilt, have negative belief about self or world, decreased interest in activities, emotional detachment, amnesia of event, sleep disturbance, hypervigilence, and impaired concentration lasting for more than 1 month after exposure to a life threatening trauma most likely suffers from ..

A

Post Traumatic Stress Disorder (PTSD)

How well did you know this?
1
Not at all
2
3
4
5
Perfectly
137
Q

What is the best medical treatment for nightmares associated with PTSD (post-traumatic stress disorder)?

A

prazosin

How well did you know this?
1
Not at all
2
3
4
5
Perfectly
138
Q

An infant presenting with constipation followed by lethargy, poor sucking, weak cry and impaired gag reflex after being fed honey most likely suffers from ..

A

Infant Botulism

How well did you know this?
1
Not at all
2
3
4
5
Perfectly
139
Q

An HIV pt who has a recent history of severe pain in mouth, throat and epigastric region and then develops ocular pain, photophobia, scotomas (large glistening off-white lesions with indistinct borders on funduscopy) and fever most likely suffers from ..

A

Candida endophthalmitis

(risk factors: central venous catheter, TPN, broad spectrum antibiotics, prior abdominal surgery, neutropenia, steroid therapy, IV drug abuse)

How well did you know this?
1
Not at all
2
3
4
5
Perfectly
140
Q

What is the best medical therapy for candida endophthalmitis?

A

systemic amphotericin B or fluconazole

also vitrectomy if vitreal involvement

How well did you know this?
1
Not at all
2
3
4
5
Perfectly
141
Q

In a normal distribution graph, 68% of all observations lie within … standard deviations of the mean, 95% of all observations lie within …. standard deviations of the mean, and 99.7% of all observations lie within …standard deviations of the mean..

A

1 (68%); 2 (95%); 3 (99.7%)

How well did you know this?
1
Not at all
2
3
4
5
Perfectly
142
Q

What type of study involves a group of subjects being selected and their exposure status is initially determined before being followed for a certain period and observed for development of the outcome?

A

Prospective Cohort Study

How well did you know this?
1
Not at all
2
3
4
5
Perfectly
143
Q

What type of preventative medicine strategy is defined as the prevention of risk factors for a disease?

A

Primordial prevention

How well did you know this?
1
Not at all
2
3
4
5
Perfectly
144
Q

What type of preventative medicine strategy is defined as an action that attempts to halt the progression of a disease at its initial stage before irreversible pathological changes take place, thus preventing complications?

A

Secondary prevention

How well did you know this?
1
Not at all
2
3
4
5
Perfectly
145
Q

What type of preventative medicine strategy takes place when the disease process has advanced beyond early stage and all actions available are taken to limit impairments and disabilities?

A

Tertiary Prevention

How well did you know this?
1
Not at all
2
3
4
5
Perfectly
146
Q

What type of preventative medicine strategy is an action taken before a patient develops the disease and acts to prevent the occurrence of the disease itself?

A

Primary Prevention

How well did you know this?
1
Not at all
2
3
4
5
Perfectly
147
Q

… is when a test is able to diagnose more patients that actually have the disease (more true positives and less false negatives) such that less “sick” pts will have negative test results

A

Sensitivity

(sensitivity = a/ (a+c))

(associated with high negative predictive value)

How well did you know this?
1
Not at all
2
3
4
5
Perfectly
148
Q

… is when a test is able to identify pts who are actually healthy (more true negatives and less false positives) such that less “healthy” patients will have a positive test result

A

Specificity

(specificity = d/ b+d)

(SpIn= specificity rules in disease)

How well did you know this?
1
Not at all
2
3
4
5
Perfectly
149
Q

When determining which test to use as a screening test, what is more important for the initial test used as a screening test: specificity or sensitivity?

A

sensitivity (use more sensitive test first then more specific test as confirmation)

How well did you know this?
1
Not at all
2
3
4
5
Perfectly
150
Q

What physical activity is strictly prohibited in pregnancy?

A

scuba diving (risk of decompression injury and gas emboli in fetus)

How well did you know this?
1
Not at all
2
3
4
5
Perfectly
151
Q

What is the next best step in management of a pt presenting with symptomatic normocytic, normochromic anemia?

A

reticulocyte count (to distinguish between hemolytic disorder and disease of decreased RBC production)

low retic count: disease of decreased RBC production
high retic count: hemolysis

How well did you know this?
1
Not at all
2
3
4
5
Perfectly
152
Q

What is the typical interval for breastfeeding a newborn?

A

every 2-3 hours (no more than 4 hours overnight)

feed early if signs of hunger: rooting, hand/ finger sucking, arm movement toward mouth

How well did you know this?
1
Not at all
2
3
4
5
Perfectly
153
Q

What type of drug test is the preferred screening test to identify recent drug use in the emergency setting?

A

Urine immunoassay

antibodies to opioids, cocaine, marijuana, phencyclidine, amphetamines and alcohol used in last 1-3 days

How well did you know this?
1
Not at all
2
3
4
5
Perfectly
154
Q

A girl presenting with cyclic pelvic/ abdominal pain with primary amenorrhea, appearance of bluish tissue bulging between labia and a small rounded palpable mass in the suprapubic area in the setting of secondary sexual characteristics adequately developed for age most likely suffers from ..

A

Imperforate Hymen (hematocolpos)

How well did you know this?
1
Not at all
2
3
4
5
Perfectly
155
Q

What is the best initial step in management of a pt with non-suicidal self-injury (cutting behavior)?

A

inform relatives that pt is not in danger, screen for suicidal ideation and perform comprehensive psychiatric evaluation

How well did you know this?
1
Not at all
2
3
4
5
Perfectly
156
Q

An HIV pt not on medication presenting with rapidly progressive focal neurologic deficits such as cognitive impairment, hemiparesis, aphasia, ataxia, visual field deficits, seizure activity and a brain biopsy reveling oligodendrocytes with intranuclear inclusions, demyelination and astrogliosis most likely suffers from ..

A

Progressive Multifocal Leukoencephalopathy (PML)

tx: HAART therapy prolongs life for 2 years but not cure
(prognosis: without treatment, most die within 3-6 months)

How well did you know this?
1
Not at all
2
3
4
5
Perfectly
157
Q

What is the best step in management of a battery ingestion in which the battery is lodged in the esophagus? What is the best step in management of a battery ingestion in which the battery has made it to the stomach?

A

esophagus: immediate endoscopic removal
stomach: outpatient management (pass on own)

How well did you know this?
1
Not at all
2
3
4
5
Perfectly
158
Q

A pt with syncopal episode and an EKG showing short PR interval with delta wave (initial slow upstroke of QRS) and prolonged QRS most likely suffers from ..

A

tachyarrhythmia secondary to Wolff- Parkinson-White syndrome

pre-excitation syndrome due to accessory pathway

How well did you know this?
1
Not at all
2
3
4
5
Perfectly
159
Q

What is the best treatment for symptomatic pts with Wolff-Parkinson White syndrome?

A

catheter ablation therapy

How well did you know this?
1
Not at all
2
3
4
5
Perfectly
160
Q

What is the most common complication of bacterial conjunctivitis (unilateral eye redness with mucopurulent discharge)?

A

Keratitis (inflammation of cornea)

(presents as foreign body sensation, photophobia, corneal opacity/ ulceration especially in contact wearers)
(tx: antibiotics and urgent optho consult/ referral)

How well did you know this?
1
Not at all
2
3
4
5
Perfectly
161
Q

What is the best initial treatment for bacterial conjunctivitis (due to Strep penumoniae, Moraxella catarrhalis, H. influenzae)? (3)

A
  1. erythromycin ointment
  2. sulfa drops
  3. polymixin/ trimethoprim drops
How well did you know this?
1
Not at all
2
3
4
5
Perfectly
162
Q

When can a pt with bacterial conjunctivitis return to work/ school?

A

after 24 hours of antibiotic therapy (but should wait until discharge has resolved if can)

How well did you know this?
1
Not at all
2
3
4
5
Perfectly
163
Q

When is it acceptable to disclose genetic information to relatives?

A

if disclosure serves to ameilorate/ prevent a highly likely and forseeable harm to relative
(Huntington’s does not do this bc get disease regardless)

How well did you know this?
1
Not at all
2
3
4
5
Perfectly
164
Q

What is the most appropriate next step in management if a pt with positive genetic testing results for a serious hereditary illness refuses to disclose the information?

A

discuss with pt the reason for decision and document counseling and pt’s refusal to disclose in chart

How well did you know this?
1
Not at all
2
3
4
5
Perfectly
165
Q

What is the next best step in management of a pt who is competent and refusing appropriate health care?

A

identify possible barriers to accepting appropriate health care by asking pt why he/she is refusing

How well did you know this?
1
Not at all
2
3
4
5
Perfectly
166
Q

What is the finding in an aspiration of an affected joint associated with a diagnosis of gout?

A

negatively birefringent needle-shaped crystals

How well did you know this?
1
Not at all
2
3
4
5
Perfectly
167
Q

A pt on cyclosporin who presents with acute swelling of the first metatarsophalangeal joint associated with erythema, warmth and tenderness most likely suffers from ..

A

Gout

cyclosporin causes hyperuricemia via decreasing urate excretion

How well did you know this?
1
Not at all
2
3
4
5
Perfectly
168
Q

What is the best initial treatment of gout in a pt with renal failure or post- renal transplant?

A

intra-articular glucocorticoids (avoids systemic effect of steroids) or increase dose of systemic steroids if on them

How well did you know this?
1
Not at all
2
3
4
5
Perfectly
169
Q

What is the best initial treatment for prolactinomas (erectile dysfunction, galactorrhea, sexual dysfunction, visual changes, amenorrhea, hot flashes, decreased bone density) regardless of size?

A

Dopaminergic receptor agonist (bromocriptine, cabergoline)

use trans-sphenoidal surgery if pt fails medical therapy especially if have significant neuro symptoms

How well did you know this?
1
Not at all
2
3
4
5
Perfectly
170
Q

What is the best initial step in management of a pt with symptomatic bradycardia?

A

IV atropine

How well did you know this?
1
Not at all
2
3
4
5
Perfectly
171
Q

What is the next best step in management of symptomatic bradycardia if there is no response to atropine?

A

transvenous cardiac pacing

How well did you know this?
1
Not at all
2
3
4
5
Perfectly
172
Q

A pt presenting with decreased hearing, history of aural irrigation for removal of cerumen, otalgia (ear pain), purulent drainage/ discharge from the ear (otorrhea), ear lobe tenderness, and visible granulation tissue on floor of external auditory canal at osseocartilaginous junction most likely suffers from ..

A

Malignant Otitis Externa

usually in elderly pt with diabetes, HIV pt, immunocompromised
(due to Pseudomonas aeruginosa)

How well did you know this?
1
Not at all
2
3
4
5
Perfectly
173
Q

What is the treatment for malignant otitis externa?

A

IV antibiotics initially then switched to oral to complete 6-8 weeks

  1. IV ciprofloxacin
  2. IV piperacillin/ ticarcillin with/ without aminoglycoside
  3. IV ceftazidime
How well did you know this?
1
Not at all
2
3
4
5
Perfectly
174
Q

A pt presents with fever, malaise, hydrophobia (water triggers pharyngeal spasms causing pt to fear drinking), aerophobia (fear of fresh air), agitation, ascending flaccid paralysis, aphasia, drooling and lethargy after an encounter with an animal most likely suffers from ..

A

Rabies

(bats especially is spelunking/ cave exploration, raccoons)

(once symptomatic, poor prognosis)

How well did you know this?
1
Not at all
2
3
4
5
Perfectly
175
Q

Why should selective estrogen receptor modulators (raloxifene, tamoxifen) be discontinued 4 weeks prior to surgery?

A

risk of venous thromboembolism

How well did you know this?
1
Not at all
2
3
4
5
Perfectly
176
Q

What other treatments should be given to a pt who is on chronic glucocorticoid therapy? (2)

A
  1. calcium
  2. vitamin D

(b/c longterm steroids increase risk of osteoporesis via decrease intestinal absorption of calcium, increase renal excretion of calcium and acclerated bone resorption)

How well did you know this?
1
Not at all
2
3
4
5
Perfectly
177
Q

A pt presenting with symmetric polyarthritis involving MCP and PIP joints, associated with morning stiffness lasting more than 30 minutes for several weeks most likely suffers from ..

A

Rheumatoid arthritis

dx: rheumatoid factor and anti-cyclic citrullinated peptide/ CCP, X-rays

How well did you know this?
1
Not at all
2
3
4
5
Perfectly
178
Q

What is the first line treatment for rheumatoid arthritis?

A

NSAIDs (ibuprofen)

(use disease modifying antirheumatic drugs if higher risk of disease progression: multiple inflamed joints, functional limitation, extraarticular disease, bone erosion on X-ray, markedly elevated RF, positive anti-CCP)

How well did you know this?
1
Not at all
2
3
4
5
Perfectly
179
Q

What is the first line disease modifying anti-rheumatic drugs (DMARDs) used for rheumatoid arthritis?

A

Methotrexate

How well did you know this?
1
Not at all
2
3
4
5
Perfectly
180
Q

What P-value signifies statistical significance in studies?

A

P- value less than 0.05

How well did you know this?
1
Not at all
2
3
4
5
Perfectly
181
Q

What is a major side effect of Ginko Biloba used for memory enhancement?

A

increased bleeding risk

bleeding and platelet dysfunction

How well did you know this?
1
Not at all
2
3
4
5
Perfectly
182
Q

What is a major side effect of Ginseng used for improved mental performance?

A

increased bleeding risk

How well did you know this?
1
Not at all
2
3
4
5
Perfectly
183
Q

What are major side effects of Saw Palmetto used for benign prostatic hyperplasia? (2)

A
  1. Hypertension

2. increased bleeding risk

How well did you know this?
1
Not at all
2
3
4
5
Perfectly
184
Q

What are major side effects of Black Cohosh used for post-menopausal symptoms (hot flashes, vaginal dryness)? (2)

A
  1. hepatic injury

2. increased bleeding risk

How well did you know this?
1
Not at all
2
3
4
5
Perfectly
185
Q

What are major side effects of St John’s Wort used for depression and insomnia? (2)

A
  1. drug interactions (with antidepressants causing serotonin syndrome, OCP, anticoagulants causing decreased INR, digoxin)
  2. hypertensive crisis
How well did you know this?
1
Not at all
2
3
4
5
Perfectly
186
Q

What is a major side effect of Kava kava used for anxiety and insomnia?

A

severe liver damage (hepatotoxicity)

How well did you know this?
1
Not at all
2
3
4
5
Perfectly
187
Q

What are major side effects of Licorice used for stomach ulcers and bronchitis/ viral infections? (2)

A
  1. hypertension (due to inhibition of 11-beta hydroxysteroid dehydrogenase leading to elevated cortisol)
  2. hypokalemia
How well did you know this?
1
Not at all
2
3
4
5
Perfectly
188
Q

What is a major side effect of Echinacea used for treatment and prevention of cold and flu?

A

anaphylaxis (more likely in asthmatics)

How well did you know this?
1
Not at all
2
3
4
5
Perfectly
189
Q

What are major side effects of ephedra used for treatment of cold and flu and for weight loss and improved athletic performance? (4)

A
  1. hypertension
  2. arrhythmia/ MI/ sudden death
  3. stroke
  4. seizure
How well did you know this?
1
Not at all
2
3
4
5
Perfectly
190
Q

What cell type is responsible for acoustic neuromas observed in patients with neurofibromatosis?

A

Schwann cells

How well did you know this?
1
Not at all
2
3
4
5
Perfectly
191
Q

What is the best initial screening test for Down’s syndrome detection during pregnancy?

A

integrated test

(ultrasound measurement of nuchal translucency thickness at 10 weeks, PAPP-A level in first trimester, and in second trimester alfa-fetaoprotein, HCG, unconjugated estriol, dimeric inhibin-A levels)

How well did you know this?
1
Not at all
2
3
4
5
Perfectly
192
Q

What is the most accurate test for detecting Down’s Syndrome in a fetus during pregnancy?

A

fetal karyotyping from CVS or amniocentesis

How well did you know this?
1
Not at all
2
3
4
5
Perfectly
193
Q

What mode of inheritance is associated with all children of an affected mother being affected; however no children of an affected father being affected?

A

Mitochondrial

How well did you know this?
1
Not at all
2
3
4
5
Perfectly
194
Q

What is the next best step in management for a pt with short duration syncopal episodes triggered by prolonged standing/ emotional distress/ painful stimuli and associated with prodrome with dizziness/ nausea/ pallor/ diaphoresis/ abdominal pain/ generalized sense of warmth if an EKG is normal and there is no orthostatic hypotension?

A

no further testing

Vasovagal Syncope is clinical diagnosis; can use tilt table test if uncertain

How well did you know this?
1
Not at all
2
3
4
5
Perfectly
195
Q

Is a retroverted uterus a possible etiologic cause of spontaneous abortions in a woman experiencing recurrent miscarriages?

A

No

How well did you know this?
1
Not at all
2
3
4
5
Perfectly
196
Q

What is the next best step of management for a female pt older than 45 years old who experiences periods of amenorrhea followed by irregular unpredictable bleeding (anovulatory bleeding) with associated hot flashes and sleeping difficulties?

A

Endometrial Biopsy

symptoms suggestive of menopausal transition/ perimenopause
(to assess for endometrial hyperplasia and cancer)

How well did you know this?
1
Not at all
2
3
4
5
Perfectly
197
Q

A pt presenting with shoulder pain associated with ipsilateral ptosis, miosis, enophthalmos and anhidrosis, as well as weight loss, supraclavicular lymph node enlargement, and weakness/ atrophy of intrinsic hand muscles along with pain and paresthesisa of 4th finger, 5th finger, medial arm, and forearm most likely suffers from …

A

Pancoast Tumor

(superior pulmonary sulcus tumor is subset of non-small cell lung cancer and affects paravertebral sympathetic chain and inferior cervical ganglion)

How well did you know this?
1
Not at all
2
3
4
5
Perfectly
198
Q

What is the most worrisome findings in the setting of a pt having a Pancoast Tumor?

A

low back pain and asymmetric lower-extremity deep tendon reflexes (hyper-reflexia on affected side)

(suggest spinal cord compression)

How well did you know this?
1
Not at all
2
3
4
5
Perfectly
199
Q

A pt with history of atherosclerotic disease (cartoids, coronary, HTN, diabetes)/ vasculitis/ hematologic disease presenting with painless acute vision loss in 1 eye, an afferent pupillary defect, retinal whitening/ cherry red spot in macula and diffuse ischemic retinal pallor on funduscopy most likely suffers from ..

A

Central Retinal Artery Occlusion

cherry red spot due to preserved circulation via posterior ciliary arteries

How well did you know this?
1
Not at all
2
3
4
5
Perfectly
200
Q

What is the treatment for central retinal artery occlusion? (4)

A
  1. urgent ophthalmology consult
  2. lower intraocular pressure (ocular massage, anterior chamber paracentesis, IV acetazolamide, mannitol)
  3. intra-arterial thrombolytics if needed
  4. long term atherosclerosis risk factor modification (statin, antiplatelet agent)
How well did you know this?
1
Not at all
2
3
4
5
Perfectly
201
Q

A pt presenting with acute painless vision loss in 1 eye and tortuous and dilated veins, diffuse hemorrhages, disk swelling and cotton wool spots on funduscopy most likely suffers from ..

A

Central Retinal Vein Occlusion

How well did you know this?
1
Not at all
2
3
4
5
Perfectly
202
Q

How should difficult patient encounters due to situations beyond the physician’s control (i.e. long waiting times) be managed?

A

apologize, acknowledge the delay and pt’s frustration, provide brief explanation

How well did you know this?
1
Not at all
2
3
4
5
Perfectly
203
Q

A young pt or pt with history of Paget’s disease who develops hypercalcemia in the setting of low PTH and normal vitamin D levels (1,25 Vitamin D may be low) while in the hospital most likely has hypercalcemia due to ..

A

Immobilization

excessive bone resorption leads to hypercalcemia

How well did you know this?
1
Not at all
2
3
4
5
Perfectly
204
Q

How do you correct the calcium level in the setting of hypoalbuminemia?

A

for every 1 g/dl decrease in serum albumin below 4 g/dl, add 0.8 mg to the total calcium level

(corrected calcium = measured total calcium + 0.8( 4- measured serum albumin))

How well did you know this?
1
Not at all
2
3
4
5
Perfectly
205
Q

What is the treatment for akathisia (subjective feeling of inner restlessness and the urge to move) due to antipsychotic medications?

A

Beta blocker (propanolol)

How well did you know this?
1
Not at all
2
3
4
5
Perfectly
206
Q

What is the best initial step in management to establish the cause of a syncopal episode?

A

through history and physical with an EKG

How well did you know this?
1
Not at all
2
3
4
5
Perfectly
207
Q

What is the next best step in management of a patient presenting with substernal chest discomfort that is relieved by nitroglycerin and has an EKG with findings of ST depression suggesting acute coronary syndrome (unstable angina/ non-ST elevation MI)? (6)

A
  1. dual antiplatelet therapy (aspirin and P2Y12 receptor blocker- clopidogrel/ prasugrel/ ticagrelor)
  2. nitrates
  3. beta blockers
  4. statins
  5. anticoagulant therapy (unfractionated/ LMW heparin, fondaparinux)
  6. oxygen
How well did you know this?
1
Not at all
2
3
4
5
Perfectly
208
Q

What is the best initial step in management for a female pt who has just become menopausal to prevent osteoporesis? (3)

A
  1. regular weight bearing and anti-gravity exercises
  2. 1200 mg of calcium (dietary and/or supplemented)
  3. 600-800 IU of vitamin D (dietary and/or supplemented)
How well did you know this?
1
Not at all
2
3
4
5
Perfectly
209
Q

When should postmenopausal women aged less than 65 years old undergo a bone density screening? (5)

A
  1. body weight less than 58 kg (127 lbs)
  2. chronic steroid use
  3. smoking
  4. malabsorptive disorders (cystic fibrosis)
  5. parental hx of hip fracture or personal hx of low-impact fracure
How well did you know this?
1
Not at all
2
3
4
5
Perfectly
210
Q

A pt presenting with expanding, painless lesions with central clearing, headache, fatigue, malaise, myalgias, and fever after spending time in woods most likely suffers from …

A

Early Limited Lyme Disease

(dx with initial ELISA followed by confirmatory western blot if history and physical not clear)

(transmitted by brown tick)

How well did you know this?
1
Not at all
2
3
4
5
Perfectly
211
Q

What is the treatment for Early Limited Lyme Disease?

A

oral doxycycline/ amoxicillin/ cerfuroxime

How well did you know this?
1
Not at all
2
3
4
5
Perfectly
212
Q

A pt presenting with history of cramping with volitional movements in early morning (i.e. stretching in bed), asymmetric weakness in distal extremity progressing to muscle wasting, atrophy, twitching and fasciculations that then spreads to multiple limbs with associated hyperactive deep tendon reflexes, spastic resistance to passive movement and difficulty chewing and swallowing most likely suffers from …

A

Amyotrophic lateral sclerosis (ALS)

How well did you know this?
1
Not at all
2
3
4
5
Perfectly
213
Q

What treatment has been shown to be beneficial in pts with amyotrophic lateral sclerosis (ALS)?

A

Riluzole

glutamate inhibitor prolongs survivial and delays tracheostomy

How well did you know this?
1
Not at all
2
3
4
5
Perfectly
214
Q

What is the risk of developing type 1 diabetes in an offspring of a parent with type 1 diabetes?

A

if mother has it: 3%

if father has it: 6%

How well did you know this?
1
Not at all
2
3
4
5
Perfectly
215
Q

What is the next best step in management for medication induced psychosis once the minimum effective dose of anti-parkinsonian medication has been established in a pt with parkinson’s disease (i.e. development of psychosis while on minimum effective dose of levodopa)?

A

add quetiapine (low dose antipsychotic)

second line is clozapine

How well did you know this?
1
Not at all
2
3
4
5
Perfectly
216
Q

A pt presenting with asymptomatic mild hypercalcemia, normal vitamin D level, PTH level within normal range but at upper limit of normal, urine calcium less than 100 mg/day and a family history of hypercalcemia most likely suffers from ..

A

Familial Hypocalciuric Hypercalcemia

mutation in calcium sensing receptor in parathyroid gland and kidneys

How well did you know this?
1
Not at all
2
3
4
5
Perfectly
217
Q

A pt presenting with asymptomatic mild hypercalcemia, normal vitamin D level, PTH level within normal range but at upper limit of normal, urine calcium more than 200 mg/day most likely suffers from ..

A

Primary Hyperparathyroidism

How well did you know this?
1
Not at all
2
3
4
5
Perfectly
218
Q

What are the procedures that warrant antibiotics prophylaxis in patients with high risk cardiac conditions? (4)

A
  1. dental procedures w/ manipulation of gingiva/ periapical region of tooth/ perforation of oral mucosa
  2. respiratory tract procedures with incision/ biopsy
  3. GI or GU procedure in pt with established infection
  4. procedures on infected skin/ musculoskeletal tissue
How well did you know this?
1
Not at all
2
3
4
5
Perfectly
219
Q

What is the next best step in management for a pt diagnosed with familial adenomatous polposis after finding more than 100 colonic adenomatous polps on colonoscopy (especially in setting of positive family history)?

A

proctocolectomy or total colectomy with ileoanal anastomosis

otherwise inevitably progresses to colorectal cancer by 50 years old

How well did you know this?
1
Not at all
2
3
4
5
Perfectly
220
Q

A pt presenting with asymmetric oligoarthritis that is inflammatory (elevated WBC on synovial aspiration) yet sterile (culture negative), circinate balanitis (painless, shallow penile ulcers)/ uveitis/ conjunctivitis/ oral ulcers/ keratoderma blennorrhagicum/ achilles enthesitis (pain at ligament/ tendon insertion site), dactylitis in the setting of a history of chlamydia infection 1-4 weeks ago most likely suffers from ….

A

Reactive Arthritis

(can also be associated with gastroenteritis from Shigella, Salmonella, Clostridium, Yersinia, Campylobacter)

(associated with positive HLA- B27)

(can’t see, can’t pee, can’t climb a tree)

How well did you know this?
1
Not at all
2
3
4
5
Perfectly
221
Q

What is the next best step in management in a pt with new onset reactive arthritis that have a history of chlamydia infection?

A

repeat chlamydia testing (urinary nucleic acid amplification for Chlamydia trachomatis)

How well did you know this?
1
Not at all
2
3
4
5
Perfectly
222
Q

What is the treatment for reactive arthritis from recurrent chlamydia infection?

A

antibiotics with naproxen (NSAID for symptomatic relief)

How well did you know this?
1
Not at all
2
3
4
5
Perfectly
223
Q

An african american pt presenting with dry cough, weight loss, fever, maculopapular rash, erythema nodosum (tender erythematous nodules on shin), bilateral hilar adenopathy and interstitial infiltrates on CXR, uveitis, lymphadenopathy, polyarthritis, hypercalcemia, suppressed PTH, increased urinary calcium, elevated alkaline phosphatase, hepatomegaly, and central diabetes insipidus most likely suffers from ..

A

Sarcoidosis

non-caseating granulomas

How well did you know this?
1
Not at all
2
3
4
5
Perfectly
224
Q

What is the cause of hypercalcemia in a pt with sarcoidosis?

A

calcitriol (1,25-OH Vitamin D) overproduction

(noncaseating granulomas produce 1-alpha hydroxylase which converts 25-OH Vitamin D to 1,25-OH Vitamin D to increased calcium absorption in GI tract)

How well did you know this?
1
Not at all
2
3
4
5
Perfectly
225
Q

What is the best treatment for hypercalcemia secondary to sarcoidosis?

A

glucocorticoids (steroids)

How well did you know this?
1
Not at all
2
3
4
5
Perfectly
226
Q

A pt presents with a persistent cough (lasting more than 5 days up to 3 weeks) after a respiratory illness with associated wheezing, rhonchi and chest wall tenderness but lacking fever and chills most likely suffers from ..

A

Acute Bronchitis

ts: symptomatic with NSAIDs and bronchodilators

How well did you know this?
1
Not at all
2
3
4
5
Perfectly
227
Q

What area of the heart is supplied by the left circumflex artery?

A

lateral and posterolateral wall of the left ventricle

affected if left main coronary artery occluded b/c branches off left main

How well did you know this?
1
Not at all
2
3
4
5
Perfectly
228
Q

What area of the heart is supplied by the left anterior descending artery?

A

anterior wall of the left ventricle

affected if left main coronary artery occluded b/c branches off left main

How well did you know this?
1
Not at all
2
3
4
5
Perfectly
229
Q

What area of the heart is supplied by the right coronary artery?

A
  1. inferoposterior wall of left ventricle

2. right ventricle

How well did you know this?
1
Not at all
2
3
4
5
Perfectly
230
Q

What is the next best step in management of a pt with signs of hepatic encephalopathy (hallucinations, wax and waning of consioucness) in the setting of spontaneous bacterial peritonitis (more than 250 neutrophils in ascitic fluid)?

A

lactulose (for hepatic encephalopathy) and antibiotics (for bacterial peritonitis)

How well did you know this?
1
Not at all
2
3
4
5
Perfectly
231
Q

What parameters are most useful in predicting a patient’s 90-day mortality from liver disease?

A
  1. bilirubin
  2. INR
  3. serum creatinine

(MELD score = 3.8 [Ln bilirubin] + 11.2 [Ln INR] + 9.6 [Ln creatinine])

How well did you know this?
1
Not at all
2
3
4
5
Perfectly
232
Q

What MELD score in a patient with liver disease correlates with a good outcome from placement of a transjugular intrahepatic portosystemic shunt (TIPS procedure)?

A

MELD score less than 14 (have high survival likelihood)

How well did you know this?
1
Not at all
2
3
4
5
Perfectly
233
Q

A pt presenting with diarrhea, fever, abdominal pain and leukocytosis in the setting of recent antibiotic use most likely suffers from ..

A

Clostridium difficile associated colitis

tx: metronidazole

How well did you know this?
1
Not at all
2
3
4
5
Perfectly
234
Q

What is the next best step if there is a high clinical suspicion for C. difficile colitis but laboratory testing is negative (negative PCR or enzyme immunoassay of stool)?

A

limited colonoscopy or sigmoidoscopy

detect pseudomembranous colitis- raised yellow plaques over the sigmoid mucosa

How well did you know this?
1
Not at all
2
3
4
5
Perfectly
235
Q

What are the most common antibiotics associated with causing C. difficile colitis? (4)

A
  1. fluoroquinolones (ciprofloxacin)
  2. enhanced spectrym penicillins
  3. cephalosporins
  4. clindamycin
How well did you know this?
1
Not at all
2
3
4
5
Perfectly
236
Q

What is the cervical cancer screening guidelines for patients age 21-29 years old?

A

Pap smear every 3 years

How well did you know this?
1
Not at all
2
3
4
5
Perfectly
237
Q

What is the cervical cancer screening guidelines for patients aged 30-65 years old?

A
  1. Pap smear every 3 years
    or
  2. Pap smear with HPV testing every 5 years
How well did you know this?
1
Not at all
2
3
4
5
Perfectly
238
Q

What is the cervical cancer screening guidelines for immunocompromised patients (HIV, lupus, organ transplant)?

A

start at onset of sexual intercourse performing every 6 months for 2 years and then annually

How well did you know this?
1
Not at all
2
3
4
5
Perfectly
239
Q

What is the next best step in management for a patient with clinical signs (gait disturbance- wide based and magnetic, dementia/ cognitive decline, urinary incontinence) and brain imaging (ventriculomegaly out or proportion to sulcal enlargment) suggestive of normal pressure hydrocephalus?

A

Miller Fisher test/ Lumbar tap test

(evaluate gait speed, stride length, verbal memory, visual attention before and after removal of 30-50 ml of CSF; if improvement after removal than will respond to VP shunt)

How well did you know this?
1
Not at all
2
3
4
5
Perfectly
240
Q

What is the best prognostic indicator for the severity of acute pancreatitis?

A

APACHE II score
(12 variables used at time of admission and daily)
(if score greater to or equal to 8, pt should have CT scan at 72 hours to evaluate for necrosis)

How well did you know this?
1
Not at all
2
3
4
5
Perfectly
241
Q

What is the next best step in management of a dog bite on the hand (not other places on body)?

A

cleaned, irrigated with normal saline, debrided and left open to drain with close observation
(high risk of infection, similar with any cat/ human bite and puncture wounds)

How well did you know this?
1
Not at all
2
3
4
5
Perfectly
242
Q

What intervention most significantly improves the quality of patient handoffs?

A

systemiatic sign out checklists and templates

(pt identifying info, DNR/ do not intubate status, hospital course and recent events, acuity/current conidtion, anticipatory info, plan and tasks by organ system)

How well did you know this?
1
Not at all
2
3
4
5
Perfectly
243
Q

A pt presenting with fatigue, paroxysmal nocturnal dyspnea, orthopnea, jugular venous distension, rales/ crackles, presence of S3, increased cardiac silhouette and pulmonary congestion on CXR, bilateral lower extremity edema, nocturnal cough, dyspnea on exertion and hepatomegaly most likely suffers from..

A

Congestive Heart Failure

dx: clinical- 2 major- goes up to CXR and 2 minor criteria

How well did you know this?
1
Not at all
2
3
4
5
Perfectly
244
Q

A pt presenting with pain and swelling behind an ear after a recent otitis media infection most likely suffers from

A

Acute mastoiditis

infection of mastoid air cells

How well did you know this?
1
Not at all
2
3
4
5
Perfectly
245
Q

A pt presenting with acute onset of upper abdominal pain radiating to the back, associated nausea and vomiting, blunting of left costophrenic angle on CXR most likely suffers from …

A

Acute Pancreatitis

dx: lipase and amylase levels

How well did you know this?
1
Not at all
2
3
4
5
Perfectly
246
Q

What is the treatment for acute pancreatitis?

A

aggressive IV fluids (prevent hypovolemia from third spacing), morphine (pain control), NPO

How well did you know this?
1
Not at all
2
3
4
5
Perfectly
247
Q

What is the next best step in management of a pt with acute pancreatitis after initiating treatment?

A

Obtain liver function tests (if not done already), abdominal ultrasound, lipid panel, assess alcohol use, calcium level

(assess cause of pancreatitis: gallstones, alcohol, hypertriglyceridemia)

How well did you know this?
1
Not at all
2
3
4
5
Perfectly
248
Q

What is the next best step in management of a pt with acute pancreatitis who was initially hemodynamically stable but deteriorates several days later?

A

contrast- enhanced CT scan of abdomen (assess for pancreatic necrosis or infection)

How well did you know this?
1
Not at all
2
3
4
5
Perfectly
249
Q

A pt with hyponatremia, low plasma osmolality, elevated urine osmolality and high urinary sodium (greater than 40 mEq/L) with normal renal, adrenal and thyroid function test most likely suffers from ..

A

Syndrome of Inappropriate ADH secretion (SIADH)

due to CNS disorders, HIV, tumors- small cell lung cancer, carbamazepine, cyclophosphamide, SSRIs, post-op pts

How well did you know this?
1
Not at all
2
3
4
5
Perfectly
250
Q

What antibiotics have an increased risk of seizure in the setting of renal insufficiency/ older age/ pre-existing CNS disease/ use of proconvulsant drugs?

A

beta-lactams and fluoroquinolones

penicillins, cephalosporins, monobactams, carbapenems- imipenem

How well did you know this?
1
Not at all
2
3
4
5
Perfectly
251
Q

An african american child presenting with painful swelling of the hands and feet (dactylitis or hand-foot syndrome), peripheral cyanosis, unlocalized pain, hemolytic anemia, howell-jolly bodies on smear and crying most likely suffers from ..

A

Vaso-Occlusive Painful Crisis of Sickle Cell Disease

dx: hemoglobin electrophoresis

How well did you know this?
1
Not at all
2
3
4
5
Perfectly
252
Q

What is the best screening test for suspected sarcoidosis (primarily in African American, Puerto Ricans, Irish, Scandinavians)?

A

Chest X-ray (bilateral hilar adenopathy/ mediastinal widening and interstitial infiltrates)

How well did you know this?
1
Not at all
2
3
4
5
Perfectly
253
Q

What is the best test to confirm sarcoidosis?

A

biopsy if easily accessible lesion

fiberoptic bronchoscopy with transbronchial lung biopsy if no accessible lesion

How well did you know this?
1
Not at all
2
3
4
5
Perfectly
254
Q

In a non-inferiority trial, if the test variable’s mean and confidence interval is to the right of the non-inferiority margin line, what does that mean?

A

the test drug is non-inferior to the control drug

new drug is not unacceptably worse than the control drug in reference to the variable

How well did you know this?
1
Not at all
2
3
4
5
Perfectly
255
Q

In a non-inferiority trial, if the test variable’s confidence interval crosses the zero line, what does that mean?

A

the test drug is not superior to the control drug

(if confidence interval is to right of zero line, test drug is superior to control)
(if confidence interval is to left of zero line, test drug is inferior to control)

How well did you know this?
1
Not at all
2
3
4
5
Perfectly
256
Q

A pt preseting with small, rough, erythematous and keratotic papules on sun-exposed areas that show signs of chronic photodamage (dyspigmentation, wrinkling, thinning, telangiectasia) in the setting of spending lots of one’s life in the sun most likely suffers from ..

A

Actinic Keratosis

(tx: liquid nitrogen cryotherpy, surgical excision, curettage)

(biopsy if diagnosis unclear/ lesion greater than 1 cm in diameter/ lesion indurated/ ulceration rpesent/ tenderness present/ rapidly growing lesion/ fails to respond to therapy)

How well did you know this?
1
Not at all
2
3
4
5
Perfectly
257
Q

What disease is there an increased risk for if a pt has actinic keratosis?

A

Squamous Cell carcinoma

How well did you know this?
1
Not at all
2
3
4
5
Perfectly
258
Q

A pt presenting with an enlarging fleshy nodule with ulceration on sun-exposed skin in the setting of spending lots of one’s life in the sun most likely suffers from..

A

Basal Cell Carcinoma

How well did you know this?
1
Not at all
2
3
4
5
Perfectly
259
Q

A young female athlete presenting with chronic anterior knee pain that worsens with squatting/ running/ prolonged sitting/ using the stairs (activities that require quadricep contraction) and has crepitation and pain on extension of the knee with anterior patellar compression most likely suffers from ..

A

Patellofemoral pain syndrome

tx: exercises to stretch and strengthen the thigh muscles and avoid activities that aggravate pain

How well did you know this?
1
Not at all
2
3
4
5
Perfectly
260
Q

An athlete who plays sports that involve alot of jumping (volleyball/ basketball) presenting with episodic anterior knee pain and tenderness at the inferior patella most likely suffers from ..

A
Patellar tendonitis
(jumper's knee)
How well did you know this?
1
Not at all
2
3
4
5
Perfectly
261
Q

An adolescent athlete who recently underwent a growth spurt presenting with anterior knee pain that worsens while playing sport and is relieved by rest, and has tenderness and swelling at the tibial tubercle most likely suffers from …

A

Osgood-Schlatter syndrome

How well did you know this?
1
Not at all
2
3
4
5
Perfectly
262
Q

A pt presenting with periodic vertigo (room spinning), unilateral hearing loss and tinnitus most likely suffers from …

A

Meniere’s syndrome

How well did you know this?
1
Not at all
2
3
4
5
Perfectly
263
Q

An elderly pt who withdraws from social life, avoids leaving home to avoid conversation, listens to the radio/ TV at high volumes, and failing the whispered voice test most likely suffers from ..

A

Presbycusis

age-related hearing loss

How well did you know this?
1
Not at all
2
3
4
5
Perfectly
264
Q

A pt presenting with joint pain and restriction of joint movement, elevated alkaline phosphatase, normal calcium level, thickening of outer cortex of the bone on X-ray and increased uptake on technetium bone scan most likely suffers from …

A

Paget’s Disease of bone

tx: bisphosphonates
(dx: radionuclide bone scan)

How well did you know this?
1
Not at all
2
3
4
5
Perfectly
265
Q

What is the treatment for Paget’s Disease and when is it indicated? (5)

A

Bisphosphonates (alendronate, risedronate, pamidronate)

  1. intolerable pain
  2. involves weight bearing bones
  3. neuro involvement
  4. hypercalcemia/ hypercalciuria
  5. CHF
How well did you know this?
1
Not at all
2
3
4
5
Perfectly
266
Q

A pregnant pt presenting with pruritic erythematous papules within the striae gravidum most likely suffer from ..

A

Papular urticarial papules and plaques of pregnancy (PUPPP)

tx: topical steroids

How well did you know this?
1
Not at all
2
3
4
5
Perfectly
267
Q

A pregnant pt presenting abdominal pruritis that develops into papules and vesicles surrounding the umbilicus most likely suffers from …

A

Herpes gestationis (pemphigoid gestationis)

autoimmune disorder

How well did you know this?
1
Not at all
2
3
4
5
Perfectly
268
Q

What is the best treatment for herpes gestationis (pemphigoid gestationis)?

A

corticosteroids (use systemic if fail topical)

How well did you know this?
1
Not at all
2
3
4
5
Perfectly
269
Q

What is the next best step in management of an asymptomatic pt with an incidental findings of Actinomyces while using an intrauterine device (IUD)?

A

advise pt to call if she develops pelvic infection symptoms

Actinomyces is normal flora of vagina but is associatd with infection in IUD users

How well did you know this?
1
Not at all
2
3
4
5
Perfectly
270
Q

What is the first line treatment for primary dysmenorrhea (crampy lower abdominal pain with the onset of menstrual bleeding, may have associated nausea and vomiting)?

A

NSAIDs (naproxen)

How well did you know this?
1
Not at all
2
3
4
5
Perfectly
271
Q

What is the second line treatment for primary dysmenorrhea (crampy lower abdominal pain with the onset of menstrual bleeding, may have associated nausea and vomiting)?

A

combined hormonal contraceptive pills

if fail to improve, evaluate for secondary causes

How well did you know this?
1
Not at all
2
3
4
5
Perfectly
272
Q

A pt presenting with a severe and sudden onset headache different from previous headache patterns (or described as worst headache of life) associated with nausea, vomiting, brief loss of consciousness, focal neuro deficits or meningismus especially in the setting of having hypertension/ being a smoker or heavy alcohol drinker most likely suffers from …

A

Subarachnoid Hemorrhage

due to ruptured saccular aneursym

How well did you know this?
1
Not at all
2
3
4
5
Perfectly
273
Q

What is the preferred initial test for establishing a diagnosis of subarachnoid hemorrhage?

A

Head CT without contrast

if negative or equivocal, then do lumbar puncture- xanthochromia

How well did you know this?
1
Not at all
2
3
4
5
Perfectly
274
Q

If a head CT without contrast is negative but history and physical is suggestive of subarachnoid hemorrhage, what is the next best step in management of the pt?

A

Lumbar puncture (assess for elevated opening pressure, xanthrochromia- pink/yello tint of CSF due to hemoglobin degradation products), consistently elevated RBCs in all tubes)

How well did you know this?
1
Not at all
2
3
4
5
Perfectly
275
Q

A pt with the history of celiac sprue presenting with abdominal pain, weight loss, diarrhea, and anemia despite being adherent to gluten-free diet most likely suffers from …

A

Intestinal T cell Lymphoma

How well did you know this?
1
Not at all
2
3
4
5
Perfectly
276
Q

What treatment in a pregnant woman lowers the risk of having an infant with spina bifida?

A

folic acid

How well did you know this?
1
Not at all
2
3
4
5
Perfectly
277
Q

A newborn with muscular weakness of the lower extremities and the presence of a protrusion of tissue in the midline lumbar region consisting of a sac of fluid most likely suffers from …

A

Myelomeningocele

no imaging, immediate neurosurgical eval and surgical closure within 24-48 hours

How well did you know this?
1
Not at all
2
3
4
5
Perfectly
278
Q

A pt with skin that has a leathery appearance with coarse, deep wrinkles and furrows, variable pigmentation, clusters of dilated capillaries and venules, and scattered flesh-colored scaly patches with slightly irregular margins most likely suffers from …

A

Photoaging

tobacco smoking is associated

How well did you know this?
1
Not at all
2
3
4
5
Perfectly
279
Q

What is the best treatment for photoaging of skin?

A

tretinoin (all-trans retinoic acid)

How well did you know this?
1
Not at all
2
3
4
5
Perfectly
280
Q

What is the next best step in management for a pt diagnosed with rhabdomyolysis (muscle tenderness, increased muscle tone, renal failure, hyperkalemia, hypocalcemia, hyperphosphatemia, elevated creatine kinase in the setting of cocaine/ alcohol intoxication/ severe trauma/ neuroleptic malignant syndrome)?

A

aggressive IV isotonic saline (followed by alkalinication of urine)

How well did you know this?
1
Not at all
2
3
4
5
Perfectly
281
Q

What is the next best step in management of a pt with loud snoring without other alarming symptoms (daytime hypersomnolence, nocturnal choking/ gasping/ apnea with oxygen desaturations, hypertension)?

A

lose weight, stop smoking, avoid alcohol near bedtime

How well did you know this?
1
Not at all
2
3
4
5
Perfectly
282
Q

What is the first line treatment for Raynaud phenomenon (fingers get numb and turn blue when exposed to cold)?

A

Dihydropyridine calcium channel blockers (nifedipine and amlodipine) and diltiazem

How well did you know this?
1
Not at all
2
3
4
5
Perfectly
283
Q

What is the next best step in the treatment of a pt with Raynaud’s phenomenon who is resistant to treatment with calcium channel blockers and has arthralgias and myalgias?

A

obtain ANA and rheumatoid factor (along with chemistries U/A and complement levels)

How well did you know this?
1
Not at all
2
3
4
5
Perfectly
284
Q

What are favorable prognostic factors for schizophrenia? (10)

A
  1. female
  2. older age of onset (older than 40)
  3. acute onset of symptoms
  4. identifiable precipitant
  5. mostly positive symptoms (hallucinations, delusions)
  6. presence of mood symptoms
  7. goo pre-morbid functioning
  8. no family history of schizophrenia
  9. good family support
  10. shorter duration of active symptoms
How well did you know this?
1
Not at all
2
3
4
5
Perfectly
285
Q

A young to middle aged woman presents with painless hepatomegaly and incidentally found elevated AST and ALT with normal alkaline phosphatase and normal bilirubin and may have a history of arthritis/ thyroiditis/ pericarditis most likely suffers from ..

A

Autoimmune hepatitis

dx: ANA and anti-smooth muscle antibodies

How well did you know this?
1
Not at all
2
3
4
5
Perfectly
286
Q

What is the antithrombotic guideline for patients with mechanical heart valves and no risk factors for thromboembolism?

A

aspirin (75-100 mg/day) with warfarin with INR goal of 2-3

How well did you know this?
1
Not at all
2
3
4
5
Perfectly
287
Q

What is the antithrombotic guideline for patients with mechanical heart valves and risk factors for thromboembolism (a-fib, severe LV dysfunction with ejection fraction less than 30%, prior thromboembolism, hypercoagulable state)?

A

aspirin (75-100 mg/day) with warfarin with INR goal of 2.5-3.5

How well did you know this?
1
Not at all
2
3
4
5
Perfectly
288
Q

A pt presenting with progressively worsening back pain with morning stiffness which improves with exercise for more than 3 months, reduced range of forward flexion of lumbar spine on Schober testing, and reduced chest expansion most likely suffers from ..

A

Ankylosing Spondylitis

exercise improves overall functional status, no reduced life expectancy

How well did you know this?
1
Not at all
2
3
4
5
Perfectly
289
Q

What is the best initial step in a pt with suspected ankylosing spondylitis?

A

X-ray of sacroiliac joint (sacrolitis, erosion of ischial tuberosity and iliac crest, squaring of vertebral bodies)

How well did you know this?
1
Not at all
2
3
4
5
Perfectly
290
Q

What is used to monitor the disease progression in a pt with ankylosing spondylitis?

A

X-rays (AP and lateral views of lumbar spine, lateral view of cervical spine, pelvic X-ray) and acute phase reactants

How well did you know this?
1
Not at all
2
3
4
5
Perfectly
291
Q

What are extraarticular manifestations of ankylosing spondylitis? (4)

A
  1. Restrictive lung disease (due to limited costovertebral joint motion & development of apical pulmonary fibrosis)
  2. anterior uveitis
  3. aortic regurgitation
  4. IgA nephropathy
How well did you know this?
1
Not at all
2
3
4
5
Perfectly
292
Q

What screening test should be performed in a pt who is diagnosed with familial adenomatous polyposis (FAP)?

A

upper GI endoscopy (to assess for extracolonic neoplasms such as gastric and duodenal adenomas/ carcinomas)

How well did you know this?
1
Not at all
2
3
4
5
Perfectly
293
Q

What is the next best step of management for a female pt presenting with spontaneous, unilateral nipple discharge that is guaiac positive/ grossly bloody who may have the presence of paplable lump/ skin changes?

A

Mammography with ultrasound)

to assess for possible papillary tumor- most common cause of pathologic nipple discharge

How well did you know this?
1
Not at all
2
3
4
5
Perfectly
294
Q

What is the next best step of management for a female pt presenting with bilateral/ multiple duct involving nipple discharge that is milky and nonbloody and lacking palpable lump/ skin changes? (3)

A
  1. urine beta-HCG
  2. TSH
  3. prolactin
How well did you know this?
1
Not at all
2
3
4
5
Perfectly
295
Q

What is the first line therapy for acute gout attack?

A

NSAIDs (indomethacin or ibuprofen) or colchicine

How well did you know this?
1
Not at all
2
3
4
5
Perfectly
296
Q

What is the best anti-hypertensive medication for treatment of hypertension in a pt with gouty arthritis?

A

ARBs (angiotensin receptor block- “sartans”)

has uricosuric effect and antihypertensive

How well did you know this?
1
Not at all
2
3
4
5
Perfectly
297
Q

What is the best treatment for a pt presenting with mild symptoms of acute otitis externa (minor discomfort, pruritus and minimal canal edema)?

A

gently clean canal using wire loop followed by topical acidifiers (acetic acid, domeboro)

(can add topical steroids to relieve itching and pain)

How well did you know this?
1
Not at all
2
3
4
5
Perfectly
298
Q

What is the best treatment for a pt presenting with moderate to severe symptoms of acute otitis externa (moderate pain, pruritis, partial/ complete canal occlusion from edema)?

A

gently clean canal using wire loop followed by topical antibiotics, wick placement if canal is completely occluded

(can add topical steroids to relieve itching and pain)

How well did you know this?
1
Not at all
2
3
4
5
Perfectly
299
Q

A pt presenting with ear pain, ear pruritis, erythema and edema of ear canal, and discharge along with pain caused by tragal pressure or traction on the auricle most likely suffers from …

A

Otitis Externa

swimmer’s ear due to Pseudomonas or S. aureus

How well did you know this?
1
Not at all
2
3
4
5
Perfectly
300
Q

What is the best preoperative management of type 1 diabetic during labor and cesarean section?

A

normal insulin dose on the night before surgery followed by insulin drip and infusion of D5 1/2 NS with 40 mEq of KCl to keep the blood glucose below 160 mg/dl during procedure

(insulin requirement significantly decreases following delivery of pregnant diabetic)

How well did you know this?
1
Not at all
2
3
4
5
Perfectly
301
Q

What is the formula for relative risk (RR)?

A

relative risk = risk of exposed group/ risk of unexposed group

(risk ratio comparing risk of an outcome among the exposed to that among the unexposed)

How well did you know this?
1
Not at all
2
3
4
5
Perfectly
302
Q

What is the formula for attributable risk percent (ARP)?

A

ARP = (risk in exposed - risk in unexposed) / risk in exposed

ARP = (relative risk -1)/ relative risk

(measure of excess risk and estimates the proportion of disease among exposed subjects that is attributed to exposure status)

How well did you know this?
1
Not at all
2
3
4
5
Perfectly
303
Q

What is the formula for population attributable risk percent (PARP)?

(Q ID: 5206)

A

PARP = (risk in total population - risk in unexposed)/ risk in total population

risk in total population = (risk of exposed) (proportion of exposure in population) + (risk of unexposed) (proportion of exposure in population)

PARP = (prevalence) (relative risk - 1)/ [ (prevalence) (relative risk -1) + 1]

(measure of excess risk in the total population that is attributed to exposure status)

How well did you know this?
1
Not at all
2
3
4
5
Perfectly
304
Q

A pt with sudden onset confusion, lethargy, bradycardia, skin flushing, miosis, wheezing and garlic like odor of clothing most likely suffers from ..

A

Organophosphate poisoning

tx: atropine and pralidoxime- cholinesterase activator

How well did you know this?
1
Not at all
2
3
4
5
Perfectly
305
Q

What diagnostic test is used to confirm the diagnosis of organophosphate poisoning?

A

RBC cholinesterase activity level

used to diagnosis and assess severity

How well did you know this?
1
Not at all
2
3
4
5
Perfectly
306
Q

What medical condition is saw palmetto most commonly used to treat?

A

Benign Prostatic Hyperplasia (BPH)

improves urinary symptoms scores, nocturia, peak urine flow

How well did you know this?
1
Not at all
2
3
4
5
Perfectly
307
Q

What medical condition is garlic most commonly used to treat?

A

Hypercholesterolemia

How well did you know this?
1
Not at all
2
3
4
5
Perfectly
308
Q

What medical condition is glucosamine and/ or chondroitin used to treat?

A

Osteoarthritis

How well did you know this?
1
Not at all
2
3
4
5
Perfectly
309
Q

What is the most effective parameter for preventing central-line associated bloodstream infections?

A

fully draping the body during insertion

(also use maximal barrier precautions, avoid femoral site, skin cleansing with chlorhexidine, prompt catheter removal when no longer needed)

How well did you know this?
1
Not at all
2
3
4
5
Perfectly
310
Q

What is the next best step in management of a pt who is not competent to make decisions and whose family members can not agree on the approach to the pt’s medical care?

A

involve hospital’s ethics committee in decision-making and mediation process

How well did you know this?
1
Not at all
2
3
4
5
Perfectly
311
Q

What is the next best step in management for a pt with a lab finding of isolated elevation of anti-HBc (total core antibody including IgG and IgM)?

A

measure IgM anti-HBc and liver enzymes

after repeating hep B serologies to rule out false positive

How well did you know this?
1
Not at all
2
3
4
5
Perfectly
312
Q

What does a finding of elevated liver enzymes and positive IgM anti-HBc in a pt who was previously found to have isolated elevation of anti-HBc (total core antibody) mean?

A

window period of acute hepatitis B virus infection

HBsAg has fallen but anti-HBs has not risen yet

How well did you know this?
1
Not at all
2
3
4
5
Perfectly
313
Q

What does a finding of normal liver enzymes and negative IgM anti-HBc in a pt who was previously found to have isolated elevation of anti-HBc (total core antibody) mean?

A

years after recovery from acute Hep B virus infection

How well did you know this?
1
Not at all
2
3
4
5
Perfectly
314
Q

What is the next best step in management for pt with abnormal liver function tests or evidence of chronic liver disease with negative IgM anti-HBc after having been found to have isolated elevation of anti-HBc (total core antibody)?

A

HBV DNA level (if detectable, chronic disease)

How well did you know this?
1
Not at all
2
3
4
5
Perfectly
315
Q

What is the main long-term complication of IVC (inferior vena cava) filter placement?

A

recurrent deep venous thrombosis (DVTs)

How well did you know this?
1
Not at all
2
3
4
5
Perfectly
316
Q

What is the initial best step in management for a pt with suspected idiopathic intracranial hypertension/ pseudotumor cerebri (obese women, headache, visual problems, on OCPs)?

A

ophthalmoscopic exam

assess for papilledema before brain imaging

How well did you know this?
1
Not at all
2
3
4
5
Perfectly
317
Q

What is the best treatment for a large retrosternal goiter that is causing compressive symptoms (dysphagia)?

A

surgical removal via cervical approach

How well did you know this?
1
Not at all
2
3
4
5
Perfectly
318
Q

What diagnostic test consists of hyperflexion of both wrists to reproduce characteristic symptoms of Carpal Tunnel Syndrome?

A

Phalen’s test

more sensitive than tinel sign

How well did you know this?
1
Not at all
2
3
4
5
Perfectly
319
Q

What diagnostic test consists of tapping or percussing over the region of the median nerve at the carpal tunnel to reproduce characteristic symptoms of Carpal Tunnel Syndrome?

A

Tinel sign

How well did you know this?
1
Not at all
2
3
4
5
Perfectly
320
Q

What diagnostic test can be used to confirm the diagnosis of carpal tunnel syndrome?

A

nerve conduction study
(shows slowed conduction along median nerve at level of carpal tunnel with normal conduction velocity proximally; along with reduced amplitudes)

How well did you know this?
1
Not at all
2
3
4
5
Perfectly
321
Q

What is the first line treatment for nongonococcal urethritis (dysuria, watery urethral discharge, many neutrophils with absence of bacteria on urethral swab in sexually active pt)?

A

Azithromycin 1 gm single dose (or doxycycline 100 mg PO BID for 7 days)

(chlamydia is most common cause)

How well did you know this?
1
Not at all
2
3
4
5
Perfectly
322
Q

What is the first line treatment for gonococcal urethritis (dysuria, purulent urethral discharge, many neutrophils with gram negative diplococci on urethral swab of sexually active pt)?

A

Injection of ceftriaxone

How well did you know this?
1
Not at all
2
3
4
5
Perfectly
323
Q

What is the likely cause of nongonococcal urethritis (dysuria, watery urethral discharge, many neutrophils with absence of bacteria on urethral swab in sexually active pt) in a pt who does not respond to azithromycin despite being compliant and not having repeated sexual exposures?

A

Trichomonas vaginalis

tx: metronidazole

How well did you know this?
1
Not at all
2
3
4
5
Perfectly
324
Q

A pt presenting with erythematous, scaling plaques and patches with indistinct margins on the scalp, central face, ears, chest, upper back, axilla, and pubic area along with dandruff of the scalp most likely suffers from …

A

Seborrheic dermatitis

should suspect HIV infection if new onset and severe

How well did you know this?
1
Not at all
2
3
4
5
Perfectly
325
Q

A pt presenting with a well circumscribed erythematous patch on skin/ mucous membrane that rapidly worsens and evolves into nodular patches marked by hemorrhage, ulceration and necrosis in a pt with an indwelling catheter/ central venous catheter or following infection of traumatic/ surgical infection most likely suffers from …

A

Ecythma gangrenosum

usually due to Pseudomonas aeruginosa bacteremia

How well did you know this?
1
Not at all
2
3
4
5
Perfectly
326
Q

What is the treatment for ecythema gangrenosum?

A

IV antibiotics (aminoglycoside with extended spectrum antipseudomonal penicillin/ antipseudomonal cephalosporin) for 7-10 days after catheter has been removed

(for 14 days after neutrophil count returns to normal in neutropenia pt)

How well did you know this?
1
Not at all
2
3
4
5
Perfectly
327
Q

What is the best management step for providing nutrition to a pt with oropharyngeal dysphagia who has failed oral feeding modifications?

A

gastrostomy tube feeding

How well did you know this?
1
Not at all
2
3
4
5
Perfectly
328
Q

… is when pt views another individual as perfect and flawless and is unable to tolerate any evidence to the contrary; often occuring in pts with borderline personality disroder when interacting with a “savior” (person who cared for him/her in time of crisis)

A

Primitive Idealization

type of splitting

How well did you know this?
1
Not at all
2
3
4
5
Perfectly
329
Q

What is the best treatment for borderline personality disorder?

A

dialectical behavior therapy

How well did you know this?
1
Not at all
2
3
4
5
Perfectly
330
Q

According toe Light’s criteria, what type of pleural fluid has a pleural fluid protein/ serum protein ratio greater than 0.5, a pleural fluid lactact dehydrogenase/ serum LDH ratio greater than 0.6, and/ or pleural fluid LDH > 2/3 the upper limit of normal for serm LDH?

A

Exudative Pleural Fluid

associated with autoimmue, esophageal rupture, infection, malinancy, pancreatitis, post CABG, pulmonary embolism

How well did you know this?
1
Not at all
2
3
4
5
Perfectly
331
Q

What is the primary treatment for hepatic hydrothorax (right sided transudative pleural effusion in pt with cirrhosis and ascites)?

A

Sodium restriction and Diuretics

second line is TIPS- transjugular intrhepatic portosystemic shunt

How well did you know this?
1
Not at all
2
3
4
5
Perfectly
332
Q

A pt with history of alcohol abuse presenting with flank dullness, positive fluid shift, hypoalbuminemia and transudative pleural effusion most likely suffers from …

A

Hepatic Hydrothorax

How well did you know this?
1
Not at all
2
3
4
5
Perfectly
333
Q

What is the first line, second line and third line treatment for nocturnal enuresis (urinary incontinence at night in a child 5 years old or older)?

A
  1. behavior modification (void immediately before bed, minimize fluid intake prior to bedtime, reward system) for 3- 6months
  2. enuresis alarm (best long term outcome)
  3. desmopressin (immediate improvement but high relapse)
How well did you know this?
1
Not at all
2
3
4
5
Perfectly
334
Q

A pt presenting with headache, nausea, malaise, and dizziness in the setting of other family members with similar symptoms most likely suffers from …

A

Carbon Monoxide Poisoning

dx: carboxyhemoglobin level

How well did you know this?
1
Not at all
2
3
4
5
Perfectly
335
Q

What is the next best step in management of a simple breast cyst if the mass completely disappears after fine needle aspiration and discharge is nonbloody?

A

ultrasound breast tissue in 4-6 weeks

if discharge bloody, perform mamogram and excision

How well did you know this?
1
Not at all
2
3
4
5
Perfectly
336
Q

A pt with Raynaud phenomenon and GERD who presents with severe hypertension and associated symptoms (headache, papilledema, blurred vision) most likely suffers from ..

A

Scleroderma renal crisis

due to renal arteriole wall thickening and narrowing leading to ischemia and renin-angiotensin activation

How well did you know this?
1
Not at all
2
3
4
5
Perfectly
337
Q

What is the first line treatment for scleroderma renal crisis?

A

captopril (ACE inhibitor)

add IV nitroprusside if malignant HTN with CNS manifestations

How well did you know this?
1
Not at all
2
3
4
5
Perfectly
338
Q

A pt presenting with right upper quadrant pain, fever and jaundice along with elevated levels of direct bilirubin and alkaline phosphatase without rise in aminotransferases most likely suffer from ..

A

Acute Cholangitis

tx: blood cultures, antibiotics, fluid hydration, vital sign monitoring

How well did you know this?
1
Not at all
2
3
4
5
Perfectly
339
Q

What is Reynold’s pentad?

A
  1. right upper quadrant pain
  2. jaundice
  3. fever
  4. confusion (due to sepsis)
  5. hypotension (due to sepsis)

(suggests suppurative cholangitis)

How well did you know this?
1
Not at all
2
3
4
5
Perfectly
340
Q

What is the pathophysiologic mechanism behind toxic shock syndrom?

A

bacterial exotoxin production leading to widespread T cell activiation (releasing cytokines)

How well did you know this?
1
Not at all
2
3
4
5
Perfectly
341
Q

What is the treatment for toxic shock syndrome?

A

remove foreign body, extensive IV fluids, antibiotics (clindamycin with antistaphylococcal antibiotic)

How well did you know this?
1
Not at all
2
3
4
5
Perfectly
342
Q

What are the two major risk factors for pediatric dental caries?

A
  1. nighttime bottle feeding (especially with juice)
  2. frequent exposure to sugary snacks

(use fluoride toothpast 2x a day, exam teeth as soon as erupt)

How well did you know this?
1
Not at all
2
3
4
5
Perfectly
343
Q

What is the treatment for diabetic ketoacidosis?

A

normal saline bolus given over an hour followed by insulin drip and potassium containing IV fluids (if pt has low to normal potassium level)

How well did you know this?
1
Not at all
2
3
4
5
Perfectly
344
Q

A pt presenting with glucose greater than 200 mg/dL, bicarbonate less than 15 mEq/L, venous pH less than 7.3 and an anion gap greater than 14 most likely suffers from ..

A

Diabetic Ketoacidosis

(polyuria, polydipsia, polyphagia, kussmaul respirations- deep rapid breathing, dehydration, vomiting, abdominal pain, fatigue, altered mental status)

How well did you know this?
1
Not at all
2
3
4
5
Perfectly
345
Q

What are pts with diabetic ketoacidosis at greatest risk of developing (which is why they should be admitted to ICU)?

A

cerebral edema

How well did you know this?
1
Not at all
2
3
4
5
Perfectly
346
Q

An immigrant presenting with weight loss, abdominal pain, and diarrhea who is found to have eosinophilia most likely suffers from ..

A

Helminthiasis
(infection with nematodes- round worm, hook worm, whipworm)

(tx: albendazole/ mebendazole)

How well did you know this?
1
Not at all
2
3
4
5
Perfectly
347
Q

What is the next best step in management of depression if the pt is intolerant of an SSRI?

A

Switch to another SSRI

different SSRIs can have different side effect profiles

How well did you know this?
1
Not at all
2
3
4
5
Perfectly
348
Q

A pt being treated for diabetic ketoacidosis who then develops fever, facial swelling, maxillary pain and tenderness, foul smelling nasal discharge, ophthalmoplegia and headache most likely suffers from ..

A

Mucomycosis (due to infection with Zygomycetes)

How well did you know this?
1
Not at all
2
3
4
5
Perfectly
349
Q

What is the treatment for mucomycosis?

A

debridement of necrotic tissue and amphotericin B

How well did you know this?
1
Not at all
2
3
4
5
Perfectly
350
Q

When is valvular surgery for mitral regurgitation advised?

A

if presence of symptoms (exertional dyspnea, fatigue, overt heart failure) and LV ejection fraction less than 60%

How well did you know this?
1
Not at all
2
3
4
5
Perfectly
351
Q

An HIV pt (or immunocompromised pt) who presents with multiple, discrete flesh to red colored papules with central umbilication on the face and/or trunk most likely suffers from ..

A

Cutaneous Cryptococcosis

(assess for systemic involvement via CXR, blood and CSF cultures, india ink stain of CSF, cryptococcal antigens of serum and CSF)

How well did you know this?
1
Not at all
2
3
4
5
Perfectly
352
Q

What is the diagnostic test of choice for cutaneous cryptococcosis?

A

biopsy of skin lesion using Periodic Acid Schiff and Gomori’s methenamine silver nitrate

(granulomatous inflammation with multinucleated giant cells, histiocytes, lymphocytes, neutrophils and plasma cells with numerous yeast like organisms)

How well did you know this?
1
Not at all
2
3
4
5
Perfectly
353
Q

What allergen is most frequently associated with asthma?

A

house dust mites

How well did you know this?
1
Not at all
2
3
4
5
Perfectly
354
Q

What disease/ disorder is suggested to be the cause of a young patient with diabetes, osteoporesis, hypertension, hypokalemia and metabolic alkalosis?

A

Cushing’s syndrome

(dx: 24 hour urinary free cortisol or overnight dexamethasone suppression test- cortisol level greater than 3 in the morning)

How well did you know this?
1
Not at all
2
3
4
5
Perfectly
355
Q

A child presenting with sudden acceleration of growth prior to puberty that is out of sequence (not testicular followed by penis followed by pubic hair followed by growth spurt) along with severe cystic acne most likely suffers from …

A

Severe androgen excess (likely secondary to late-onset congenital adrenal hyperplasia)

(precocious pseudo-puberty)

How well did you know this?
1
Not at all
2
3
4
5
Perfectly
356
Q

How long should a pt wait before starting an MAO inhibitor after discontinuing SSRI use?

A

at least 5 weeks

How well did you know this?
1
Not at all
2
3
4
5
Perfectly
357
Q

A pt presenting with mental status changes, autonomic dysregulation (diaphoresis, tachycardia, hypertension, hyperthermia), and neuromuscular hyperactivity (hyperreflexia, tremor, myoclonus, ocular clonus) in setting of using an SSRI and MAO inhibitor most likely suffers from …

A

Serotonin syndrome

tx severe cases with cyproheptadine

How well did you know this?
1
Not at all
2
3
4
5
Perfectly
358
Q

What is the formula for standardized incidence ratio (SIR)?

A

SIR = observed cases/ expected cases

(determines if occurrence of disease in a small population is high or low relative to an expected value derived from larger comparison population)

How well did you know this?
1
Not at all
2
3
4
5
Perfectly
359
Q

A pt who develops facial asymmetry after a recent upper respiratory infection most likely suffers from …

A

Bell’s palsy

How well did you know this?
1
Not at all
2
3
4
5
Perfectly
360
Q

A pt who presents with perioral numbness, muscle cramps, carpopedal spasm, positive chvostek sign (ipsilateral contraction of facial muscles on tapping of angle of jaw) and positive Trousseau’s sign (rapid development of carpopedal spasm on occlusion of blood supply to upper extremity) most likely suffers from …

A

Hypocalcemia

How well did you know this?
1
Not at all
2
3
4
5
Perfectly
361
Q

What is Hungry Bone Syndrome?

A

sudden withdrawal of PTH in pts with severe hyperparathyroidism causes increased influx of calcium into bone 2-4 days after parathyroid surgery

How well did you know this?
1
Not at all
2
3
4
5
Perfectly
362
Q

A pt presenting with early diastolic murmur at the left sternal border (aortic regurgitation), long extremities (especially arms), long fingers, tall stature, flat feet and scoliosis/ pectus deformity/ kyphosis and family member with similar problems most likely suffers from ..

A

Marfan’s syndrome

autosomal dominant defect in fibrillin-1
(ectopia lentis, aortic root dilation/ regurgitation/ dissection)

How well did you know this?
1
Not at all
2
3
4
5
Perfectly
363
Q

What diagnostic test is essential to detect the major cause of death in a pt with Marfan’s syndrome?

A

transthoracic ECHO (at diagnosis and every 6 months to assess aortic root and ascending aorta)

(replace aortic root if diameter greater than 50 mm)

How well did you know this?
1
Not at all
2
3
4
5
Perfectly
364
Q

What size induration on a tuberculin skin test is considered positive in a normal healthy pt? What size induration on tuberculin skin test is considered positive in moderate risk pt (immigrant, IV drug user, employee at high risk setting, kids less than 4 years old, homeless, prisoner)?

A

healthy: greater than 15 mm

high risk: greater than 10 mm

How well did you know this?
1
Not at all
2
3
4
5
Perfectly
365
Q

What size induration on a tuberculin skin test is considered positive in a high risk pt (HIV, recent contact of known TB case, CXR with nodular/ fibrotic changes, transplant, immunocompromised)?

A

greater than 5 mm

How well did you know this?
1
Not at all
2
3
4
5
Perfectly
366
Q

What is the next best step in management of a pt who presents with gross hematuria without signs of glomerular disease (red cell casts, dysmorphic red cells), older than 40 years old, and has a smoking history?

A

Cystoscopy (to visualize lower urinary tract) and CT urogram (to visualize upper urinary tract)
(assess for suspected urinary tract malignancy)

How well did you know this?
1
Not at all
2
3
4
5
Perfectly
367
Q

A young healthy pt presenting with lymphadenopathy and white plaques on the buccal mucosa and palate that can be easily scraped off revelaing hyperemia most likely suffers from …

A

Candidiasis secondary to HIV test

dx: gram stain/ KOH prep of scrapings and HIV test

How well did you know this?
1
Not at all
2
3
4
5
Perfectly
368
Q

What is the best step in management of hyperthyroidism during pregnancy?

A

use PTU (propylthiouracil) during 1st trimester (b/c methimazole is teratogenic) then switch to methimazole for second and third trimester (b/c PTU is hepatotoxic)

How well did you know this?
1
Not at all
2
3
4
5
Perfectly
369
Q

A pt who develops platelet reduction by more than 50% from baseline, arterial/venous thrombosis, necrotic skin lesions at heparin injection sites or anphylactoid reaction after 5-10 days after heparin administration most likely suffers from …

A

Type 2 heparin induced thrombocytopenia (HIT)

dx: serotonin release assy
(tx: stop heparin and use direct thrombin inhibitor- argatroban or fondaparinux)

How well did you know this?
1
Not at all
2
3
4
5
Perfectly
370
Q

When should warfarin treatment be started after a patient has developed HIT (heparin induced thrombocytopenia) from heparin use?

A

after treatment with non-heparin anticoagulant and platelet recovery to more than 150,000

How well did you know this?
1
Not at all
2
3
4
5
Perfectly
371
Q

What is the treatment for Lyme arthritis (usually affecting knee)?

A

28 days of doxycycline (if not pregnant and older than 8 years old) or amoxicillin

(good prognosis; cured after 6-12 months)

How well did you know this?
1
Not at all
2
3
4
5
Perfectly
372
Q

What is the recommendation for pilots taking Viagra?

A

wait at least 6 hours after taking viagra before a flight

due to disturbance in blue-green differentiation
(six hours from viagra to throttle time)

How well did you know this?
1
Not at all
2
3
4
5
Perfectly
373
Q

What is the next best step for a child with a foreign body visualized to be in the stomach on imaging?

A

no intervention

How well did you know this?
1
Not at all
2
3
4
5
Perfectly
374
Q

A pt who develops new or worsening respiratory symptoms which include respiratry distress, diffuse crackles, hypoxemia, bilateral alveolar infiltrate on CXR, and a PaO2/ FiO2 ratio less than 300 mmHg without signs of fluid overload most likely suffers from …

A

Acute Respiratory Distress Syndrome (ARDS)

How well did you know this?
1
Not at all
2
3
4
5
Perfectly
375
Q

What is the treatment for acute respiratory distress syndrome (ARDS)?

(Q ID 5307, 5308)

A

low-tidal volume ventilation (goal of plateau pressure less than 30 cm H2O and arterial oxygen of 55-80 mmHg or oxygen saturation of 88-95%)

(increase FiO2/ PEEP to improve oxygenation and prevent overdistension by opening collapsed alveoli)

(lower FiO2/ PEEP once overshoot oxygenation goal while continuing mechanical ventilation)

How well did you know this?
1
Not at all
2
3
4
5
Perfectly
376
Q

What is the best way to form an initial therapeutic alliance with a psychotic patient?

A

permit pt to maintain privacy and interpersonal distance

How well did you know this?
1
Not at all
2
3
4
5
Perfectly
377
Q

What is the next best step in management of a pt with history sytemic lupus erythematous who develops renal involvement?

A

renal biopsy (to determine type of renal involvement to determine treatment)

How well did you know this?
1
Not at all
2
3
4
5
Perfectly
378
Q

What substances can be measured and used to monitor the activity level of lupus nephritis? (2)

A
  1. serum complement

2. anti- ds-DNA level

How well did you know this?
1
Not at all
2
3
4
5
Perfectly
379
Q

What substance can be measured and used to monitor the current level of immunosupression in an HIV pt?

A

CD4 count
(the immunologic damage that has already occured)

(viral load determines disease activity and is the damage that is about to occur)

How well did you know this?
1
Not at all
2
3
4
5
Perfectly
380
Q

What is the initial treatment for hyperkalemia with severe EKG changes (prolonged PR interval, prolonged QRS, disappearance of P wave; all more severe than peaked T waves)?

A

IV calcium gluconate (stabilizes myocardium membrane potential)

How well did you know this?
1
Not at all
2
3
4
5
Perfectly
381
Q

What is the next best step for a depressed pt with a partial response to monotherapy (after trial of 2 antidepressants)?

A

aument with second agent (second generation anti-psychotic or an antidepressant from different class)

How well did you know this?
1
Not at all
2
3
4
5
Perfectly
382
Q

A pt who develops pituitary enlargement, bitemporal hemianopsia and hyperigmentation following bilateral adrenalectomy for Cushing’s disease most likely suffers from ..

A

Nelson’s syndrome

(dx: brain MRI- pituitary microadenoma with suprasellar extension and ACTH level- high)
(tx: surgery and/or local radiation)

How well did you know this?
1
Not at all
2
3
4
5
Perfectly
383
Q

What is the best initial test for a women under 30 years old with a breast mass on clinical exam>

A

Ultrasound

(if solid mass with atypical features, core needle biopsy)
(if cystic mass, followup routine clinical exams or FNA if desired)

How well did you know this?
1
Not at all
2
3
4
5
Perfectly
384
Q

A pt presenting with severe anion gap metabolic acidosis, rapid and deep breathing (kussmaul’s respiration), nausea, vomiting, slurred speech and ataxia most likely suffers from …

A

Ethylene gylcol (anti-freeze) intoxication

How well did you know this?
1
Not at all
2
3
4
5
Perfectly
385
Q

What is the best treatmeent for ethylene gylcol (anti-freeze) intoxication?

A

fomepizole

How well did you know this?
1
Not at all
2
3
4
5
Perfectly
386
Q

What is the next best step in management of worsening thrombotic disease while on sub-therapeutic level of warfarin (INR less than 2-3)?

A

admit and start IV heparin (until can get warfarin at therapeutic level)

How well did you know this?
1
Not at all
2
3
4
5
Perfectly
387
Q

What is the best initial treatment for a pt with visible, palpable, tortuous superificial veins on the legs with leg cramping, heaviness, fatigue and leg swelling?

A

leg elevation, weight reduction and compression stockings

varicose veins

How well did you know this?
1
Not at all
2
3
4
5
Perfectly
388
Q

What other cardiac abnormality is highly associated with a bicuspid aortic valve?

A

aortic root and ascending aortic dilation

screen for with ECHO

How well did you know this?
1
Not at all
2
3
4
5
Perfectly
389
Q

A pt presenting with eye pain/ discomfort, decreased visual acuity and visualization of a floating, white layer in the anterior chamber (hypopyon), and conjunctival and lid edema and erythema most likely suffers from …

A

Endophthalmitis

ophthalmic emergency- immediate ophthalmology referral

How well did you know this?
1
Not at all
2
3
4
5
Perfectly
390
Q

An infant presenting with staccato cough (inspiration between every single cough), history of concurrent conjunctivitis, and auscultatory and radiologic findings out of proportion to healthy appearance of child (hyperinflation, peribronchial thickening, bilateral symmetrical infiltrates on CXR), elevated eosinophil count in the setting of no fever most likely suffers from …

A

Chlamydia pneumonia

(due to vaginal contact during delivery)

(tx: oral erythromycin for 14 days)

How well did you know this?
1
Not at all
2
3
4
5
Perfectly
391
Q

A pt presenting with fever, sudden onset sore throat with tonsillar erythema and exudates, tender anterior cervical nodes and palatal petechiae most likely suffers from ..

A

Streptococcal tonsillopharyngitis (strep throat)

dx: rapid strep antigen test, throat culture
(tx: penicillin or amoxicillin)

How well did you know this?
1
Not at all
2
3
4
5
Perfectly
392
Q

A pt presenting with sudden onset shortness of breath, pallor, diaphoresis, pulmonary edema, hypotension, hyperdynamic precordium/ apical impulse, and a soft low pitched decrescendo systolic murmur head best at the lower left sternal border most likely suffers from …

A

Acute Mitral Regurgitation

usualy due to ruptured mitral chordae tendineae
(can be due to papillary muscle rupture if 2-7 days after MI)

How well did you know this?
1
Not at all
2
3
4
5
Perfectly
393
Q

A pt with joint hypermobility, recurrent joint dislocations, velvety hyperextensible skin with easy bruisabilty, anddelayed healing of atrophic scars most likely suffers from ..

A

Ehler’s Danlos Syndrome

How well did you know this?
1
Not at all
2
3
4
5
Perfectly
394
Q

What cardiac abnormality is associated with Ehler’s Danlos syndrome?

A

mitral valve prolapse

that can lead to acute mitral regurgitation from rupture of chordae tendineae

How well did you know this?
1
Not at all
2
3
4
5
Perfectly
395
Q

An infant presenting with tomatoe red papulovesicular lesions in the diaper area including the crural folds with satellite papules (usually after treatment with an antibiotic) most likely suffers from …

A

Candidal diaper rash

tx: antimycotic cream - clotrimazole or nystatin

How well did you know this?
1
Not at all
2
3
4
5
Perfectly
396
Q

A pt presenting with excoriated, lichenified plaques that are itchy and located near the belt-line most likely suffers from …

A

Allergic Contact Dermatitis

probably secondary to nickel allergy from belt

How well did you know this?
1
Not at all
2
3
4
5
Perfectly
397
Q

What is the next best step in a pt presenting with difficulty initiating swallowing accompanied by coughing, drooling and/or aspiration in the dysphagia was initially for solids but progresses to include liquids?

A

Nasopharyngeal Laryngoscopy or Barium Esophagram

assess for mechanical obstruction leading to dysphagi

How well did you know this?
1
Not at all
2
3
4
5
Perfectly
398
Q

What is the next best step in a pt presenting with dysphagia initially involving both solids and liquids accompanied by coughing, drooling, and/ or aspiration?

A

Videofluoroscopic barium swallow

assess for neuromuscular disorder

How well did you know this?
1
Not at all
2
3
4
5
Perfectly
399
Q

What is the most likely histopathologic type of tumor to be found in the upper esophagus?

A

Squamous Cell Carcinoma

associated with chronic smoking and alcohol use

How well did you know this?
1
Not at all
2
3
4
5
Perfectly
400
Q

What is the most likely histopathologic type of tumor to be found in the lower esophagus?

A

Adenocarcinoma

associated with chronic GERD and Barrett’s esophagus

How well did you know this?
1
Not at all
2
3
4
5
Perfectly
401
Q

What are the common causes of hepatic encephalopathy? (8)

A
  1. GI bleeding
  2. hypokalemia
  3. hypovolemia
  4. hypoxia
  5. sedatives and tranquilzers
  6. hypoglycemia
  7. metabolic alkalosis
  8. infection (spontaneous bacterial peritonitis)
How well did you know this?
1
Not at all
2
3
4
5
Perfectly
402
Q

What are the three conditions in which potassium must be replaced immediately if hypokalemia is present? (3)

A
  1. hepatic encephalopathy (associated with loop diuretic)
  2. ventilatory failure
  3. cardiac arrhythmia
How well did you know this?
1
Not at all
2
3
4
5
Perfectly
403
Q

An HIV pt who presents with sensory ataxia (falling b/c feel off balance), lancinating pain (shooting/ burning), urinary incontinence, normal pupillary constriction with accommodation but not with light, aflexia, and impaired sensations most likely suffers from ..

A

Late Neurosyphillis

dx: test for treponema pallidum infection

How well did you know this?
1
Not at all
2
3
4
5
Perfectly
404
Q

… is a measure of risk that describes the chance of events occurring in one study arm compared to another and can be calculated at multiple time intervals throughout a study period

A

Hazard Ratios

(HR less than 1 means decreased risk)
(HR greater than 1 means increased risk)
(HR equal to 0 means no change in risk)

(only statistically significant if 95% confidence interval does not include null value of 1)

How well did you know this?
1
Not at all
2
3
4
5
Perfectly
405
Q

What are the measures of central tendency and their definitions? (3)

A

mode: value that occurs most frequently
Mean: sum divided by the total number of values
median: value separating the top half from lower half of data (50th percentile)

How well did you know this?
1
Not at all
2
3
4
5
Perfectly
406
Q

What are the contraindications to using dabigatran (or other target specific oral anticoagulants like rivaroxaban, apixaban, edoxaban)? (2)

A
  1. valvular a-fib (presence of moderate to severe valvular regurgitation/stenosis, prosthetic vallves)
  2. end stage renal disease
How well did you know this?
1
Not at all
2
3
4
5
Perfectly
407
Q

What is the best treatment for asymptomatic pulmonary sarcoidosis (i.e. hilar adenopathy with erythema nodosum)?

A

no treatment (spontaneous remission)

How well did you know this?
1
Not at all
2
3
4
5
Perfectly
408
Q

What are the three main disorders that make up Schmidt’s syndrome (polyglandular autoimmune failure type II)?

A
  1. Addison’s disease (no cortisol)
  2. Type 1 diabetes
  3. autoimmune thyroid disease
How well did you know this?
1
Not at all
2
3
4
5
Perfectly
409
Q

A pt with an EKG showing fast rate, narrow and regular QRS complexes with absent P waves most likely suffers from ..

A

Supraventricular tachycardia

How well did you know this?
1
Not at all
2
3
4
5
Perfectly
410
Q

A pt with an EKG showing regular rate and rhythm, presence of p waves, PR interval less than 0.12 sec, a slurred initial part of activation of the QRS complex and a QRS durationof about 0.12 sec most likely suffers from ..

A

Wolff Parkinson White syndrome

delta wave- slurried initial part of QRS complex

How well did you know this?
1
Not at all
2
3
4
5
Perfectly
411
Q

A pt who develops hypoxemia and increased work of breathing 2-5 days after thoracoabdominal surgery with a CXR showing small lung volumes most likely suffers from …

A

Atelectasis

due to splinting, reduced cough from abdominal pain, retained secretions and diminished lung compliance

How well did you know this?
1
Not at all
2
3
4
5
Perfectly
412
Q

What is the colon cancer screening guideline for a pt who has a first degree relative diagnosed before age 60 with colorectal cancer or advanced adenomatous polyps?

A

colonscopy at age 40 years old or 10 years before age of onset in relative (whichever comes first); repeat every 3-5 years

How well did you know this?
1
Not at all
2
3
4
5
Perfectly
413
Q

What is the best initial diagnostic test for renovascular disease/ hypertension in a pt with renal insufficiency?

A

renal duplex doppler ultrasonography

avoid contrast induced nephropathy with CTA and nephrogenic systemic fibrosis with MRA

How well did you know this?
1
Not at all
2
3
4
5
Perfectly
414
Q

What is the time interval between giving a phosphodiesterase inhibitor (sildenafil) and an alpha blocker (doxazosin)?

A

4 hours

How well did you know this?
1
Not at all
2
3
4
5
Perfectly
415
Q

What is the next best step in management of a symptomatic pt with a EKG documented STEMI?

A

percutaneous coronary intervention (if within 12 hours of symptom onset and 90 minutes of initial contact)

(fibrinolysis (tpa) if cant undergo PCI)

How well did you know this?
1
Not at all
2
3
4
5
Perfectly
416
Q

A pt who develops hypotension, diaphoresis, jugular venous distension, and cold extremities with clear lungs shortly after being diagnosed with an inferior wall MI (leads II, III, avf) most likely suffers from …

A

Right Ventricular MI

How well did you know this?
1
Not at all
2
3
4
5
Perfectly
417
Q

What is the best treatment for right ventricular MI?

A

bolus of IV fluids (increased preload)

avoid nitrates, diuretics, opiods that lower preload

How well did you know this?
1
Not at all
2
3
4
5
Perfectly
418
Q

What does asymmetry in a funnel plot suggest?

A

publication bias (studies showing opposite effect are absent as they are less likely to be published)

How well did you know this?
1
Not at all
2
3
4
5
Perfectly
419
Q

What is the best initial treatment for symptomatic (CNS symptoms) or severe hyponatremia (Na below 115)?

A

3% NaCl (hypertonic saline)

goal correction rate of 1.5-2 mEq/L/hour for first 3-4 hours; no more than 12 mEq/L in first 24 hours

How well did you know this?
1
Not at all
2
3
4
5
Perfectly
420
Q

What is the first line treatment used for torsade de pointes as well as to prevent recurrence?

A

IV Magnesium sulfate

depsite normal serum Mg levels

How well did you know this?
1
Not at all
2
3
4
5
Perfectly
421
Q

What is the next best step in management for torsade de pointes if pt does not respond to IV magnesium sulfate?

A

temporary transvenous pacing

How well did you know this?
1
Not at all
2
3
4
5
Perfectly
422
Q

What is the next best step in management of a pt with a palpable renal mass?

A

ultrasound: if low risk of cancer

CT abdomen: if high risk (older pt, smoking, elevated Hct and Hb, night sweats, weight loss)

How well did you know this?
1
Not at all
2
3
4
5
Perfectly
423
Q

A pt presenting with long standing smoking history, flank pain, flank mass, increased hematocrit and hemoglobin levels and night sweats most likely suffers from ..

A

Renal Cell Carcinoma

How well did you know this?
1
Not at all
2
3
4
5
Perfectly
424
Q

What conditions are associated with the presence of multiple skin tags (pedunculated, skin colored papules mostly on areas of friction)?

A
  1. insulin resistance/ metabolic syndrome
  2. pregnancy
  3. crohns disease (if located in perianal region)
How well did you know this?
1
Not at all
2
3
4
5
Perfectly
425
Q

What medical condition is associated with dermatitis herpetiformis (pruritc, erythematous, grouped papuloesicular lesions that usually occur on elbows)?

A

Celiac disease

How well did you know this?
1
Not at all
2
3
4
5
Perfectly
426
Q

What medical condition is associated with pyoderma gangrenosum (purulent ulcer)?

A

Crohn’s disease

How well did you know this?
1
Not at all
2
3
4
5
Perfectly
427
Q

A pt presenting with scaly patches with slight erythema in their hair with associated hair loss and cervical lymphadenopathy most likely suffers from ..

A

Tinea Capitis

How well did you know this?
1
Not at all
2
3
4
5
Perfectly
428
Q

What is the first line treatment for tinea capitis?

A

oral griseofulvin (alternative is oral terbinafine)

How well did you know this?
1
Not at all
2
3
4
5
Perfectly
429
Q

What is the most appropriate next step in management of a pt who is found to have multifocal atrial tachycardia (tachycardia with narrow QRS, 3 or more different P wave morphologies, variable PR and RR intervals)?

A

obtain arterial blood gas

(to assess for hypoxemia as cause of MAT which if corrected can fix MAT)
(followed by chemistries for electrlyte imbalance)

How well did you know this?
1
Not at all
2
3
4
5
Perfectly
430
Q

What are common causes of multifocal atrial tachycardia? (6)

A
  1. hypoxia
  2. COPD
  3. hypokalemia
  4. hypomagnesemia
  5. heart disease
  6. meds (theophyilline, aminophylline, isoproterenol)
How well did you know this?
1
Not at all
2
3
4
5
Perfectly
431
Q

A child presenting with emotional lability, decreased school performance, distal hand movements, facial grimacing, feet jerking, decreased strength, delayed relaxation phase of patellar reflex, and positive pronator drift sign in the setting of a history of self-resolving prior infection most likely suffers from ..

A

Sydenham chorea

complication of untreated strep pharyngitis; acute rheumatic fever

How well did you know this?
1
Not at all
2
3
4
5
Perfectly
432
Q

What is the JONES criteria and what disease is it used to diagnosed?

A

acute rheumati fever if have 2 major or 1 major with 2 minor criteria

major: Joints (migratory arthritis), carditis, Nodules (subcutaneous), Erythema marginatum, Sydenham chorea
minor: fever, arthalgias, elevated ESR/ CRP, prolonged PR interval

How well did you know this?
1
Not at all
2
3
4
5
Perfectly
433
Q

What is the best initial step in management of a pt diagnosed with sydenham chorea secondary to untreated strep pharyngitis?

A

long acting IM penicillin (until adulthood to erradicate grap A strep)

How well did you know this?
1
Not at all
2
3
4
5
Perfectly
434
Q

A pt presenting with sudden onset vertigo, nystagmus, loss of pain and temperature in ipsilateral face and contralateral body, hoarseness, ipsilateral horner’s syndrome, decreased gag reflex and aspiration most likely suffers from..

A

Lateral Medullary Infarction

wallenberg syndrome
(loss of pain and temp make it lateral)
(cranial nerves 9 and 10 make it medullary)

How well did you know this?
1
Not at all
2
3
4
5
Perfectly
435
Q

What is the best initial step in management for a pt presenting with chronic diarrhea?

A

microscopic exam of stool

for leukocytes, ova, parasites, occult blood, fat staining, pH, osmotic gap

How well did you know this?
1
Not at all
2
3
4
5
Perfectly
436
Q

What are the findings on small intestinal biopsy that are suggestive of Celiac disease?

A

villus blunting, loss of normal villus architecture, increased lymphocytic infiltration of mucosa

How well did you know this?
1
Not at all
2
3
4
5
Perfectly
437
Q

What is the next best step in management of an HIV pt with CNS symptoms who is found to have positive syphillis test results?

A

Lumbar puncture

to assess whether pt has neurosyphilis

How well did you know this?
1
Not at all
2
3
4
5
Perfectly
438
Q

What is the treatment for late latent syphillis (more than 12 months of infection), syphilis of unknown duration or gummatous/ cardiovascular syphilis?

A

benzathine penicillin G IM each week for 3 weeks

How well did you know this?
1
Not at all
2
3
4
5
Perfectly
439
Q

What is the treatment for primary, secondary or early latent (less than 12 months of infection) syphilis?

A

single dose of benzathine penicillin G IM

How well did you know this?
1
Not at all
2
3
4
5
Perfectly
440
Q

What is the treatment for neurosyphilis?

A

aqueous penicillin G IV every 4 hours for 10-14 days

How well did you know this?
1
Not at all
2
3
4
5
Perfectly
441
Q

What is the treatment for congenital syphilis?

A

aqueous penicillin G IV every 8-12 hours for 10 days

How well did you know this?
1
Not at all
2
3
4
5
Perfectly
442
Q

A pt being treated for syphilis who develops fever, malaise, chills, headache, and myalgias within 24 hours of initiation the treatment most likely suffers from …

A

Jarisch-Herxheimer reaction

no effective prevention

How well did you know this?
1
Not at all
2
3
4
5
Perfectly
443
Q

… is the ability of a test to correctly identify patients with the disease and therefore is best to rule out a diagnosis

A

Sensitivity

SnOut= sensitivity rules out disease

How well did you know this?
1
Not at all
2
3
4
5
Perfectly
444
Q

What is the formula for positive likelihood ratio?

A

positive LR = sensitivity / (1- specificity)

independent of prevalence

How well did you know this?
1
Not at all
2
3
4
5
Perfectly
445
Q

What is the formula for negative likelihood ratio?

A

negative LR = (1- sensitivity)/ specificity

independent of prevalence

How well did you know this?
1
Not at all
2
3
4
5
Perfectly
446
Q

… occurs when a study uses gold standard testing selectively in order to confirm a positive/ negative result of preliminary testing; therefore performing the test in a random sample of participants reduces it occurence

A

Verification bias

How well did you know this?
1
Not at all
2
3
4
5
Perfectly
447
Q

What is the next best step in management of a pt presenting with substernal chest pain and dyspnea relieved with sublingual nitroglycerine?

A

admit, EKG and serial cardiac enzymes

takes up to 6 hours for cardiac enzymes to be positive

How well did you know this?
1
Not at all
2
3
4
5
Perfectly
448
Q

What is the best initial intervention of an unknown dry/ powdered chemical?

A

brush off remaining chemical

followed by irrigation with copious amounts of low pressure water for 15-30 minutes

How well did you know this?
1
Not at all
2
3
4
5
Perfectly
449
Q

What is the next best step in management of an acute exacerbation that is nor responding to nebulized albuterol therapy (shortness of breath and wheezing persist)?

A

IV corticosteroids (systemic steroids)

same in pregnant pt; PaCO2 greater than 35 suggests respiratory compromise in pregnant pt

How well did you know this?
1
Not at all
2
3
4
5
Perfectly
450
Q

what is the next best step in management of a depressed pt with passive suicidal ideation (wish to go to sleep and not awake, better off dead) without a plan?

A

outpatient treatment with antidepressant and close follow-up

How well did you know this?
1
Not at all
2
3
4
5
Perfectly
451
Q

A pt presenting with progressive fatigue, shortness of breath, fever, systolic murmur that increases with inspiration heard at LLSB, CXR showing circumscribed round pulmonary infiltrates and tricuspid vegetation on ECHO most likely suffers from ..

A

IV drug use induced infective endocarditis

right heart endocarditis: tricuspid valve abnormalities and septic pulmonary emboli

How well did you know this?
1
Not at all
2
3
4
5
Perfectly
452
Q

What is the formula for standardized mortality ratio?

A

SMR = observed number of deaths/ expected number of deaths

adjusted measure of overall mortality used in occupational epidemiology

How well did you know this?
1
Not at all
2
3
4
5
Perfectly
453
Q

What is the latent period?

A

time elapsed from initial exposure to clinically apparent disease

How well did you know this?
1
Not at all
2
3
4
5
Perfectly
454
Q

…. occurs when a study fails to reject a null hypothesis that is false; in other wrose the failure to detect a difference between groups when a difference exists and it is related to the … of a study

A

type II error; power (sample size)

How well did you know this?
1
Not at all
2
3
4
5
Perfectly
455
Q

A pt with history of anatomical change of GI tract/ motility disorder (scleroderma, diabetes)/ immunocompromised/ acid suppresion presenting with abdominal pain, diarrhea, bloating, excess flatulence, malabsorption, weight loss, anemia and nutritional deficiencies most likely suffers from ..

A

Small Intestine Bacterial Overgrowth

(lack of protective mechanisms such as bacterial degradation by proteolytic digestive enzymes, bacteria trapping in intestinal mucus layer, intact ileocecal valve preventing retrograde bacterial movement from colon, gastric acidity, peristalsis)

How well did you know this?
1
Not at all
2
3
4
5
Perfectly
456
Q

What is the most accurate test for diagnosing small intestine bacterial overgrowth?

A

endoscopy with jejunal aspirate showing more than 10,000 organisms/ ml

(less accurate is hydrogen breath test)

How well did you know this?
1
Not at all
2
3
4
5
Perfectly
457
Q

What is the next best step in management of a pt with rheumatoid arthritis that is resistant to methotrexate and steroid after 6 months of treatment?

A

switch to or add anti-cytokine (infliximab or etanercept)

screen for TB first

How well did you know this?
1
Not at all
2
3
4
5
Perfectly
458
Q

What is the next best step in management of a pt with rheumatoid arthritis who presents with new onset monoarticular arthritis with associated fever?

A

Joint Aspiration

to assess for septic arthritis

How well did you know this?
1
Not at all
2
3
4
5
Perfectly
459
Q

A pt presenting with recurrent and unexpected epsiodes of palpitations, sweating, shaking, and shortness of breath and have concern about having additional epsiodes/ about implications or consequences of episodes/ significant change in behavior because of episodes most likely suffers from ..

A

Panic Disorder

(associated with agoraphobia- fear of being in public should a panic attack occur and subsequently avoid public situations)

How well did you know this?
1
Not at all
2
3
4
5
Perfectly
460
Q

An infant who lives near farm/ construction site in California/ Pennsylvania/ Utah presenting with constipation and cranial nerve palsies (ptosis, pupillary paralysis, weak suck) followed by progressive hypotonia and loss of deep tendon reflexes most likely suffers from ..

A

Infant Botulism

(due to ingestion of C. botulinum spore in environmental dust; or ingestion of honey)
(good prognosis; 1-3 month hospitalization with full recovery)

How well did you know this?
1
Not at all
2
3
4
5
Perfectly
461
Q

what is the best treatment for infant botulism?

A

IV human derived botulism immunoglobulin

along with ICU, NG tube feedings, laxatives, PT/OT

How well did you know this?
1
Not at all
2
3
4
5
Perfectly
462
Q

What is the best treatment for foodborne botulism (due to ingestion of preformed C botulinum toxin)?

A

equine-derived botulism toxin

How well did you know this?
1
Not at all
2
3
4
5
Perfectly
463
Q

What is the best initial step in management of a man presenting with testicular swelling 9especially if painless nad negative transillumination)?

A

scrotal ultrasound

assess whether intra or extra tesicular lesions and whether it is solid or cystic mass

How well did you know this?
1
Not at all
2
3
4
5
Perfectly
464
Q

What is the next best step in management of a pt with lobular carcinoma in situ (nonmalignant lesion) on needle biopsy?

A

excisional biopsy

b/c has significant associated with future development of invasive breast cancer

How well did you know this?
1
Not at all
2
3
4
5
Perfectly
465
Q

What is the best treatment for symptomatic aortic stenosis (dysnea ofheart failure, anginal pain, syncope)?

A

aortic valve replacement

marked reduction in mortality and symptoms

How well did you know this?
1
Not at all
2
3
4
5
Perfectly
466
Q

What does non-overlapping confidence intervals suggest when comparing one group to another in a study and the P-value is significant?

A

significant differences between groups

How well did you know this?
1
Not at all
2
3
4
5
Perfectly
467
Q

A pt presenting with sandy sensation in eyes, dry eyes, prominent bulbar blood vessels and stringy discharge from eyes, oral candidiasis, dental caries, chronic esophagitis and dry mouth most likely suffers from …

A

Sjogren syndrome

sicca syndrome is keratoconjunctivitis- dry eyes and xerostomia- dry mouth

How well did you know this?
1
Not at all
2
3
4
5
Perfectly
468
Q

A pt presenting with wrist swelling, decreased grip strength, pain and tenderness on the radial aspect of the wrist after falling on outstretched hand with dosifleced wrist most likely suffers from …

A

Scaphoid fracture

pain located in the anatomical snuffbox

How well did you know this?
1
Not at all
2
3
4
5
Perfectly
469
Q

What is the next best step in management of a pt with suspected scaphoid fracture in which initial X-rays are negative? (3)

A
  1. MRI/ CT of wrist
  2. repeat X-ray in 7-10 days
  3. radioscintigraphy bone scane in 3-5 days
How well did you know this?
1
Not at all
2
3
4
5
Perfectly
470
Q

What is the treatment for nondisplaced scaphoid fractures?

A

short arm thumb spica cast

inadequate treatment results in nonunion and avascular necrosis

How well did you know this?
1
Not at all
2
3
4
5
Perfectly
471
Q

An athlete presents with excruciating pain and swelling of the knee, difficulty bearing weight, tenderness at the anterior aspect of the knee, inability to actively extend the leg, and inability to maintain passive extension of knee against gravity after a traumatic injury most likely suffers from ..

A

Patellar Tendon Rupture

tx: surgery

How well did you know this?
1
Not at all
2
3
4
5
Perfectly
472
Q

What is the best initial treatment for bipolar maniaduring pregnancy?

A

haloperidol

How well did you know this?
1
Not at all
2
3
4
5
Perfectly
473
Q

A bone marrow transplant pt presenting with fever, cough, chest pain, localized headache, nasal bleeding and has patchy infiltrate on CXR most likely suffers from ..

A

Invasive Aspergilliosis

How well did you know this?
1
Not at all
2
3
4
5
Perfectly
474
Q

A pt presenting with eyelid erythema, swelling and tenderness along with proptosis, ophthalmoplegia and pain with eye movements most likely suffers from ..

A

Orbital Cellulitis

tx: inpatient Iv antibiotics

How well did you know this?
1
Not at all
2
3
4
5
Perfectly
475
Q

A pt presenting with eyelid erythem, swelling and tenderness along with fever, and leukocytosis most likely suffers from …

A

Preseptal Cellulitis

tx: oral antibiotics

How well did you know this?
1
Not at all
2
3
4
5
Perfectly
476
Q

What are complications of orbital cellulitis?

A
  1. orbital abscess
  2. intracranial infection
  3. cavernous sinus venous thrombosis
How well did you know this?
1
Not at all
2
3
4
5
Perfectly
477
Q

What is the next best step in the management of an elevated capillary lead level in a child?

A

obtain venous lead level

(mild: 5-44; no meds, repeat level in 1 month)
(moderate: 45-69, treat with DMSA)
(severe: more than 70; treat with dimercaprol with EDTA)

How well did you know this?
1
Not at all
2
3
4
5
Perfectly
478
Q

What is the next best step in management for a pt on amiodarone who develops abnormal thryoid biomarkers in which there is nomal to high TSH, low T3 and high T4?

A

repeat thryoid function testing in few weeks

effect of decreased peripheral conversion of T4 to T3 from amiodarone usually improves after 3-4 months

How well did you know this?
1
Not at all
2
3
4
5
Perfectly
479
Q

What is the next best step for a pt with epigastric/ abdominal discomfort and nausea worse after eating that does not respond to trial with PPI and has no alarming symptoms (age greater than 55, weight loss, anemia, gross bleeding)?

A

H. pylori testing (stool antigen testing or breath test)

How well did you know this?
1
Not at all
2
3
4
5
Perfectly
480
Q

What is the best initial treatment for oral candidiasis (cottony taste in mouth, white plaques on buccal mucosa that can be scraped off leaving hyperemic spots)?

A

nystatin suspension or clotrimazole troches

(topical antifungal)
(if associated with use of inhaler in asthma, rinse mouth out after using inhaler and proper technique)

How well did you know this?
1
Not at all
2
3
4
5
Perfectly
481
Q

What is the best initial treatment for moderate lead toxicity (lead 45-69)?

A

oral meso-2-3-dimercaptosuccinic acid (DMSA, succimer)

How well did you know this?
1
Not at all
2
3
4
5
Perfectly
482
Q

What is the best initial treatment for severe lead toxicity (lead 70 or greater)?

A

dimercaprol with EDTA (calcium disodium edetate)

How well did you know this?
1
Not at all
2
3
4
5
Perfectly
483
Q

What is the best initial diagnostic test for suspected acute diverticulitis?

A

abdominal CT (showing colonic wall thickening and stranding of mesenteric fat, diverticulae, as well as complications such as abscess, perforation, fistula)

How well did you know this?
1
Not at all
2
3
4
5
Perfectly
484
Q

What is the next best step in management of individuals exposed to pt with active TB whose PPD test result is negative?

A

repeat PPD in 3 months

if PPD positive, monotherapy with INH for 9 months

How well did you know this?
1
Not at all
2
3
4
5
Perfectly
485
Q

What is the best next step in management of acute aortic dissection after pain management?

A

IV beta blocker (esmolol- short acting)

to slow HR, lower sBP to 100-120 mm/Hg, reduced LV contractility

How well did you know this?
1
Not at all
2
3
4
5
Perfectly
486
Q

What is the next best step in management of acute aortic dissection if systolic blood pressure remains above 100-120 mmHg after IV beta blocker?

A

nitroprusside

How well did you know this?
1
Not at all
2
3
4
5
Perfectly
487
Q

A pt presenting with episode of syncope while standing that was proceeded by nausea, lightheadness, pallor and diarphoresis and recovered while supine without jerky movements, tongue biting, and incontinence most likely suffers from …

A

Vasovagal Syncope (Neurocardiogenic)

How well did you know this?
1
Not at all
2
3
4
5
Perfectly
488
Q

An obese boy presenting with hip pain that may spread to the thigh and knee without recent history of trauma, and the hip is externally rotated most likely suffers from …

A

Slipped Capital Femoral Epiphysis (SCFE)

acute less than 3 weeks; chronic more than 3 weeks
(stable can bare weight; unstable cant bare weight)

How well did you know this?
1
Not at all
2
3
4
5
Perfectly
489
Q

What is the treatment for slipped capital femoral epiphysis (SCFE)?

A

immediate internal fixation with pins

How well did you know this?
1
Not at all
2
3
4
5
Perfectly
490
Q

What antibiotics can be used as first line treatment for UTI in a pregnant patient? (4)

A
  1. nitrofurantoin
  2. amoxicillin
  3. cephalexin
  4. amoxicillin- clavulanate
How well did you know this?
1
Not at all
2
3
4
5
Perfectly
491
Q

What is the best initial treatment of pyelonephritis (fever, chills, dysuria, nausea, vomiting, CVA tenderness) in a pregnant pt?

A

hospitalization and IV antibiotics until afebrile for 24-48 hours and symptom improvement, then outpatient antibiotics for total of 10-14 days

(ceftriaxone, aztreonam, ampicillin with gentamicin)

How well did you know this?
1
Not at all
2
3
4
5
Perfectly
492
Q

A pt with a lumbar puncture showing elevated RBC, elevated WBC at a ratio of 1 WBC to 750-1000 RBCs, elevated protein and glucose level in the absence of xanthochromia most likely suffers from …

A

Traumatic Lumbar Tap

How well did you know this?
1
Not at all
2
3
4
5
Perfectly
493
Q

An elderly pt presenting with signs of osteoporesis such as multiple fractures that is not responding to bisphosphonate therapy along with constitutional symptoms, weight loss and pallor most likely suffers from …

A

Multiple Myeloma

How well did you know this?
1
Not at all
2
3
4
5
Perfectly
494
Q

What is the diagnostic test for multiple myeloma?

A

serum and urine protein electrophoresis

How well did you know this?
1
Not at all
2
3
4
5
Perfectly
495
Q

An IV drug abuser who presents with malaise, nausea, jaundice and right upper quadrant pain along with have elevated transaminases (10-20 times upper limit) most likley suffers from ..

A

Acute Hepatitis

(if anti HAV antibodies, HBsAg, anti HBsAg antibodies and anti HCV antibodies are negative, likely hep C infection prior to formation of antibodies)

How well did you know this?
1
Not at all
2
3
4
5
Perfectly
496
Q

What are the absolute contraindications to combined hormonal contraceptives? (9)

A
  1. migraine with aura
  2. smoking more than 15 cigs/ day and older than 35
  3. stage 2 HTN (greater than 160/100)
  4. hx of venous thromboembolic disease
  5. hx of stroke/ ischemic heart disease
  6. breast cancer
  7. cirrhosis and liver cancer
  8. major surgery with prolonged immoblization
  9. less than 3 weeks postpartum
How well did you know this?
1
Not at all
2
3
4
5
Perfectly
497
Q

What type of cancer is at increased risk in a pt with Klinefelter’s syndrome (hypogonadism, low testosterone, gynecomastia, karyotype of 47 XXY)?

A

Breast Cancer

50x more likely compared to pt with normal karyotype

How well did you know this?
1
Not at all
2
3
4
5
Perfectly
498
Q

What is the formula for positive predictive value (PPV)?

A

PPV = true positive/ (true positive + false positive)

PPV = a/ (a+b)

(probability that an individual truly has disease given a positive test result; dependent on prevalence of disease)

(increasing specificity results in increasing positive predictive value)

How well did you know this?
1
Not at all
2
3
4
5
Perfectly
499
Q

What is the formula for negative predictive value (NPV)?

A

NPV = true negative/ (true negative + false negative)

PPV = d / (d +c)

(probability that an individual truly does not have disease given a negative test result; dependent on prevalence of disease)

How well did you know this?
1
Not at all
2
3
4
5
Perfectly
500
Q

A pt who develops swinging fever, leukocytosis, cough and shoulder tip pain 14-21 days after abdominal surgery most likely suffers from …

A

Subphrenic Abscess

dx: abdominal ultrasound

How well did you know this?
1
Not at all
2
3
4
5
Perfectly
501
Q

What is the best diagnostic test for monitoring cardiotoxicity from anthrocycline chemotherapy agents (doxorubicin, daunorubicin)?

A

Radionuclide Ventriculography (MUGA) to assess LV ejection fraction

(chemotherapy contraindicated if ejection fraction less than 30%; discontinue therapy if ejection fraction decreases by 10% or more)

How well did you know this?
1
Not at all
2
3
4
5
Perfectly
502
Q

What is the best initial test used to determine the cause of a couple’s infertility issues (unable to conceive after 1 year of unprotected sex)?

A

semen analysis

easy and detects male cause which is 20-30% of time

How well did you know this?
1
Not at all
2
3
4
5
Perfectly
503
Q

What is the first line treatment of hypertension in a pt with bipolar depression who is being treated with lithium?

A

Calcium channel blockers

avoid ACE inhibitors, ARBs, diuretics as affect renal exretion of lithium

How well did you know this?
1
Not at all
2
3
4
5
Perfectly
504
Q

What is the best initial step in management of an adult pt with mental retardation or dementia?

A

ask about guardianship status

How well did you know this?
1
Not at all
2
3
4
5
Perfectly
505
Q

What is the serum-ascites albumin gradient (SAAG) formula and what is it used for?

A

SAAG = serum albumin - ascitic fluid albumin

used to identify presence or absence of portal HTN
(SAAG greater or equal to 1.1 is portal HTN)
(SAAG less than 1.1 is non-portal HTN - peritoneal TB or carcinomatosis, nephrotic syndrome, pancreatitis, serositis)

How well did you know this?
1
Not at all
2
3
4
5
Perfectly
506
Q

A pt who develops anorexia, weight loss, insomnia, palpitations, tachycardia, elevated BP, irritability and agitation after starting treatment for ADHD most likely suffers from …

A

Methylphenidate/ stimulant toxicity

long term: decreased height and weight

How well did you know this?
1
Not at all
2
3
4
5
Perfectly
507
Q

What is the best initial treatment for keloids (benign fibrous growths in scar tissue secondary to overproduction of extracellular matrix and dermal fibroblasts)?

A

intralesional glucocorticoids

high recurrence rate; use excisional removal if fail steroids

How well did you know this?
1
Not at all
2
3
4
5
Perfectly
508
Q

When should a C-section be performed for an HIV positive pregnant pt?

A

if vial load greater than 1000 copies/ ml

(if pregnant pt on HAART therapy prior to conceiving maintain same HAART therapy during pregnancy to minimize drug resistance and loss or viral suppression)

How well did you know this?
1
Not at all
2
3
4
5
Perfectly
509
Q

What is the best management of a newborn of an HIV positive mother?

A

Zidovudine for 6 or more weeks with serial HIV PCR testing

avoid breastfeeding; use formula

How well did you know this?
1
Not at all
2
3
4
5
Perfectly
510
Q

What are maternal contraindications to breastfeeding? (6)

A
  1. active untreated TB (can after 2 weeks of therapy)
  2. maternal HIV infection
  3. herptic breast lesions
  4. varicella infection less than 5 days before or 3 days after delivery
  5. chemotherapy/ radiation therapy
  6. alcohol or drug use
How well did you know this?
1
Not at all
2
3
4
5
Perfectly
511
Q

What is the next best step in management of a pt with a pituitary lesion that was incidentally found and who lacks clinical or laboratory evidence of pituitary abnormality/ dsfunction?

A

repeat MRI of pituitary in 6-12 months

incidentaloma

How well did you know this?
1
Not at all
2
3
4
5
Perfectly
512
Q

A pt presenting with dyspnea, tachypnea, hypoxemia and hemoptysis after a blunt chest traumain which CXR shows homogenous opacification of the lung fields without a specific anatomic segment involved most likely suffers from ….

A

Pulmonary Contusion

How well did you know this?
1
Not at all
2
3
4
5
Perfectly
513
Q

What is the next best step in management of a pt with pulmonary contusion?

A

admit to hospital and monitor for 24-48 hours (for signs of clinical deterioration)

How well did you know this?
1
Not at all
2
3
4
5
Perfectly
514
Q

What is the effect of magnitude of the expected effect size (size of expected difference between the groups) on the power of a study?

A

the smaller the magnitude of expected effect size, the higher the power of the study needs to be to see effect
(therefore more subjects are required to see the effect)

How well did you know this?
1
Not at all
2
3
4
5
Perfectly
515
Q

An HIV pt presenting with fat tissue deposition on the back of the neck and abdomen along with thin extremities and face most likely suffers from …

A

HIV lipodystrophy due to insulin resistance

tx: metformin and “zones”

How well did you know this?
1
Not at all
2
3
4
5
Perfectly
516
Q

What is the next best step in management of an HIV pt with dyslipidemia particularly marked triglyceridemia (TGs greater than 500)?

A

Fibrates (gemfibrozil)

How well did you know this?
1
Not at all
2
3
4
5
Perfectly
517
Q

What is the next best step in management of an HIV pt with dyslipidemia with moderate elevation if triglycerides (TGs less than 500)?

A

statin

How well did you know this?
1
Not at all
2
3
4
5
Perfectly
518
Q

What are the common associated disorders of having Turner’s syndrome (XO karyotype)? (4)

A
  1. coarctation of aorta
  2. bicuspid aortic valve
  3. horseshoe kidney
  4. streaked ovaries/ amenorrhea/ infertility
How well did you know this?
1
Not at all
2
3
4
5
Perfectly
519
Q

What is the best next step in preventing further retinal damage or peripheral neuropathy damage in a diabetic patient?

A

strict glycemic control (goal of HbA1c below 7%)

improves microvascular complications of diabetes such as retinopathy, peripheral neuropathy and nephropathy

How well did you know this?
1
Not at all
2
3
4
5
Perfectly
520
Q

What is the best next step in management of a pt who develops lithium-induced hypothyroidism (elevated TSH, fatigue, constipation) while being treated for bipolar disorder?

A

continue lithium and add levothyroxine

especially if hx of severe mood episodes that respond to lithium

How well did you know this?
1
Not at all
2
3
4
5
Perfectly
521
Q

What is the best initial diagnostic test for a pt who presents with hemoptysis?

A

Chest X-ray

How well did you know this?
1
Not at all
2
3
4
5
Perfectly
522
Q

What is the best next step in management of a pregnant pt at high risk of cervical insufficiency (in ability of cervix to hold pregnancy in the absence of labor resulting in increased risk of preterm birth)?

A

serial ultrasound cervical lengths and evaluations for cerclage placement during second trimester (16-24 weeks)

How well did you know this?
1
Not at all
2
3
4
5
Perfectly
523
Q

What are risk factors for cervical insufficiency? (5)

A
  1. collagen abnormalities (Ehlers- Danlos)
  2. uterine abnormalities (septate/ bicornuate uterus)
  3. prior mechanical cervical dilation (D&C, pregnancy termination)
  4. prior OB cervical laceration
  5. prior cone or LEEP procedure for CIN
How well did you know this?
1
Not at all
2
3
4
5
Perfectly
524
Q

What is the most common extraneural complication of myelomeningoceles (bulging sac covered with membranes in lower back)?

A

Genitourinary system involvement (bladder dysfunction, urinary tract and renal dysfunction)

How well did you know this?
1
Not at all
2
3
4
5
Perfectly
525
Q

What is considered hypoglycemia?

A

blood sugar level less than 60 mg/dL

normal is 70-100 mg/dL

How well did you know this?
1
Not at all
2
3
4
5
Perfectly
526
Q

What is the target pre- meal blood sugar level in diabetic patients?

A

80-120 mg/dL

if blood sugar level outside this range, adjust insulin

How well did you know this?
1
Not at all
2
3
4
5
Perfectly
527
Q

What is Whipple’s triad?

A
  1. low blood glucose level
  2. symptoms of hypoglycemia (headache, palpitations, sweating, irritability)
  3. symptomatic relief with glucose administration/ eating

(suggests true hypoglycemia)

How well did you know this?
1
Not at all
2
3
4
5
Perfectly
528
Q

What is the next best step in management of exercise induced hypoglycemic episodes in diabetic patient? (2)

A
  1. eat source of glucose before exercising

2. decrease level of basal insulin (in morning if exercise in afternoon)

How well did you know this?
1
Not at all
2
3
4
5
Perfectly
529
Q

What is a common gram negative anaerobe found in human bites?

A

Eikenella corrodens

How well did you know this?
1
Not at all
2
3
4
5
Perfectly
530
Q

What is the best antibiotic treatment for human bite wounds (polymicrobial with mixture of anaerobic and aerobic organisms)?

A

Amoxicillin- clavulanate

How well did you know this?
1
Not at all
2
3
4
5
Perfectly
531
Q

A pt presenting with episodes of well-circumscribed and raised plaques with central pallor and intensely itchy in which lesions appear and enlarge over minutes to hours before disappearing within 24 hours and episodes occur for more than 6 weeks most likely suffers from …

A

Chronic Urticaria (Hives)

dx: clinical
(prognosis: spontaneous resolution within 2-5 years)

How well did you know this?
1
Not at all
2
3
4
5
Perfectly
532
Q

What is the best initial treatment for chronic urticaria?

A

second generation antihistamine (loratadine, certirizine) and avoid aggravating factors

(if fail: increase dose or add additional first generation H1 blocker like hydroxyzine/ leukotriene receptor antagonist like montelukast/ H2 blocker like ranitidine, or short course of steroids)

How well did you know this?
1
Not at all
2
3
4
5
Perfectly
533
Q

A pt presenting with knee pain, swelling and stiffness after hitting the knee on the dashboard during a motor vehicle accident most likely suffers from …

A

Posterior cruciate ligament injury

(posteriorly directed force on anterior aspect of proximal tibia with knee flexed such as with athlete falling on flexed knee with foot plantar flexed)

How well did you know this?
1
Not at all
2
3
4
5
Perfectly
534
Q

A pt presenting with multiple psychotic episodes with concurrent major depressive or maniac symptoms along with at least 2 weeks of delusions or hallucinations in absence of mood symptoms most likely suffers from …

A

Schizoaffective Disorder

How well did you know this?
1
Not at all
2
3
4
5
Perfectly
535
Q

What is the most common cause of hereditary/ inhertied thrombophilia?

A

Factor V Leiden

increased risk of venous thromboembolism

How well did you know this?
1
Not at all
2
3
4
5
Perfectly
536
Q

A pt who develops nightmares and flashbacks of a traumatic event, negative mood, avoids thoughts/ feelings/ external reminders of the event, sleep disturbances, irritability/ outbursts, hypervigilence, impaired concetration and exaggerated startle reflex 3 days to 1 month after a life- threatening trauma most likely suffers from …

A

Acute Stress Disorder

tx: trauma focused cognitive behavioral therapy and short term benzos for agitation/ insomnia

How well did you know this?
1
Not at all
2
3
4
5
Perfectly
537
Q

What is the best next step in management of an infant 6 months or older with hypoplastic, hypopigmented, poorly rugated and empty scrotum with bilateral inguinal fullness without signs of torsion (tenderness, swelling, discoloration)?

A

elective ochiplexy

cryptorchidism may resolve during first 6 months of life but if not surgery to bring testes down

How well did you know this?
1
Not at all
2
3
4
5
Perfectly
538
Q

What is the most common adverse event in a hospitalized patient who is not undergoing surgery?

A

adverse drug events

in surgery pts it is wound infections, bleeding and DVTs

How well did you know this?
1
Not at all
2
3
4
5
Perfectly
539
Q

What are the 4 criteria that are used to determine whether a patient has SIRS (systemic inflammatory response syndrome- only need 2 of them)?

A
  1. temp greater than 38.5
  2. HR greater than 90
  3. respiratory rate greater than 20
  4. WBC greater than 12,000
How well did you know this?
1
Not at all
2
3
4
5
Perfectly
540
Q

What is the best initial step in management of a pt with SIRS (systemic inflammatory response syndrome)?

A

aggressive fluid resuscitation until central venous pressure is 8-12 mmHg

(use pressors if not responsive to fluids)

How well did you know this?
1
Not at all
2
3
4
5
Perfectly
541
Q

What is the mechanism responsible for exacerbated hypotension in patients on long term corticosteroids who develop septic shock?

A

suppression of pituitary-adrenal axis

long term use of steroids negatively feedbacks to decrease ACTH causing decrease cortisol response to septic shock

How well did you know this?
1
Not at all
2
3
4
5
Perfectly
542
Q

A pt presenting with episodes of chest pain and dysphagia lasting a few seconds to minutes, has a classic corkscrew esophagus on esophagogram, and manometric studies show simultaneous high amplitude peristaltic contractions with normal relaxation of lower esophageal sphincter most likely suffers from …

A

Diffuse Esophageal Spasm

tx: antispasmodics, dietary modification, psychiatric counseling

543
Q

A female pt presenting with complaints of food sticking in the throat, halitosis and regurgitation of food most likely suffers from …

A

Zenker’s Diverticulum

tx: surgery

544
Q

What are the symptoms described by SMIGECAPS for diagnosis of depression?

A
S: sleep disorder
M: mood depressed
I: interest deficit (anhedonia)
G: guilt (worthlesness, regret)
E: energy deficit
C: concetration deficit
A: appetite changes (weight loss/gain)
P: psychomotor agitation/ retardation
S: suicidality

(depression dx requries at least 5 with 1 being depressed mood for at least 2 weeks)
(tx: SSRIs)

545
Q

What is the next best step in management for a pt with symptoms suggestive of coronary heart disease (chest pain on exertion) with a normal EKG and inability to exercise?

A

adenosine myocardial perfusion imaging

avoid if pt has asthma or hx of bronchospasm

546
Q

What are the recommendations for lung cancer screening?

A

chest CT yearly in pts aged 55-80 who have a 30 pack year smoking history and are current smokers or quit smoking within the last 15 years

(less than 10% of pts with positive CT- noncalcified nodule 4 mm or greater actually have lung cancer)

547
Q

What is the best next step in management of a preterm labor (24 to 24 weeks gestation)?

A

intramuscular betamethasone or dexamethasone

to reduce risk of infant respiratory distress syndrome and intraventricular hemorrhage

548
Q

What is the most important recommendation for a pt with suspected infectious mononucleosis (fever, fatigue, sore throat, tender posterior cervical lymphadenopathy, splenomegaly, atypical lymphocytes, rash after amoxicillin treatment)?

A

avoid sports

(to reduce risk of splenic rupture)

(rash due to circulating immune complexes; discontinue antibiotic)

549
Q

What is the best next step in a pt with adult polycystic kidney disease who is considering peritoneal dialysis?

A

colonoscopy (rule out diverticula that can affect peritoneal dialysis)

550
Q

What is the next best step in management of a diabetic pt with symptomatic multivessel coronary artery disease?

A

Coronary Artery Bypass Graft (CABG)

551
Q

What is the first line treatment for hyperbilirubinemia (generally bilirubin greater than 18 mg/dL)?

A

Phototherapy (convert bilirubin to water soluble form)

use exchange transfusion if phototherapy ineffective or bilirubin levels already at toxic level

552
Q

A female child presenting with short stature, hypogonadism, short fourth metacarpal bones, multiple nuchal folds, and wide spaced nipples most likely suffers from …

A

Turner Syndrome (XO karyotype)

cardiac abnormalities, visual and hearing deficits, horseshoe kidney, hypothyroidism, streaked gonads

553
Q

An adolescent found to have isolated proteinuria most likely suffers from …

A

Orthostatic Proteinuria

elevated protein excretion during day (upright) and normal excretion at night (supine)
(benign, no treatment)

554
Q

What two diagnostic tests can be used to diagnose orthostatic proteinuria?

A
  1. split 24 hour urine collection for protein (elevated in day, normal at night)
  2. comparing urine protein- creatine ratio in urine samples collected in supine and standing positions
555
Q

What is the next best step in management of a pt with a glasgow coma score of 8 or lower?

A

Intubate (to maintain adequate airway)

556
Q

What is the best step in management of a pregnant woman who is Rh (D) negative and has an anti-D antibody screen that is negative if the father is Rh positive or status is unknown?

A

anti-D immune globulin at 28 weeks gestation

as well as peripartum to reduce risk of isoimmunization due to fetomaternal transfusion during delivery

557
Q

What is the best step to smoking cessation in a patient with history of seizure disorder or eating disorder?

A

varenicline or nicotine replacement therapy (patch-plus combination: long acting with short acting)

558
Q

What is the best initial test for suspected infectious mononucleosis?

A

Monospot testing

(detects heterophile antibodies that agglutinate horse RBCs)

(if monospot negative: perform EBV specific antibodies- IgM and IgG viral capsid antigen without IgG EBV nuclear antigen)

559
Q

What is the best treatment for infectious mononucleosis (due to EBV)?

A

bed rest and NSAIDs

560
Q

What is the next best step in management of a pt with infectious mononucleosis who develops shortness of breath while recumbent, pharyngeal edema and swelling of soft tissues of the nexk?

A

admit and start IV corticosteroids

(steroids for infectious mononucleosis with airway obstruction imminent, overwhelming infection, aplastic anemia, thrombocytopenia)

561
Q

What is the best treatment for community acquired pneumonia (fever, cough, tachypnea, adventitious lung sounds) in a pre-school aged child or a child with focal lung findings?

A

high dose Amoxicilin

likely due to strep. pneumoniae

562
Q

Whatis the best treatment for community acquried pneumonia (fever, cough, tachypnea, adventitious lung sounds) in older child or well appearing child with bilateral lung findings?

A

Azithromycin

likely mycoplasma pneumoniae

563
Q

What is the sudden onset of hyperglycemia (insulin resistance) in a patient receiving total parenteral nutrition (TPN) suggestive of?

A

Sepsis

564
Q

What is the next best step in management for a pt with fractures following minor trauma (ground level fall)?

A

Dual-Energy X-ray Absorptiometry

DEXA scan for osteoporesis
(T score of -1 to -2.5: osteopenia)
(T score of -2.5 or lower: osteoporesis)

565
Q

When is treatment of osteoporesis with bisphosphonates recommended in postmenopausal women?

A
  1. have T score of -2.5 or less

2. have hx of low-trauma hip or vertebral fracture regardless of T score

566
Q

How should amputated body parts be transported to preserve ability to replant body part on pt?

A

wrap in gauze, moisten with saline and place in sealed, sterile plastic bag and place bag in containerwith ice mixed with saline/ sterile water (maintain cold ischemia)

567
Q

What is the best initial step in management of a pt with end stage renal disease who has anemia with hemoglobin less than 10 g/dL?

A

erythropoietin stimualting agents (epo)

568
Q

What is the next best step in management of a pt with end stage renal disease with anemia (hemoglobin less than 10 g/dL) with a transferrin saturation less than or equal to 30% and ferritin less than or equal to 500 ng/mL?

A

IV iron supplementation

569
Q

A pt presenting with intensely itchy rash that is worse at night and located in flexor surfaces of wrist/ lateral surface of fingers/ and finger webs and looks like excoriations with multiple small crusted red papules with linear burrows most likely suffers from ….

A

Scabies

dx: skin scrapings revealing mites, ova and feces under light microscopy

570
Q

What is the best initial treatment for scabies?

A

topical 5% permethrin cream or oral ivermectin

place beeding and clothes in plastic bag for 2-3 days after cleaning to eradicate mite

571
Q

A pt with history of periodic restrosternal chest pain and transient dysphagia presenting with difficulty swallowing solid foods, need for prolonged and careful chewing and drinking excess liquids with meals, and swallowing small portions most likely suffers from while being young and lacking alarming symptoms …

A

Esophageal Stricture

complication of untreated GERD and erosive esophagitis

572
Q

What are common causes of decreased vision in elderly patients?

A
  1. cataracts
  2. macular degeneration

(associated with each other)

573
Q

What is the next best step in managment of an infant with an elevated hematocrit (greater than 65%) on heel prick (capillary sample)?

A

recheck hematocrit using peripheral venous blood

neonatal polycythemia if hct higher than 65% or Hb greater than 22 g/dL

574
Q

What is the treatment for symptomatic polycythemia in an infant (irritability, drowsy, poor feeding, hypotonia, abdominal distension, plethoric, hypoglycemia, jaundice, apneic)?

A

IV hydration and partial exchange transfusion

problems due to hyperviscosity

575
Q

An elderly pt presenting with calcium elevation, renal insufficiency, anemia (normocytic), and bone pain (lytic lesions) (CRAB symptoms) along with hyperproteinemia most likely suffers from …

A

Multiple Myeloma

(dx: serum and urine protein electrophoresis showing monoclonal protein and bone marrow biopsy showing greater than 10% clonal plasma cells)

576
Q

What is the next best step in management of a pt with recent diagnosis of multiple myeloma?

A

complete X-ray skeletal survery (to assess for fractures, lytic lesions)

577
Q

A pt with hx of multiple myeloma who develops nasal/ oral bleeding, blurry vision, confusion, headache and possibly heart failure most likely suffers from …

A

Hyperviscosity Syndrome

tx: plasmapheresis

578
Q

What should a pt with diagnosis of Marfan’s syndrome and aortic root dilation/ moderate to severe mitral regurgitation/ family history of aortic dissection/ sudden death be counselled on?

A

avoid strenous physical activity

579
Q

A pt presenting with acute onset chest pain, dyspnea, and fever following repeated episodes of vomiting along with pleural effusion on CXR most likely suffers from ..

A

Esophageal Perforation (Boerhaave’s syndrome)

(associated with pneumomediastinum- radiolucent band adjacent to cardiac border and pneumothorax)
(fever suggests mediastinitis)

580
Q

What is the best diagnostic test for esophageal perforation (Boerhaave’s syndrome)?

A

esophagogram with water soluble contrast

second line is barium contrast if high suspicion

581
Q

A pt presenting with a rash that started as a small, elevated red area and gradually involved the surrounding skin to become erythematous, itchy and painful along with new onset diabetes most likely suffers from …

A

Glucagonoma

skin rash is necrolytic migratory erythema
(dx: glucagon level; tx: surgical removal)

582
Q

What acne treatments are associated with the adverse effect of photosensitivity? (2)

A
  1. benzoyl peroxide

2. tetracycline and doxycycline

583
Q

What is the best treatement for photosenstivity rash/ sunburn?

A

fluid replenishment and NSAIDs (ibuprofen and indomethacin)

584
Q

What activity should be avoided in patients who are being treated for acne with isotretinoin?

A

avoid alcohol consumption

isotretinoin causes hypertriglyceridemia which increases risk of pancreatitis especially with alcohol consumption

585
Q

What steps in management should be taken if a female pt of reproductive age is going to have her acne treated with isotretinoin?

A

have 2 negative pregnancy tests before start treatment and use 2 concurrent methods of contraception 1 month before, during and 1 month after

(assess plans for possible pregnancy)

586
Q

What is the best initial treatment for patients with hypertension and chronic kidney disease who are found to have proteinuria (in excess of 500-1000 mg/day)?

A

ACE inhibitor (or ARB)

then add diuretic and then calcium channel blocker if needed

587
Q

A pt who presents with rapid onset unilateral facial weakness in which the pt is unable to raise the eyebrow, close the eye or wrinkle the forehead on the affected side along with drooping of the mouth corner and disappearance of the nasolabial fold with an associated funny feeling at time of onset most likely suffers from …

A

Bell’s Palsy

7th cranial nerve neuropathy
(unilateral upper and lower facial weakness)

588
Q

What is the treatment for Bell’s Palsy?

A

corticosteroids within 3 days of onset and eye care if poor eyelid closure (artificial tears and eye patching)

589
Q

A pt who has a distal radius fracture possibly with shortening and dorsal displacement after falling on an outstretched hand most likely suffers from…

A

Colles’ fracture

associated with scaphoid fracture, ulnar styloid fracture, acute carpal tunnel syndrome and osteoporesis

590
Q

What is the best initial diagnostic test for routine screening for HIV?

A

HIV p24 antigen with HIV antibodies

confirm with HIV-1/HIV-2 differentiation immunoassay

591
Q

What is the most common short term complication after cervical conization (cold knife conization and LEEP)?

A

bleeding

592
Q

What is the next best step in management of a pregnant pt at less than 34 weeks gestation who has elevated blood pressure with proteinuria and without symptoms (nausea, vomiting, headache, SOB, upper abdominal pain, visual abnormalities)?

A

admit to hospital for further evaluation of severity of pre-eclampsia
(repeat BP measurements, lab tests, protein measurement, monitor for symptoms)

593
Q

What is the best treatment for catatonia (immobility, unresponsiveness, catalepsy- passively allowing examiner to position pt, echolalia- mimics speech, echopraxia- mimics movement, posturing- spontanous postures against gravity, waxy flexibility- resistance to positioning, mutism, negativism- resistance to instructions)?

A

Lorazepam (benzos) or ECT (if fail benzo or urgent case)

dx catatonia with lorazepam challenge test- give 1-2 mg and observe for partial, temporary relief withing 5-10 mins

594
Q

A pt presenting with flaky and itchy scalp associated with red, oily skin with inflammation and scaling of the scalp/ face/ upper trunk most likely suffers from ..

A

Seborrheic dermatitis

dandruff

595
Q

What is the best initial treatment for seborrheic dermatitis?

A

medicated shampoo (tar-based preparations, zinc pyrithione, selenium sulfate)

596
Q

An elderly pt with history of smoking who presents with recurrent pneumonia that responds to antibiotics most likely suffers from …

A

Bronchogenic carcinoma

tumor causes endobronchial obstruction preventing adequate clearance of secretions

597
Q

What is the best diagnostic test to evaluate a pt with persistent/ non-resolving pneumonia that responds to antibiotics (suggests endobronchial obstruction)?

A

Flexible bronchoscopy (visualize obstruction and biopsy)

best initial test would be high resolution CT scan- persistent scar

598
Q

What are the effects of intermittent heavy drinking (more than 7 drinks/ week or 3 drinks at 1 time in women; more than 14 drinks/week or 4 drinks at 1 time in men) and binge drinking (4 or more drinks at once in women; 5 or more drinks in men) in a diabetic pt? (3)

A
  1. elevated blood pressure
  2. unstable glucose levels (hypoglycemia with sulfonylureas)
  3. increased risk of cardiovascular disease (a-fib, dilated cardiomyopathy, coronary disease)
599
Q

What do overlapping standard error of measurement bars in a study’s results suggest?

A

non-statistically significant difference

600
Q

What is the most common complication of sodium glucose contransporter 2 inhibitor use in diabetic patients?

A

Vulvovaginal candidiasis

reduce hyperglycemia via promoting renal glucose excretion leading to glucosuria which increases infections

601
Q

A pt presenting with inability to extend the knee against resistance (quadriceps weakness), sensory loss over anteromedial aspect of thigh, medial shin and arch of foot, and decreased knee jerk reflex most likely suffers from …

A

Femoral Nerve Injury

arch of foot is saphenous nerve- branch of femoral

602
Q

What is the best treatment for Lyme disease in a pregnant patient?

A

amoxicillin or cefuroxime

603
Q

What are the most common complications associated with silicone breast implants? (2)

A
  1. capsular contracture resulting in pain
  2. shape distorion/ implant deflation/ implant rupture

(no effect to fetus during pregnancy or to infant during breast-feeding)
(routine mamamogram screening for breast cancer and MRI every 2-3 years to screen for rupture recommended)

604
Q

What is the best step to confirm eradication of H. pylori infection after triple therapy for H. pylori positive ulcers?

A

urea breath test or fecal antigen testing 4 weeks after completing therapy

605
Q

What is the pathophysiologic mechanism responsible for heparin induced thrombocytopenia type 2?

A

antibodies to platelet factor 4 (PF4) complexed with heparin

606
Q

What is the best recommendation for patients who have a history of HIT (heparin induced thrombocytopenia)?

A

avoid all forms of heparin (low molecular weight, unfractionated) for lifetime including heparin flushes and heparin coated catheters

(heparin allergy)

607
Q

A pt presenting with hyperventilation (breathing heavy), tinnitus, restlessness, nausea, vomiting and mild GI distress that progresses to drowsiness and loss of consciousness, acute renal failure, elevated aminotransferases, and metabolic acidosis with elevated anion gap (especially in setting of history of drug abuse or suicide attempt) most likely suffers from …

A

Salicylate (aspirin) Intoxication

hyperventilation without dyspnea is respiratory alkalosis compensating for metabolic acidosis

608
Q

What is the treatment for salicylate (aspirin) intoxication?

A

alkalinization of urine (if salicylate level greater than 35 mg/dL), gastric lavage, activated charcoal

609
Q

A pt who presents with history of recurrent headaches the last 30 minutes to 3 hours and assocaited with severe retro-orbital pain, lacrimation, conjunctival injection, rhinorrhea, sweating and pallor most likely suffers from ..

A

Cluster Headaches

treat acutely with 100% oxygen or second line is sumatriptan, NSAIDs, ergotamine

610
Q

What is the best initial treatment for prevention of cluster headaches?

A

verapamil

assess for cardiac effects with EKG
(prednisone and lithium are alternatives)

611
Q

What diagnostic test can be used to assess for the loss of vibration sense in a pt with suspected diabetic neuropathy?

A

tuning fork test

612
Q

What are the three drugs that are used as first line treatment for pain associated with diabetic neuropathy?

A
  1. duloxetine (SNRI)
  2. pregabalin
  3. TCAs
613
Q

What is the best initial diagnostic test for imaging skeletal metastasis for malignancies that are primarily osteoblastic (prostate cancer, small cell lung, Hodgkin lymphoma)?

A

radionuclide bone scan

614
Q

What is the best initial diagnostic test for imaging skeletal metastasis for malignancies that are primarily osteolytic (multiple myeloma, non-small cell lung, non-Hodgkin lymphoma)?

A

X-ray and PET scans

615
Q

What is the best step in management of a pt with a lightening injury resulting in asytole or ventricular fibrillation?

A

uninterrupted CPR (continue CPR while giving epinephrine or vassopressin)

616
Q

What medical treatment can be used before diving to prevent the incidence of ear and sinus barotrauma?

A

non-sedating decongestants (pseudoephedrine)

617
Q

What factor is responsible for increased risk of transverse limb abnormality when undergoing chorionic villus sampling (CVS)?

A

age of gestation

high risk if gestation less than 9 weeks

618
Q

What is the best follow-up procedure for a pt with autosomal dominant polycystic kidney disease?

A

regular blood pressure checks

hypertension is poor prognostic factor

619
Q

What is the best initial treatment for hypertension in a pt with autosomal dominant polycystic kidney disease?

A

ACE inhibitors

620
Q

What is the most common extrarenal manifestation of autosomal dominant polycystic kidney disease?

A

hepatic cysts

621
Q

What is the next best step in management for family members of a pt with autosomal dominant polcystic kidney disease?

A

abdominal ultrasound

to assess kidney- 3 to 5 cysts in each kidney

622
Q

What is the best preventative step in patients with chronic liver disease (chronic Hep C infection)?

A

Hep A vaccination in unvaccinated

hepatic decompensation and liver failure if develop Hep A while have chronic Hep C

623
Q

What is the prognosis for a child who is diagnosed with attention deficit hyperactivity disorder (ADHD)?

A

one to two thirds of kids witll experience ADHD symptoms into adulthood but medication is symptomatically beneficial

624
Q

True or False. Stimulant therapy for attention deficit hyperactivity disorder does increase the risk of developing substance use disorder.

A

False

current research does not support increase risk

625
Q

What is the best initial step in management of shoulder dystocia (difficulty delivering baby’s shoulder) during delivery?

A

BECAL(M)
B- breath, dont push; lower head of bed
E- elevate legs into McROberts position (sharp hip flexion while supine)
C- call for help
A- apply suprapubic pressure (downward and lateral)
L- enlarge vaginal opening with episiotomy
(m)- other maneuvers

626
Q

A trauma pt presents with unilateral decreased/ absent breath sounds with ipsilateral decreased chest excursions, hypoxia, hypotension, low oxygen saturation, and shifting of mediastinal structures (trachea, heart) most likely suffers from …

A

Tension Pneumothorax

tx: needle decompresion at 2nd/ 3rd intercoastal space in mid clavicular line followed by chest tube thoracostomy

627
Q

A pt presenting with nasal congestion, rhinorrhea, sneezing, and postnasal drainage that began after age 20 years old, has erythematous and boggy nasal mucosa, and has no known triggers but symptoms can wrosen with seasonal changes most likely suffers from …

A

Non-allergic Rhinitis

tx: intranasal corticosteroids/ antihistamines

628
Q

A pt being treated for a “mood disorder” who presents with gradual onset of ataxia, sluggishness, confusion, agitation, coarse tremor, and GI distress most likely suffers from …

A

Lithium Intoxication

(can ocurr with drug interaction with thiazide diuretics/ ACE inhibitors/ non-aspirin NSAIDs)
(other causes are overdose, volume depletion, elderly with low GFR)

(obtain lithium levels every 2-4 hours, IV hydration and bowel irrigation; hemodialysis if meet criteria)

629
Q

What are the criteria for nephrology consult and hemodialysis for a pt with lithium intoxication? (3)

A
  1. lithium level equal to or greater than 4 mEq/L
  2. significant symptoms (seizures, altered mental status) with lithium level of 2.5 mEq/L or greater
  3. increasing level despite IV fluids
630
Q

A pt presenting with speech abnormalities (echolalia, aphasia, mutism), impaired executive functioning, irritablity, hyper-oral behavior, disinhibition and progressive memory loss most likely suffers from ..

A

Pick’s Disease (Frontotemporal Dementia)

brain imaging shows symmetric atrophy of frontal and temporal lobes

631
Q

A pt presenting with acute painless unilateral vision loss, increased frequency of floaters and photopsias (flashing lights with vitresou hemorrhage and marked elevation of retina on funduscopy most likely suffers from ..

A

Retinal Detachment

632
Q

What factor makes it easier to detect the decreased ability to discriminate speech in elderly patients suffering from presbycusis?

A

noisy, distracting environemnt

background noise

633
Q

A male presenting with scrotal pain accompanied by swelling and tenderness along with a decrease in pain on testicular elevation (Prehn’s sign) and normal cremasteric reflex most likely suffers from…

A

Epididymitis

usually due to Chlamydia trachomatis infection

634
Q

A male pt presenting with severe scrotal pain, absence of cremasteric reflex, and a high riding testis on palpation most likely suffers from …

A

Testicular Torsion

635
Q

A male pt preseenting with dull scrotal pain that is relieved by recumbency (laying down) and a soft scrotal mass palpated that feels like bag of worms that disappears when recumbent most likely suffers from …

A

Varicocele

636
Q

An infant (older than 6 months) presenting with symptoms of anemia with associated low hemoglobin (less than 11), low MCV and low total red bblood cell count resulting in a Mentzer index greater than 13 most likely suffers from ..

A

Iron deficiency Anemia secondary to breast-milk predominant diet

(breast milk low in iron for 6 month and older infant, need iron fortified foods)

637
Q

What is the Mentzer index and what is it used for?

A

Mentzer index = MCV/ total RBC count; used to distinguish between iron deficiency anemia and thalassemia

(Mentzer greater than 13: iron deficient)
(Mentzer less than 13: thalassemia)

638
Q

What is the best initial treatment for recurrent pneumonia in a cystic fibrosis patient?

A

2 antipseudomonal agents (tobramycin with cefepime/ amikacin/ imipenem/ piperacillin-tazobacam) with vancomycin (if have hx of MRSA)

639
Q

What is the best initial step in management for pt with Allergic Rhinitis?

A

avoidance of trigger/ allergen (followed by intranasal corticosteroids)

640
Q

.. is a potential problem in case- control studies and is a result of inaccurately reporting the exposure status by the participants in a study and leads to ….

A

Recall bias; misclassification of exposure

641
Q

A female pt with history of maternal birth trauma presenting with postvoid dribbling followed by development of dysuria and dyspareunia (pain with sex) most likely suffers from …

A

Urethral Diverticulum (3 D’s)

can have hematuria, recurrent UTIs, stress urinary incontinence

642
Q

A female pt presenting with dysmenorrhea, dyspareunia (pain with sex), dyschezia (pain with defecation), hematochezia, hematuria and premenstrual/ postemenstrual spotting along with anterior vaginal wall fullness (possibly infertility issues) most likely suffers from …

A

Endometriosis

dx: laparoscopy- visualize ectopic endometrial tissue
(tx: NSAIDS, GnRH analogs, danazol, OCPs, surgery)

643
Q

What are two non-invasive diagnostic tests used to diagnosis urethral diverticulum?

A
  1. transvaginal ultrasound

2. MRI

644
Q

What is the formula for odds ratio (used as measure of association in case-control study)?

A

OR = (a * d)/ (b * c)

OR = (a/c) / (b/d)

(a= disease with exposure; c= disease without exposure; b= not diseased with exposure; d= not diseased wihtout exposure)

645
Q

What potential development is of most concern in a pt who was rescued from a burning building?

A

supraglottic edema (inhalation injury that can obstruct airway)

646
Q

What is the most sensitive imaging modality for diagnosing osteonecrosis of the femoral head (avascular necrosis/ osteochondritis dissecans)?

A

MRI

increased risk of disease if steroid use and excessive alcohol consumption

647
Q

What is the prognosis of infantile hemangiomas with no functional impairment or sign of ulceration?

A

proliferate during first year of life and spontaneously involute by age 7-10 years

648
Q

What is the best initial treatment for complicated or disfiguring infantile hemangiomas (visceral location, airway location, periorbital location)?

A

propanolol (non-selective beta blocker)

649
Q

A child (between 6 months and 3 years old) presenting with intermittent severe crampy abdominal pain, palpable sausage shaped mass on right side of abdomen, and currant jelly stools (bright red rectal bleeding) who may draw legs up to the abdomen during episodes of pain and vomiting most likely suffers from ..

A

Intussusception
(telescoping of proximal portion of intestine into distal portion)

(tx: air or water soluble enema)

650
Q

What is the best initial diagnostic test for suspected intussusception?

A

abdominal ultrasound

shows target sign of intestines

651
Q

What is the major complication associated with enema reduction of intussusception?

A

intestinal perforation

652
Q

What is the appropriate duration and rationale for treatment of streptococcal pharyngitis?

A

10 days of oral penicillin to prevent acute rheumatic fever

can give 5 day course of azithromycin to penicillin-allergic patients

653
Q

A male infant aged 3-6 months old presenting with non-bilious, non-bloody postprandial projectile vomiting and is immediaely hungry after vomiting along with palpable olive shaped mass in RUQ, and hypokalemic, hypochloremic metabolic alkalosis most likely suffers from …

A

Pyloric Stenosis

dx: abdominal ultrasound; can also use upper GI contrast study

654
Q

What is the best initial step in management of an infant with pyloric stenosis?

A

correct electrolyte imbalance (hypokalemic, hypochloremic metabolic alkalosis) and dehydration then pyloromyotomy

655
Q

What antibiotics is associated with the development of pyloric stenosis?

A

Oral erytromycin (usually given to infant for pertussis)

656
Q

What is the best next step in management of an acutely maniac lithium treated pt with low normal blood level of lithium?

A

increase lithium to achieve high normal blood level of lithium

657
Q

What is the next best step in management of an acutely maniac lithium treated pt with high normal blood level of lithium?

A

augment lithium by adding antipsychotic (risperidone)

658
Q

What is the best next step in management of a pregnant pt who has hypertension, proteinuria, hyperreflexia (brisk deep tendon reflexes), and edema?

A

Hydralazine (or labetalol to lower BP) and Magnesium sulfate (prevent progression into eclampsia)

(followed by delivery if term)

659
Q

What is considered a full/ complete trial of an antidepressant therapy for a depressed pt before considering switching agents or adding agents?

A

at least 6 weeks on SSRIs

660
Q

What co-morbidity has a high risk of adverse effects with the use of varenicline for smoking cessation?

A

Major depression (as well as unstable psychiatric symptoms, history of suicidal ideation)

(increases depressive symptoms)

661
Q

What is the best initial treatment for H. pylori infection?

A

triple therapy with pantoprazole, amoxicillin (1 gram twice a day), and clarithromycin (500mg) for two weeks

662
Q

What is the best initial treatment for H. pylori infection in a penicillin allergic patient?

A

triple therapy with pantoprazole, metronidazole (500 mg twice a day), and clarithromycin (500mg) for two weeks

663
Q

What is the next best step in management of a pt with H. pylori infection who fails initial therapy with triple therapy? (2)

A
  1. quadruple therapy (PPI, bismuth, tetracycline, and metronidazole for 2 weeks with meals)
  2. use different combination of triple therapy
664
Q

A pt presenting with decreased sensation over the anterolateral thigh (with numbess and burning sensation) without muscle weakness or deep tendon reflex abnormalities most likely suffers from …

A

Meralgia Paresthetica due to lateral femoral cutaneous nerve entrapment

(dx: clinical)

665
Q

What is the treatment for meralgia paresthetica (lateral femoral cutaneous nerve entrapment)?

A

reassurance, weight loss and avoid tight fitting clothes

666
Q

What is the diagnostic workup that should be performed in a child age 5 years or older suffering from enuresis (urinary incontinence)?

A
  1. urinalysis (assess specific gravity, glucosuria, signs of infection)
  2. urologic imaging if significant daytime symptoms and history of UTIs

(rule out secondary enuresis: psychological stress, UTI, diabetes mellitus, diabetes insipidus, obstructive sleep apnea)

667
Q

A pt presenting with skin lesions that are shiny, discrete, intensely pruritic, polyglonal shaped violaceous plaques and papules on flexural surfaces on extremities (especially wrist, mucous membranes) with an associated whitish, lacy pattern on the lesion surface (Wickham striae) most likely suffers from …

A

Lichen Planus

dx: clincial but can due punch biopsy

668
Q

What is the next best step in management of a pt with Lichen Planus?

A

obtain anti-hep C antibodies

lichen planus is associated with hep C infection

669
Q

True or False. Household contacts of transplant patients and pregnant patients can be safely be vaccinated with varicella vaccine and is recommended.

A

True

(monitor for development of vaccine-associated rash and isolated from immunocompromised individual if present until lesions crusted over)

670
Q

What are contraindications to varicella-zoster vaccination? (4)

A
  1. anaphylaxis to neomycin
  2. anaphylaxis to gelatin
  3. pregnancy
  4. immunodeficient state (congenital immunodeficiency, long term immunosuppresive therapy, HIV, hematologic/ solid tumor)
671
Q

What is the best next step in management of a pt presenting with low back pain along with radiculopathy (sciatica symptoms) or red flags (night time pain, older than 50, unexplained weight loss, history of malignancy)?

A

plain X-rays of back and ESR level

if no red flags or signs of cord compresion, conserative therapy- PT and analgesics for 4-6 weeks

672
Q

What is the next best step in management of a pt presenting with low back pain along with bowel/bladder dysfunction, saddle anesthesia and/ or motor weakness?

A

MRI of back (to assess for cauda equina syndrome)

673
Q

A pt presenting with dyspnea and voice hoarseness with a loud S1 (first heart sound) and an opening snap followed by a low pitched mid-diastolic rumbling murmur heard best at the cardiac apex (between 5th and 6th rib at left mid-clavicular line) most likely suffers from..

A

Mitral Stenosis

(hoarsness from compression of recurrent larygneal nerve by enlarged right atrium)
(most common cause if rheumatic heart disease)

674
Q

A pt with a mid-systolic ejection murmur heard best at the left upper sternal border along with a wide and fixed splitting of the second heart second, and right atrial and ventricular dilation on ECHO most likely suffers from …

A

Atrial Septal Defect

(can be asymptomatic until adulthood; present with decreased exercise tolerance, pulmonary HTN, right heart failure, stroke due to paradoxical embolization, atrial arrhythmias)

675
Q

A pt presenting with short statue, facial dysmorphism and congenital heart defect most likely suffers from …

A

Noonan syndrome

676
Q

A female pt presenting with acute abdomen (abdominal tenderness with rebound and guarding), a positive pregnancy test, and transvaginal ultrasound showing adnexal mass with empty uterus and/or echogenic fluid in the cul-de-sac, around adnexa, in Morrison’s pouch most likely suffers from …

A

Ruptured Ectopic Pregnancy

tx: surgical evaluation

677
Q

A pt with chronic GERD symptoms along with (age older than 50, male, white, hiatal hernia, increased BMI, intraabdominal body fat distribution or tobacco use) who is found to have velvety, reddish mucosa extending from the squamocolumnar junction into the distal esophagus most likely suffers from ..

A

Barrett’s esophagus

metaplastic columnar epithelium in the esophagus replacing normal stratified squamous epithelium

678
Q

What is the next best step in the management of a pt found to have Barrett’s esophaus (columnar lined esophagus) without signs of dysplasia?

A

endoscopic surveillance every 3-5 years

to assess for adenocarcinoma formation

679
Q

What is the next best step in management of a pt found to have Barrett’s esophagus (columnar lined esophagus) with signs of low-grade dysplasia?

A

endoscopic surveillance every 6-12 months

to assess for adenocarcinoma formation

680
Q

A child presenting with a rash that starts as erythematous macules that coalesce into non-blanching, non-pruritic purple colored palpable purpura in gravity dependent areas, arthritis/ arthralgia, abdominal pain, hematuria with/ wihtout RBC casts and proteinuria, and normal platelet level most likely suffers from …

A

Henoch- Schonlein Purpura

(IgA-mediated leukocytoclastic vasculitis)

(associated with intussusception, renal failure, prior URI)

681
Q

What is the significance of the correlation coefficient in study results?

A

shows strength and direction (positive or negative) of linear association between 2 variables

(not causality)

682
Q

A type 2 diabetic presenting with confusion, lethargy, markedly elevated glucose, serum bicarbonate near lower limits of normal, dry mucous membranes, acute renal failure, negative ketones, hyperkalemia and abnormal sodium most likely suffers from ..

A

Hyperosmolar Hyperglycemic State

683
Q

What is the best initial step in management for hyperosmolar hyperglycemia state in a type 2 diabetic?

A

agressive fluid resuscitation

isotonic saline

684
Q

What type of calcification pattern on pulmonary nodules are strongly suggestive of benign lesions? (3)

A
  1. popcorn calcifications
  2. central calcifications
  3. diffuse homogenous calcifications

(eccentric, reticular and punctate calcifications and spiculated margins are suspicious for malignancy)

685
Q

What is the next best step in management of a young pt who experiences a ischemic stroke in the setting of having deep venous thrombosis?

A

transesophageal echo (and bubble study)

assess for intracardiac communication like patent foramen ovale or ASD allowing for paradoxical emboli

686
Q

What is the most common complication of transurethral resection of the prostate (TURP) used in the management of benign prostatic hyperplasia (BPH)?

A

retrograde ejaculation

bladder neck fails to close allowing sperm to flow backward to bladder

687
Q

A pt who presents with periodic involuntary eye closure that can be provoked by external stimuli (bright light or irritants) most likely suffer from …

A

Blepharospasm (focal dystonia)

688
Q

What is the best initial treatment for blepharospasm?

A

botulinum toxin injections

689
Q

… is a type of statistical analysis that is useful in studying the difference in mean values (of a continuous variable) among several groups (catergorical variables)

A

Analysis of Variance (ANOVA)

homoscedasticity and normality must be true to make test valid

690
Q

An infant presenting with failure to pass meconium, bilious emesis, and markedly distended abdomen who is found to have multiple dilated loops of large bowel and no air in the rectum on abdominal X-ray and a normal caliber rectosigmoid colon, transition zone and dilated descending colon with constrast enema most likely suffers from …

A

Hirschsprung Disease

(aganglionic intestinal disease

(dx: rectal mucosal suction biopsy)
(tx: surgical resection of aganglionic portion of bowel)

691
Q

A pt presents with history of urge to move legs and unpleasant sensations in the legs that begins or worsens during inactivity (lying down, sitting) and is relieved by movement (walking, stretching) most likely suffers from …

A

Restless Leg Syndrome

692
Q

What is the best treatment for mild/ intermittent symptoms of restless leg syndrome? What is the best treatment for persistent/ moderate to severe symptoms of restless leg syndrom?

A

mild: supplement iron if ferritin less than 75 , avoid sleep deprivation, use leg massage/ heating pads/ exercise
persistent: dopamine agonists (pramipexole)

693
Q

What diagnostic test is usedin cases of suspected restless leg syndrome?

A

ferritin level

iron deficiency anemia is assocaited with restless leg syndrome

694
Q

A diabetic pt with history of peripheral neuropathy and labile glycemic control (multiple episodes of hypoglycemia) presents with nausea vomiting, early satiety, postprandial fullness, bloating, abdominal pain and distension and weight loss most likely suffers from …

A

Gastroparesis

dx: nuclear gastric emptying study after ruling out obstruction with upper GI endoscopy and compression with CT

695
Q

What is the treatment for gastroparesis?

A

dietary modification (small more frequent meals consisting of less fat and fiber intake)

(if fails, erythromycin or metoclopramide)

696
Q

What is the most accurate physcial exam testto detect Achilles tendon rupture?

A

Thompson squeeze test

squeeze gastrocnemius muscle and assess for lack of plantar flexion

697
Q

A pt presenting with fever, headache, malaise, intense muscle pain (myalgias), intense joint pain (arthalgias), retro-orbital pain, and a rash described as white sands in sea of red, petechiae formation after BP cuff placed on arm, and thrombocytopenia in the setting of recent travel most likely suffers from …

A

Dengue Hemorrhagic Fever

(common complication is Shock due to increased capillary permeability leading to hemoconcentration, thrombocytoenia, prolonged fever, respiratory/ circulatory failure)

698
Q

A pt presenting with episodic epigastric or RUQ pain described as dull with radiation to the back and right shoulder, and the episodes last less than 6 hours and can be associated with meals or occur at night along with nausea, vomiting and diaphoresis most likely suffers from …

A

Biiary Colic (due to gallbladder contracting and forcing gallstones into cystic duct)

699
Q

What is the best initial diagnostic test for biliary colic?

A

abdominal ultrasound

to assess for gallstones after 8 hour fast

700
Q

What is the next best step in management of a pt with gallstones on abdominal ultrasound and biliary colic symptoms?

A

acute pain management and elective prophylactic cholecystectomy

(if pt is poor surgical candidate or refuse surgery, trial of ursodeoxycholic acid)

701
Q

What is the next best step in mangement of a pt with biliary colic symptoms without gallstones on imaging?

A

cholecystokinin stimulated cholescintigraphy

to evaluate functional gallbladder disorder- surgery if low gallbladder ejection

702
Q

A pt presenting with episodic epigastric or RUQ pain, described as dull with radiation to the back and right shoulder along with nausea, vomiting, diaphoresis, elevated liver function studies but normal lipase levels most likely suffer from ….

A

Choledocholithiasis
(common bile duct stone)

(tx: endoscopic retrograde cholangiopancreatography (ERCP)- also used to confirm diagnosis if ultrasound shows dilated common bile duct without apparant stone)

703
Q

An infant presenting during first week of life with unconjugated hyperbilirubinemia, jaundice, and dehydration in the setting of being breast-fed most likely suffers from …

A

Breastfeeding Failure Jaundice

suboptimal breastfeeding

704
Q

What is the next best step in management for an infant with unconjugated hyperbilirubinemia secondary to breast milk jaundice who is gaining weight adequately and has mildly elevated bilirubin?

A

continue breastfeeding and consider phottherapy if levels continue to rise (especially after two weeks of life)

705
Q

A pt presenting with limb pain with associated pallor, paresthesia, pulselessness, poikilothermia (feels cold) and paralysis most likely suffers from …

A

Acute Limb Ischemia

6 P’s

706
Q

What charcaterizes a viable limb versus threatened limb versus nonviable limb in a pt with acute limb ischemia?

A

viable: mild pain, no sensory/motor deficit, cap refill intact, audible doppler pulses (tx: catether based TpA or surgicial revasularization
threatened: severe pain, mild sensory/motor deficit, delayed cap refill, inaudible pulses on arterial doppler but audible on venous (tx: emergency surgical revascularization)
nonviable: pain, severe sensory/motor deficit, absent cap refill, inaudiable doppler pulses (tx: amputation)

707
Q

A male pt presenting with dysuria, dull aching pain in perineal region and testicles, painful ejaculation, blood in semen with a tender and boggy prostate on exam most likely suffers from ..

A

Prostatitis

708
Q

What is the best initial step in mangement of a pt with suspected prostatitis?

A

obtain urinalysis and urine culture

709
Q

A pt with signs and symptoms of prostatitis whose culture of the prostatic secretion shows elevated leukocytes and no bacteria most likley suffer from …

A

Chronic nonbacterial prostatitis

tx: sitz baths and anti-inflammatory meds

710
Q

What is the most common complication associated with endometriosis?

A

intestinal obstruction

due to endometriosis involving the intestinal wall

711
Q

An immigrant pt presenting with chronic diarrhea, weight loss, iron deficiency anemia and eosinophilia most likely suffers from …

A

Helminthic Infection (Intestinal Parasitosis)

risk of reinfection if adequate hygenic measures are not taken

712
Q

What medications can result in decreased warfarin metabolism leading to over-anticoagulation in pts on stable dose of warfarin? (7)

(reduce warfarin dose by 25-50% when initiating new drug)

A
  1. NSAIDs
  2. amiodarone
  3. levothyroxine
  4. SSRIs
  5. omeprazole
  6. cranberry juice, ginkgo biloba, vitamin E
  7. antibiotics/ antifungals
713
Q

What medications can result in increased warfarin metabolism leading to under-anticoagulation in pts on stable dose of warfarin? (6)

(increase warfarin dose by 25-50% when initiating new drug)

A
  1. rifampin
  2. carbamazepine
  3. OCPs
  4. ginseng
  5. St Johns wort
  6. green vegetables (spinach)
714
Q

What is the best management for a sickle patient experiencing an acute pain crisis (vaso-occlusive episode usually of back, chest, abdomen or extremity)?

A

pain med (IV morphine in ED; NSAIDS outpatient) and hydration (normal saline bolus if hypovolemic/ hypoensive; 1/4 or 1/2 NS if normal BP)

715
Q

What is the next best step in management of a sickle cell patient with recurrent vaso-occlusive episodes?

A

Hydroxyurea (increased fetal hemoglobin)

716
Q

A sickle cell pt who develops a new pulmonary infiltrate on CXR along with fever (greater than 38.5), hypoxemia, chest pain and/or respiratory distress (increased work of breathing, cough, tachypnea, wheezing) most likely suffers from …

A

Acute Chest Pain

tx: ceftriaxone and azithromycin; along with IV fluids and pain control

717
Q

What are the side effects of SSRIs (citalopram, fluoxetine, paroxetine, setraline)? (4)

A
  1. fatigue/ drowsiness/ insomnia
  2. weight changes
  3. sexual dysfunction
  4. serotonin syndrome
718
Q

An HIV pt presenting with headache, vomiting, fever, blurry vision, neck stiffness and papilledema who has CSF findings of elevated opening pressure (greater than 250-300), lymphocytic leukocytosis that is low (less than 50), elevated protein and low glucose most likely suffers from …

A

Cryptococcus Neoformans meningitis

(dx: India ink stain or cryptococcal antigen test)
(tx: amphotericin B and flucytosine for induction therapy followed by fluconazole for maintenance)

719
Q

What is the next best step in management of a pt with cryptococcal meningitis who initial responds to treatment with amphotericin B and flucytosine but then redevelops symptoms again?

A

repeat lumbar puncture (to decrease the increased intracranial pressure from outflow obstruction due to the yeast)

720
Q

What is the next best step in management of a pt with cryptococcal meningitis who clinically improves with amphotericin B and flucytosine treatment for 10-14 days?

A

stop amphotericin B and fucytosine therapy and start consolidation/ maintenance therapy with fluconazole

721
Q

Why is it important not to initiate HAART therapy in an HIV pt with cryptococcal meningitis before it has been 4-10 weeks after the initiation of antifungal treatment for the cryptococcus?

A

can develop immune reconstitution inflammatory syndrome (IRIS) that increases mortality

722
Q

What is the best initial step in management of a trauma pt with symptomatic intracranial hypertension (headache, vomiting, blurred vision, papilledema, bradycardia, respiratory depression, hypertension)?

A

endotracheal intubation

to maintain airway before possible brain herniation

723
Q

A pt presenting with fever, chills, flank pain, CVA tenderness, nausea and vomiting with bacteruria and pyuria on urinalysis most likely suffers from …

A

Pyelonephritis (kidney infection)

(tx: antibiotics; IV if vomiting/ fail oral/ hemodynamically unstable/ positive blood culture/ infant less than 2 months old)

724
Q

A pt presenting with dysuria, increased urinary frequency and urgency, hematuria and suprapubic pain most likely suffers from ..

A

Cystitis

725
Q

What is the earliest sign of hypovolemia?

A

Tachycardia

726
Q

What is the next best step in management for a pt with Clostridium difficile colitis who has leukocytosis less than 15000 and normal creatinine (less than 1.5)?

A

oral metronidazole (mild-moderate colitis)

727
Q

What is the best initial step in management for a pt with Clostridium difficile colitis who has leukocytosis greater than 15,000, creatinine greater than or equal to 1.5, or albumin less than 2.5?

A

Oral vancomycin (severe colitis)

if ileus, add IV metronidazole or switch to rectal vancomycin

728
Q

What is the best treatment for the first recurrence of Clostridium difficile colitis? What is the best treatment for the second recurrence of C. difficile colitis? What is the best treatment for the third recurrence of C. difficile colitis?

A

first: repeat oral metronidazole (for mild to moderate) or oral vancomycin (for severe)
second: pulsed tapered oral vancomycin for 6-7 weeks
third: fidaxomicin and consider fecal microbiota transplant

729
Q

What is the next best step in management of a pt with hypoparathyroidism being treated with high dose of vitamin D (calciferol) and calcium but still has borderline low serum calcium and high urinary calcium?

A

add thiazide diuretic (increases calcium reuptake in kidney to increase serum calcium and decrease urinary calcium)

730
Q

A pt presenting with hypertension, hypokalemia and possibly polyuria and polydipsia most likely suffers from …

A

Conn’s syndrome (primary Hyperaldosteronism)

dx: plasma aldosterone to renin ratio greater than 30

731
Q

what is the best initial diagnostic test for a pt who presenting with typical (crushing/ pressure lasting more than 20 minutes, provoked by exercise/ emotional upheaval and relieved with nitroglycerin or res) and/or atypical chest pain who has a normal EKG and is able to exercise?

A

Exercise EKG stress test

if not normal EKG, then exercise imaging test

732
Q

A pt presenting with blurry vision, facial muscle weakness, bilateral ptosis, dilated pupils with sluggish reactivity to light, dysarthria and preserved sensory function after eating home-canned food most likely suffers from …

A

Botulism

results in descending weakness

733
Q

A pt presenting with hematuria (red-colored urine) and bilateral flank pain with dysmorphic red blood cells on microscopic exam of urine a few days after a recent upper respiratory infection most likely suffers from …

A

IgA Nephropathy

734
Q

What is kappa statistic as it relates to a study?

A

quantitative measure of inter-rate reliability (inter-rate concordance) when 2 different individuals are rating a diagnostic test

735
Q

What treatment used for breast cancer targets cells that have HER-2/neu overexpression?

A

Trastuzumab

736
Q

What is the best initial screening tool to identify unhealthy alcohol use?

A

single-item screening: asking pt how many times in the past year has he had 5 or more (she had 4 or more) drinks in a day

737
Q

An elderly pt presenting with low back pain that worsens with extension of the spine and improves with flexion, when he sits down or bends forward (i.e. using grocery cart), pedal pulses are preserved and may have abnormal gait (spaghetti legs or walking like drunken sailor) most likely suffers from …

A

Lumbar spinal stenosis

dx: MRI of spine

738
Q

After stabilizing the pt, what is the best initial step in management of a pt who experienced a blunt chest trauma?

A

12 lead EKG

assess for fractures, pneumothorax, hemothorax, widened mediastinum, aortic injury

739
Q

A pt with a mid-systolic click followed by a late systolic murmur heard best at the LLSB most likely suffers from …

A

Mitral Valve Prolapse

740
Q

A child presenting with sore throat with a non-prurtic maculo-papular and vesicular rash on the palms, soles, genitalia and butt along with vesicles surrounded by a thin halo of erythema on the hard palate/ posterior oropharynx most likely suffers from …

A

Hand Foot and Mouth Disease

Coxsackievirus infection- herpangina is vesicles on posterior oropharynx

741
Q

A pt presenting with squeezing or burning sensation in the retrosternal chest that radiates to the back, neck, jaw or arms, resolves spontaneously or after consumption of antacids, usualy occurs postprandially, can awaken pt from sleep and worsens with emotional stress most likely suffers from …

A

Gastroesophageal reflux disease (GERD)

742
Q

What are the indications for parathyroidectomy in a pt with refractory secondary or tertiary hyperparathyroidism (elevated PTH and calcium level)? (7)

A
  1. calcium greater than 10.5 and not responding to therapy
  2. hyperphosphatemia not responding to therapy
  3. PTH level greater than 1000
  4. intractable bone pain
  5. intractable pruritis
  6. calciphylaxis episode
  7. soft tissue calcification
743
Q

What is the next best step in management of a pregnant pt who has exposure to a HSV infected partner (active lesions)?

A

perform type specific IgG antibody screening for HSV-1 and HSV-2

(if positive, give acyclovir starting at 36 weeks and c-section if active lesions)

744
Q

A male pt presenting with abdominal pain, inability to void urine, suprapubic tenderness, elevated creatinine and an enlarged smooth prostate most likely suffers from …

A

Obstructive, acute renal failure secondary to benign prostatic hypertrophy

745
Q

What is the next best step in management for a pt suffering from obstructive acute renal failure secondary to benign prostatic hypertrophy?

A

urethral catheterization with foley

use suprapubic catheterization if history of pelvic/ urethral trauma, urethral strictures

746
Q

A female pt presenting with depression/ anxiety/ irritability, bloating and breast tenderness that occurs monthly and resolves promptly with onset of menses each month most likely suffers from ..

A

Premenstrual Syndrome

(premenstrual dysphoric disorder- PMDD is severe form with predominance of anger and irritability)

(increased risk of mood and anxiety disorders)

747
Q

What is the best intiial treatment for premenstrual syndrome?

A

SSRIs (if one fails, can try another or use OCPS if dont want to be pregnant)

748
Q

What is the next best step for the treatment of symptomatic rectocele (low back pain, pelvic pressure and heaviness, constipation) in a pt who refuses surgery or is not a surgical candidate?

A

pessary with vaginal estrogen cream

749
Q

What is the next best step in management of a pregnant pt who is found to have active Hep B infection (positive Hep B surface antigen and Hep B core antigen)?

A

give mom Hep B vaccination and antiviral therapy; give baby Hep B vaccine and Hep B immunoglobulin within 12 hours of birth

750
Q

What is the most effective emergency oral contraceptive mechanism for a pt who had unprotected intercourse?

A

Ulipristal (anti-progresting pill effective if taken within 5 days of intercourse)

(copper IUD is most overall effective emergency contraceptive but avoid if pelvic infection/ cervicitis/ wilson’s disease)

751
Q

What is included in post-exposure prophylaxis for sexual assault victims? (5)

A
  1. cefitraxone (gonorrhea)
  2. azithromycin (chlamydia)
  3. metronidazole (trichomonas vaginalis)
  4. tenofovir-emtricitabine with raltegravir (HIV)
  5. hep B vaccine with/ without Hep B immunoglobulin
752
Q

What is the next best step in managemnt of a pt with a curvilinear calcification in the right hypochondrium at the location of the gallbladder?

A

elective cholecystectomy (for porcelain gallbladder that has high risk of gallbladder carcinoma)

753
Q

What is the next best step in management of a stroke patient after evaluation for intracranial bleeding with CT and bedside swallow study for possible aspiration?

A

subcutaneous low dose heparin or low molecular weight heparin

(for DVT prophylaxis that can lead to PE)

754
Q

A pregnant pt who develops mild and asymptomatic thrombocytopenia late in pregnancy and has not history of thrombocytopenia outside of pregnancy or evidence of fetal thrombocytopenia most likely suffers from ..

A

Gestational Thrombcyotpenia

resolves once child born

755
Q

What medication should be avoided in a with acute coronary syndrome/ myocardial infarction?

A

immediate release calcium channel blockers like immediate release nifedipine

(increases mortality via hypotension and increased sympathetic tone)

756
Q

What is the next best step in the management of a female pt with an incidental finding of a liver lesion that is less than 5 cm, well-demarcated, hyperattenuated area on liver and the pt is asymptomatic?

A

discontinue OCPs (associated with hepatic adenoma)

suspect malignancy if increases in size after discontinue OCP or elevated AFP level

757
Q

An infant presents with vomiting, hypotension, hyponatremia, hyperkalemia, hypoglycemia and ambiguous genitalia in girl/ precocious puberty in boy most likely suffers from …

A

21 hydroxylase deficiency type congenital adrenal hyperplasia

(dx: elevated 17-hydroxyprogesterone)

(have high testosterone, low aldosterone and cortisol)

758
Q

An infant presents with hypertension, hypernatremia, and ambiguous genitalia most likely suffers from …

A

11 beta hydroxylase deficiency type congenital adrenal hyperplasia

(dx: elevated 11-deoxycorticosterone and 11- deoxycortisol)

(have high testoerone, low aldosterone and cortisol)

759
Q

What is the best treatment for congenital adrenal hyperplasia (especially 21 hydroxylase deficiency type)?

A

chornic glucocorticoid and mineralocortcoid therapy

hydrocortisone and fludrocortisone

760
Q

What treatment used during chemotherapeutic treatment of cancer has a side effect of akathisia, dystonia, gait abnormality, bradykinesia, and increased muscle tone/ rigidity (similiar to Parkinsons)?

A

Metoclopromide

central and peripheral D2- receptor antagonist used for chemotherapy induced nausea and vomiting

761
Q

What is the next best step in management of a young women with premature ovarian failure with symptoms of hot flashes, vaginal dryness and low bone density?

A

conjugated equine estrogen with medroxyprogesterone acetate

(combination of estrogen and progesterone closely monitored; has increased risk of MI, DVT, stroke, breast cancer in older pts)

762
Q

What is the next best step in management of a pt with fecal incontinence and evidence of fecal impaction?

A

enemas followed by rectal suppositories to evacuate bowel

then caninstruct pt to increase fluid intake and dietary fiber and use stool softners as needed

763
Q

What is the next best step in management of a pregnant woman with severeal flesh-coloured hyperkerototic papules in the genital area at time of labor?

A

allow labor to continue without invention

condyloma acuminata is HPV infection; no contraindication to vaginal delivery

764
Q

A sickle cell pt who is experiencing acute onset of weakness, pallor, and lethargy in the setting of acute severe anemia superimposed on chronic anemia and absent reticulocytes most likely suffers from …

A

Aplastic Anemia

transient failure of erythropoiesis

765
Q

An infant born premature who has gross motor delay within first year of life, early hand preference, hypotoni that progresses to spasticity, hyperreflexia, persistent primitive reflexes, scissoring gait (spastic diplegia) and toe walking most likely suffers from …

A

Cerebral palsy

(due to prematurity, birth asphyxia, intrauterine infection)

(dx: brain MRI)

766
Q

A pt presents symptomatic withnon-enlarged thyroid gland, low TSH and elevated T4, and has low serum thyroglobulin and low radioiodine uptake most likley suffers from …

A

Exogenous thyroid use- hyperthryoidism

767
Q

A female presenting with homogenous vaginal discharge, vaginal pH greater than 4.5, amine/ fish odor of vaginal discharge with potassium hydroxide application, and presence of irregularyly bordered epithelium cells with outlines obscured by sheets of small bacteria most likely suffers from …

A

Bacterial Vaginosis

(tx: oral clindamycin or metronidazole to relieve symptoms)

(affects 1/3 of pregnant pts and is associated with preterm delivery, premature rupture of membranes, spontaneous abortion)

768
Q

What is the best treatment regimen for type 1 diabetics?

A

basal- bolus insulin regimen

long-acting and rapid acting in combination

769
Q

What is the next best step in management of an elderly pt who wears dentures and has developed erythematous painful oral lesions?

A

remove dentures for 1-2 weeks and re-evaluate

if lesions dont subside, biopsy for possible malignancy

770
Q

A pt with history of asthma/CF who is experiencing recurrent asthma exacerbations, fever, lethargy, cough with production of brown mucus plugs, hempotysis, fleeting infiltrates on CXR, central bronchiectasis on CT, eosinophilia (greater than 500) and IgE greater than 417 most likely suffers from …

A

Allergic bronchopulmonary aspergillosis (ABPA)

(tx: glucocorticoids and itraconazole)

(due to exaggerated IgE and IgG immune mediated hypersensitivity to Aspergillus)

771
Q

What is the best initial treatment for onychomycosis (fungal infection of toenails/ fingernails resulting in thick, brittle and discolored nails)?

A

oral terbinafine for 6 weeks if fingernails and 12 weeks if toenails

(oral itraconaole is alternative)

772
Q

A pt presenting with microcytic, hypochromic anemia (low MCV, low mean corpuscular hemoglobin), elevated ferritin (due to increased RBC turnover), target cells on smear and normal red cell distribution width most likely suffers from …

A

thalassemia minor

dx: hemoglobin electrophoresis

773
Q

What is the next best step in management of a pregnant pt with elevated alfa fetoprotein level (AFP)?

A

obstetric ultrasound (to get detailed anatomic survery)

774
Q

What are the non-permanent methods of contraception that have the lowest rate of pregnancy (are most effective)? (2)

A
  1. intrauterine devides (IUD)

2. progestin implants

775
Q

A pregnant pt in the first trimester of pregnancy who is found to have a crescent-shaped hypoechoic area adjacent to the gestational sac most likely suffers from ..

A

Subchorionic hematoma

re-evaluate with repeat ultrasound in 1 week

776
Q

What complication is most likely to occur in a pregnant pt with a subchorionic hematoma?

A

spontaneous abortion

777
Q

What are the indications for surgery (parathyroidectomy) in asymptomatic pts with primary hyperparathyroidism? (4)

A
  1. serum calcium greater than 1 mg/dl above upper limit of normal
  2. young age (less than 50)
  3. bone mineral density score of less than T -2.5
  4. reduced renal function (GFR less than 60 ml/min)

(also perform if pt is symptomatic)

778
Q

What is the next best step in management for a pt presenting with history of intimate exposure to HIV positive person and clinical symptoms of primary HIV infection (fatigue, sore throat, muscle aches, fever) but had negative ELISA and Western Blot testing results?

A

HIV RNA PCR assay or test for p24 antigen

779
Q

What is the next best step in management of a pt with history of intimate exposure to HIV positive person?

A

HIV antibody testing at initial visit, 6 weeks, 12 weeks, and 24 weeks

(using ELISA confirmed by Western Blot)

780
Q

How often should a newly diagnosed HIV positive pt have their immune status monitored whether on HAART therapy or not?

A

evaluate CD4 count and HIV viral load every 3-4 months

781
Q

An infant who has difficulty passing meconium/ only passed small amount of thick meconium during initial 24-48 hours, has bilious vomiting, abdominal distention and on abdominal X-ray has dilated, gas filled loops of small bowel, absent air-fluid levels and a ground glass mass withing the right side of the abdomen in the setting of a family history of recurrent respiratory infections most likely suffers from ..

A

Cystic Fibrosis

782
Q

What is the next best step in management of a pt who is found to have elevated, corrected serum calcium levels confirmed by repeat testing?

A

measure parathyroid hormone (PTH) levels

783
Q

A pt presenting with severe hypercalcemia (calcium greater than 13), low PTH, and a long standing history of smoking most likely suffers from …

A

PTH-related peptide production by malignancy (small cell carcinoma)

784
Q

What is the next best step in mangement of a pt with hypercalcemia and suppresed/ low PTH level?

A

measure PTHrP, 25(OH) vitamin D, and 1,25(OH) vitamin D levels

(if PTHrP elevated then solid tumor malignancy)
(if 1,25(OH) vitamin D elevated, then perform chest x-ray to assess for lymphoma and sarcoidosis)
(if 25 (OH) vitamin D elevated, then vitamin D toxicity)
(if none elevated then, hyperthyroidism/ multiple myeloma/ adrenal tumor/ acromegaly/ vitamin A toxicity/ immobilization/ milk-alkali syndrome)

785
Q

A pt presenting with low back pain that is brief, shooting and radiates along nerve distribution, provoked by forward bending/ coughing/ straining and dimishes with laying down most likely suffers from …

A

Nerve Root Irritation (radiculopathy)

dx: straight leg raise

786
Q

What is the next best step in the management of a pt with acute decompensated heart failure (acute dyspnea, orthopnea, paroxysmal nocturnal dyspnea, accessory muscle use, diffuse crackles, S3, JVD, peripheral edema) in order to improve symptoms?

A

reduce cardiac preload with diuretics (furosemide)

(consider IV vasodilators like nitroglycerin if normal or elevated BP)
(use IV vasopressor like norepinephrine if hypotensive/ shock)

(order pulse oximetry, CXR, and EKG)

787
Q

What is the routine guidelines for HPV vaccination?

A

vaccinate females between 9-26 years old (usually start at 9-11 years old) with 3 doses of vaccine

(bivalent- time 0, 1 month, and 6 month)
(quadrivalent or 9-valent- time 0, 2 months and 6 months)
(avoid in pregnant pts)

788
Q

What is the next best step in management of a health care worker who experiences occupational exposure (needlestick) while caring for a pt who refuses HIV testing and HIV status is unknown?

A

HIV post-exposure prophylaxis with 2-3 antiretroviral medications

(immediate step in cases of pt refusing HIV testing, testing results pending, HIV status unknown)

789
Q

What preparation needs to be done prior to the lactose breath hydrogen test for lactose intolerance?

A

pt should fast for 8 hours before test

(during test given lactose-containing beverage and then breath into bag at set intervals over 2-3 hours and hydrogen level measure by gas chromatography; greater than 20 ppm is positive)

790
Q

What are two common treatments that can cause an unexplained increase in TSH in a pt with hypothyroidism treated with stable dose of levothyroxine (disrupt absorption of levothyroxine)?

A
  1. calcium
  2. iron

(instruct pt to take levothyroxine on empty stomach separate from other medications)

791
Q

What is the next best step in management of a hyperthyroid pt with symptomatic tachycardia?

A

beta blockers (propanolol)

followed by PTU or methimazole for long term care

792
Q

What is the next best step in management of delirium associated agitation and physical aggression in elderly patients after attempting reassurance, orienting to environment, and constant supervision?

A

IV or IM low dose antipsychotics (haloperidol/ risperidone)

793
Q

What is the next best step in management of a pt with gonococcal urethritis?

A

single dose of ceftriaxone 250 mg IM with C. trachomatis treatment (oral doxycyline 100 mg BID for 7 days or oral azithromycin 1 gram single dose)

794
Q

What is the next best step in management of a pt with gastroenteritis due to Salmonella enteritidis infection?

A

supportive therapy and observation if immunocompetent and older than 12 months of age

795
Q

What is the next best step in management of a pt presenting with fever, malaise, headache and localized pain that progresses into painful vesicular rash in the distribution of specific nerve roots?

A

Oral acyclovir for 7-10 days (for Shingles/ herpes zoster) and placed on contact isolation until skin lesion are dry and crusted over

(no diagnostic test for shingles)
(low recurrence risk but should have vaccine if 60 or older)

796
Q

What is the next best step in management of a pt with renal dysfunction who presents with bleeding in the setting of normal platelet count but prolonged bleeding time?

A

IV desmopressin (if symptomatic or prior to surgeries)

increases release of factor 8 and vwF from endothelium

797
Q

A infant (of African american/ mediterranean/ asian descent) who develops unconjugated hyprbilirubinemi, jaudince, and scleral icterus withing the first 24 hours of life and has a negative Coombs test most likely suffers from …

A

G6PD deficiency

physiologic jaundice does not appear until 24 hours after birth

798
Q

What is an important long term complication of donor nephrectomy (donating a kidney)?

A

increased risk of gestational complications in female donors of childbearing age

799
Q

A pt presenting with painless blistering on the dorsal surfaces of the hands after being in the sun, facial hyperpigmentation, and facial hypertrichosis (facial hair growth) most likely suffers from …

A

Porphyria Cutanea tarda (due to uroporphyrinogen decarboxylase deficiency)

(dx: urinary uroporphyrins)
(tx: phlebotomy or hydroxychloroquine)

800
Q

What is the best initial treatment for a post-partum woman presenting with fever, breast pain, focal inflammation and a history of poor latching during breastfeeding?

A

Oral dicloxacillin or cephalexin, observe feeding for technique and position, and more frequent breastfeeding/ pumping

(treat lactional mastitis due to S. aureus)

801
Q

What is the next best step in management for a post-partum breastfeeding woman who is being treated for lactational mastitis but returns after start of antibiotic treatment with new palpable induration and/ or fluctuance?

A

Ultrasound to assess for abscess formation

(antibiotics can take 2-3 days to treat normal lactational mastitis but if abscess present, then need to use ultrasound guided needle aspiration to drain)

802
Q

A child aged 6 months to 6 years presenting with generalized tonic-clonic jerking or atony lasting less than 15 minutes in the setting of a fever (greater than 38 C) without prior history of afebrile seizures/ CNS infection/ acute systemic metabolic cause most likely suffers from …

A

Febrile Seizure

803
Q

What is the prognosis for a child who has a febrile seizure?

A

have an increased risk of developing another febrile seizure (30%) and less than 5% risk of epilepsy; have normal development/ intelligence

804
Q

What is the next best step in management of pt on SSRI treatment (for depression) who develops sexual dysfunction (decreased libido, anorgasmia, delayed ejaculatio) that is distressing to the patient?

A

switch to bupropion or mirtazapine

805
Q

A pt presenting with progressive symmetrical proximal muscle weakness, grossly elevated CPK levels and elevated ferritin most likely suffers from …

A

Inflammtory Muscular Disorder
(dermatomyositis or polymyositis)

(tx: systemic high dose glucocorticoids- prednisone 1 mg/kg)

806
Q

A pt on mechanical ventilation who is being given continuous glucose infusions and is unable to be weaned off the ventilator along with signs of respiratory muscle weakness and significant thinning of the diaphragm most likely suffers from …

A

Hypophosphatemia

impairs ATP generation needed by muscle

807
Q

A pt with an incidental finding of a small (diameter less than 10 mm; thickness less than 3 mm) densely pigmented lesion with irregular boarders on funduscopic examination most likely suffers from …

A

Ocular Melanoma

(dx: ultrasound is most sensitive; MRI used to detect extrascleral extension for staging and treatment)
(if pt asymptomatic with small lesion, repeat exam in 3 months)

808
Q

What is the next best step in management of a pt with a large (diameter greater than or equal to 10; thickness greater thn or equal to 3) ocular melanomas or pts with symptomatic ocular melanomas (eye pain, visual distubarnces)?

A

radiotherapy (brachytherapy or external beam radiotherapy)

use enucleation if very large tumors or tumors with extrascleral extension

809
Q

What are the three prophylaxis regimens that can be used for close contacts (household members, roomates, intimate contacts, child care workers, airline travelers seated next to affected person for more than 8 hours, persons directly exposed to pt’s respiratory/ oral secretions via kissing/ mouth-to mouth resuscitation/ endotracheal intubation) of individuals with neisseria meningitidis infection?

A
  1. rifampin 600 mg BID for 2 days (avoid if person on OCPs)
  2. oral ciprofloxacin 500 mg single oral dose
  3. IM ceftriaxone 250 mg single dose
810
Q

What is the next best step in management of a pt with an asthma exacerbation that is mild to moderate (FEV1 or PEF greater than or equal to 40%)?

A
  1. oxygen until 90% or greater saturation
  2. inhaled short acting beta agonist (albuterol) 3 doses/ hr
  3. systemic corticosteroids if no response to albuterol/ if recent oral corticosteroid use
811
Q

What is the next best step in management of a pt with an asthma exacerbation that is moderate to severe (FEV1 or PEF less than 40%)?

A
  1. oxygen until 90% or greater saturation
  2. oral systemic corticosteroids
  3. high dose inhaled short acting beta agonist (albuterol) with ipratropium nebulizer every 20 minutes/ continously for 1 hour
812
Q

What is the next best step in management of a pt with an asthma exacerbation with impending or actual respiratory arrest?

A
  1. inhaled short acting beta agonist (albuterol) and ipratropium nebulizer
  2. IV corticosteroids and magnesium sulfate
  3. possibly SQ/ IV epinephrine or terbutaline
  4. mechanical ventilation with intubation and 100% oxygen

(admit to ICU)

813
Q

What treatment used in patients with severe congenital anemia (such as thalassemia major) can result in significant iron overload and organ damage?

A

Hypertransfusion regimen

814
Q

What is the next best step in management for a pt with advance colorectal adenomas (3-10 adenomas, any adenoma that is greater than 1 cm or adenoma with high grade dysplasia or villous features) on screening colonscopy?

A

repeat colonscopy in 3 years

815
Q

What is the next best step in management for a pt with 1 or 2 small (less than 1 cm) tubular adenomas on screening colonscopy?

A

repeat colonoscopy in 5 years

816
Q

What is the next best step in management for pt with large (greater than 2 cm) sessile polyp removed by piecemeal exicision? What is the next best step in management for pt with polyp with adenocarcinoma (minimal invasion and greater than 2 mm margins)?

A

large sessile: repeat colonoscopy in 2-6 months

adenocarcinoma: repeat colonoscopy in 2-3 months

817
Q

A pt presenting with rapid onset severe knee pain with popping sensation at the time of injury and associated with significant swelling and joint instability after pivoting on lower extremity with foot planted or rapid change in direction most likely suffers from …

A

Anterior Cruciate Ligament (ACL) injury

dx: anterior drawer test, Lachman test, MRI
(tx: RICE- rest, ice, compression, elevation; possible surgery)

818
Q

What is the next best step in management for a type 2 diabetic presenting with hypoglycemia secondary to accidental/ intentional overdose with sulfonylurea (glyburide) after starting dextrose infusion?

A

add octreotide

decreases insulin secretion that occurs with transient hyperglycemia from dextrose infusion

819
Q

What is the most common micro-organism associated with corneal foreign bodies?

A

Coagulase negative Staphylococcus

820
Q

What is the best initial treatment for absence seizures (kid with staring spells followed by confusion that can result in poor school performance)?

A

Ethosuximide

valproate is secon line

821
Q

What type of shock results in elevated right atrial pressure (greater than 4 mmHg), increased pulmonary capillary wedge pressure (greater than 9 mmHg), decreased cardiac index ( less than 2.8 L/min/m2), increased systemic vascular resistance (greater than 1150 dynes*sec/ cm5) and decreased mixed venous saturation (less than 60%)?

A

Cardiogenic Shock

822
Q

What type of shock results in decreased right atrial pressure (less than 4 mmHg), decreased pulmonary capillary wedge pressure (less than 9 mmHg), decreased cardiac index (less than 2.8 L/min/m2), increased systemic vascular resistantce (less than 1150 dyne* sec/cm5) and decreased mixed venous oxygen saturation (less than 60%)?

A

Hypovolemic Shock

823
Q

What type of shock results in normal right atrial pressure (4 mmHg), normal pulmonary capillary wedge pressure (9 mmHg), increased cardiac index (more than 4.2 L/min/m2), decreased systemic vascular resistance (less than 1150 dynes*sec/ cm5) and increased mixed venous oxygen saturation (greater than 80%)?

A

Septic Shock

824
Q

What is the next best step in management following thyroidectomy and radioactive iodine therapy for thyroid cancer (papillary and follicular)?

A

levothyroxine
small, low risk: with target TSH 0.1-0.5 for 6-12 months, the low normal
intermediate risk: with target TSH 0.1-0.5
large, aggressive: with target TSH less than 0.1 for several years

(suppress TSH to decreases recurrence risk but avoid increase risk of bone loss and a-fib with high dose levothyroxine)

825
Q

A child presenting with sleep walking (with blank, staring face; unresponsive to attempts to awaken) or sleep terrors (abrupt arousal from sleep with panicked scream/ terror/ autonomic arousal, unresponsive to comfort) with little or no dream recall and amnesia regarding the episodes most likely suffers from …

A

Non-REM sleep arousal disorder

(prognosis: self-limiting; spontaneous resolution over years)
(can use benzos if episodes frequent, persistent and distressing)

826
Q

What is the next best step in management for an incidentally found cystic lesion in the pancreas on abdominal imaging?

A

endoscopic ultrasound and aspiration

to determine if malignant

827
Q

What are the indications for group B strep prophylaxis with penicillin during labor and delivery? (4)

A
  1. prior infant with early onset GBS disease
  2. GBS bacteruria/ urinary tract infection during pregnancy
  3. GBS positive within 5 weeks of labor (rectovaginal culture at 35-37 weeks gestation)
  4. unknown GBS status with labor at less than 37 weeks gestation/ intrapartum fever/ rupture of amniotic membranes for 18 hours or more

(adequate prophylaxis if given 4 or more hours before delivery)

828
Q

What is the next best step in management for a pt with back pain secondary to metastatic, advanced hormone refractory cancer (i.e. prostate cancer)?

A

focal external beam radiation

829
Q

A woman of childbearing age presenting with sudden onset severe pelvic pain, unilateral and tender adnexal mass, nausea, vomiting and fever most likely suffers from ..

A

Ovarian/ Adnexal torsion

(dx: pelvic ultrasound with color doppler)
(tx: laparoscopy with detorsion; salpingo-oophrectomy for necrosis or malignancy)

(make sure to rule out pregnancy)

830
Q

What is the next best step in management for halted labor after the initial baby is born vaginally in a twin gestation as long as amniotic sac intact, fetal heart rate is reassuring and baby is cephalic?

A

IV oxytocin

831
Q

What is the next beststep in management for a pt with high risk sexual encouter with an HIV positive partner?

A

Tenofovir, emtricitabine and raltegravir for 4 weeks within 72 hours of exposure

832
Q

What is the initial hematologic measure to respond to iron supplementation in iron deficiency anemia?

A

increase in reticulocyte coutn (after 1-2 weeks)

hemoglobin and hematocrit increases after 3-4 weeks

833
Q

What is the best initial treatment for sarcoidosis (symptomatic pulmonary disease with fever, fatigue, and hypercalcemia)?

A

daily oral corticosteroids

834
Q

A pt presenting with fever, lethargy, myalgias, and headache followed by development of petechial rash that starts on wrists and ankles and then spreads to soles, palms, and central body after outdoor activity during the spring/ early summer most likely suffers from …

A

Rocky Mountain Spotted Fever

(dx: clinically)
(tx: doxycycline)

(transmitted by black tick)

835
Q

A pt presenting with fatigue, exertional dyspnea, systolic murmur heard best at left sternal border that increases with Valsalva (decreased preload), asymmetric septal hypertrophy, systolic anterior motion of mitral leaflets and increased left ventricular outflow tract gradient with exercise most likely suffers from …

A

Hypertrophic Obstructive Cardiomyopathy

836
Q

What is the best initial treatment for hypertrophic obstructive cardiomyopathy?

A

Beta blockers

can add verapmil or disopyramide

837
Q

What is an important predictor of work outcome for patients with occupational back pain?

A

Recovery expectation

educate pt on natural history and prognosis to positively influence expectation

838
Q

A pt presenting with small, rough erythematous and kerototic papules that are easier to feel than to see and located on photdamaged skin (dyspigmentation, wrinkling, thinning, telangiectasia) most likely suffers from …

A

Actinic Keratosis

increase risk of squamous cell carcinoma

839
Q

What is the best treatment for actinic keratosis?

A

single lesion: cryotherpy or surgical excision

multiple lesions: field treatment with 5 fluorouracil cream

840
Q

What is the most appropriate strategy to decrease the risk of perinatal transmission of herpes virus if pt has active lesions during pregnancy?

A

antiviral agents from 36 weeks to delivery with C-section if symptomatic at term

841
Q

What intervention is used to reduced the risk of contrast induced acute kidney injury?

A

IV volume expansion (IV saline before and after scan to prevent renal hypoperfusion) and avoid nephrotoxic agents (NSAIDs)

842
Q

A pt with palpitations and an EKG with absence of P waves, irregularly irregular rhythm, with varying RR intervals and narrow QRS complexes most likely suffers from …

A

Atrial Fibrillation (with rapid ventricular response)

843
Q

What is the best initial treatment for atrial fibrillation (with rapid ventricular response)?

A

Beta blockers (avoid if have worsening CHF, hypotension, bronchospasm) or non-dihydropyridine calcium channel blockers

844
Q

What precautions are used to prevent spread of C. difficile colitis in hospitals?

A

contact isolation with soap and water hygiene

845
Q

A pt presenting with local burning pain out of proportion to injury, temperature change, edema, and abnormal skin coloring (mottled, blue discoloration) of extremity after injury that can then progress to skin thickening, muscle wasting, limited range of motion, and bone demineralization on X-ray most likely suffers from….

A

Complex Regional Pain Syndrome

dx: increase resting sweat output or MRI showing changes
(tx: sympathetic nerve block or IV regional anesthesia)

846
Q

What is the most common finding associated with massive pulmonary embolism with EKG findings of acute pulmonary hypertension and right ventricular strain (RBBB, atrial arrhythmias, inferior Q waves)?

A

Tricuspid regurgitation (and dilation of tricuspid annulus)

847
Q

A pt presenting with acute onset (less than 6 weeks) symmetrical polyarticular joint pain and swelling of small joints, with slightly elevated rheumatoid factor after a recent upper respiratory infection most likely suffers from …

A

Viral Arthritis (likely due to parvovirus)

prognosis: self-limited and no long term sequelae

848
Q

What does the area under the curve of a receiver-operating characteristic curve signify?

A

diagnostic accuracy

more AUC means better performance

849
Q

A woman who develops heart failure during last month of pregnancy or withing 5 months after delivery and has left ventricular systolic dysfunction with left ventricular ejection fraction less than 45% most likely suffers from ….

A

Peripartum Cardiomyopathy

evaluate with ECHO prior to subsequent pregnancies to assess for risk of complications

850
Q

What is the next best step in management of an elderly pt with a hip fracture who was stable and ambulatory prior to the injury?

A

surgerical correction within 48 hours

851
Q

What is the most important single risk factor for osteoporesis and osteoporetic bone fracture?

A

Age

852
Q

What are male patients who had cryptorchidism (undescended testis) still at risk for even after orchipexy?

A

malignancy

853
Q

A pt presenting with pitting of the nails, sharply demarcated erythematous plaques with thick silvery scales on extensor surfaces of elbows/ knees/ etc, and arthritis in joints characterized by early morning stiffness relieved by activity mainly affecting the DIP joints and axial skeleton most likely suffers from …

A

Psoriasis (with psoriatic arthritis)

(mild tx: high potency topical steroids, vitamin D derivatives)
(severe tx: phototherapy, systemic therapy- methotrexate)
(facial and intertriginous tx: topical tacrolimus, low potency steroids)
(guttate tx- small salmon pink spots on skin: observation, phototherpy)

854
Q

What is the best initial treatment for a pt with psoriasis with joint involvement or extensive disease?

A

Methotrexate

if mild disease: topical glucocorticoids and vitamin D derivatives like calcipotriene

855
Q

A pt presenting with arthalgia,intense fatigue, palpable purpura (red, non-blanching, nontender lesions), lymphadenopathy, hepatosplenomegaly, peripheral neuropathy, decreased serum complement levels and renal disease (hematuria, proteinuria, elevated creatinine) in the setting of Hep C infection most likely suffers from …

A

Cryoglobulinemia

dx: serum cyroglobulin level
(initial short-term tx with immunosuppressive therapy)

856
Q

What is the best long term treatment for Cryoglobulinemia?

A

anti-viral therapy (alpha interferon and ribavirin for Hep C)

857
Q

What is the best initial treatment for mild gastroesophageal reflux in an infant?

A

thicken formula with cereal

858
Q

What is the best treatment for chlamydia infections in a pregnant pt?

A
  1. erythromycin base 500 mg PO QID for 7 days

2. amoxicillin 500 mg PO TID for 7 days

859
Q

What is the best initial step in management of a pt with a thyroid nodule?

A

obtain TSH and ultrasound

(if no cancer risk or suspicious US finding with normal/ elevated TSH or if there are cancer risk factors or suspicious US finding, then FNA)
(if no cancer risk or suspicious US findings with low TSH, then iodine 123 scintigraphy/ radionuclide thyroid scan)

(cancer risk factors: family hx, past radiation exposure, cervical lymphadenopathy, compressive symptoms- hoarseness/ difficulty swallowing)

860
Q

What is the next best step in management of a pt with a thyroid nodule found to be thyroid cancer on fine needle biopsy?

A

ultrasound of neck and cervical lymph nodes to stage cancer

(to best determine type of surgery)
(if papillary cancer less than 1 cm, lobectomy)
(if papillary cancer 1 cm or greater, then total thyroidectomy)

861
Q

What is the best approach to diagnosing sicca syndrome (sjogrens) in a pt with clinical symptoms suggestive of the disorder?

A

Schirmer test (secretory deficiency) with anti-Ro/ SSA and anti-La/ SSB antibodies (as well as RF and ANA)

862
Q

What are the exceptions to laws protecting confidentiality of adolescents with regards to pregnancy, contraception, STDs, substance use, and psychiatric illness? (4)

A
  1. harm to self or other
  2. physcial or sexual abuse
  3. neglect
  4. violent injuries (gun shot or stab wound)
863
Q

What is the next best step in management of a pt on warfarin with an INR less than 5 and none/minimal bleeding?

A

hold warfarin for 1-2 days and monitor INR; or decrease wararin dose

864
Q

What is the next best step in management of a pt on warfarin with an INR between 5-9 and none/ minimal bleeding?

A

hold warfarin and resume when INR therapeutic

administer low dose (1-2.5 mg) oral vitamin K if increased risk of bleeding

865
Q

What is the next best step in management of a pt on warfarin with an INR greater than 9 with non/minimal bleeding?

A

hold warfarin and administer high dose (2.5-5mg) oral viatmin K

866
Q

What is the best initial treatment for hyponatremia in a pt with congestive heart failure (CHF)?

A

water intake restriction

(hyponatremia in CHF due to dilutional hyponatermia from increased ADH and water intake)

(use tolvaptan- vasopressin receptor antagonist if symptomatic hyponatremia or severe hyponatremia (Na less than 120))

867
Q

What is the best treatment for negative symptoms of Schizophrenia (alogia, flat affect, amotivation, social withdrawal)?

A

psychosocial intervention with social skills training

868
Q

A pt presenting with an oblong shaped, firm and deeply seated mass thay may or may not be painful located in the trunk/extremity/ intraabdominal bowel/ messentary/ abdominal wall most likely suffers from …

A

Desmoid Tumor

(high recurrence, benign agressive tumor)

(dx: CT or MRI)
(tx: surgery if symptomatic/ risk to adjacent structures/ cosmetic/ recurrent)

869
Q

What are the criteria for cardiac resynchronization therapy with biventricular pacing device in a pt with heart failure? (3)

A
  1. severe left ventricular systolic dysfunction with ejection fraction less than 35%
  2. class 2,3 or 4 heart failure symptoms
  3. left bundle branch block with QRS greater than 150msec
870
Q

A pt who presents with fever, chills, hemoglobinuria, flank pain and discomfort at infusion site within an hour of a blood transfusion most likely suffers from …

A

Acute Hemolytic Transfusion Reaction

(due to ABO incompatibility)

(tx: stop transfusion and hydrate with IV normal saline)
(dx: direct Coombs test; pink plasma)

871
Q

What is the best initial screening test for evaluating thyroid function during pregnancy?

A

TSH

(first trimester: TSH 0.1-2.5)
(second trimester: TSH 0.2-3.0)
(third trimester: TSH 0.3-3.0)

(levels of free T4 and T3 unreliable during pregnancy due to high thyroid binding-globuin levels and lower albumin levels in pregnancy; so adjust levels to 1.5x normal range or measure total T4)

872
Q

A pt presenting with nausea, headache and altered mental status after being rescued from a burning building most likely suffers from …

A

Carbon Monoxide Poisoning

dx: CO-oximetry

873
Q

What is the next best step in management of a pt with liver cirrhosis along with distended abdomen with shifting dullness, malaise, leukocytosis, fever and diffuse abdominal tenderness?

A

Abdominal paracentesis

(to assess for spontaneous bacterial peritonitis- PMN leukocyte count of greater than 250 with positive culture or gram stain)

874
Q

What are common signs of phenytoin toxicity? (5)

A
  1. nystagmus
  2. visual disturbances (diplopia, blurred vision)
  3. ataxia
  4. slurred speech
  5. decreased mentation
875
Q

A pt presenting wtih signs of right heart failure (edema, JVD, hepatic congestion, hepatomegaly), Kussmaul sign (increase in JVD with inspiration), mid-diastolic knck, calcification of cardiac borders on imaging, atrial enlargment and pulsus paradoxus (decrease in systolic BP with inspiration) most likely suffers from …

A

Constrictive Pericarditis

common in pts with CABG, radiation therapy and TB

876
Q

What is the next best step in management of a pt who meets criteria for neurofibromatosis type 1 (two or more of following: first degree relative with NF1, more than 6 cafe-au-lait spots, presence of more than 2 neurofibromas/ lisch nodules/ optic gliomas/ bone dysplasia/ axillary freckling)?

A

immediate ophthalmology referral

for early detection of optic nerve gliomas and Lisch nodules- iris hamartoma

877
Q

What is the best initial treatment for uterine prolapse (sensation of pressure or heaviness in pelvic area relieved by lying down and aggravated by prolonged standing/exertion, low back pain, dyspareunia, vissible mass at introitus in multiparous postmenopausal women)?

A

surgical correction

use pessary if not surgical candidiate or refuse surgery

878
Q

What is the most important modifiable risk factor for stroke?

A

Hypertension

879
Q

A pt presenting with a single/ multiple hyprkeratotic papules on the sole of the foot that consist of thrombosed capillaries (as is seen on scrapings of the lesions)most likely suffers from …

A

Plantar Warts

due to HPV via skin to skin contact; in young adults of pt who is a meat/ poulty/ fish handler

880
Q

What is the best initial treatment for plantar warts?

A

topical salicylic acid (takes to 2-3 weeks to begin to resolve; continue treatment 1-2 weeks after resolution)

(liquid nitrogen is alternative)

881
Q

What is the best diagnostic test for diagnosing exercise- induced bronchoconstriction (shortness of breath and chest tightness 10-15 minutes after starting exercise and lasting about 30 minutes after exercise complete)?

A

Bronchoprovovation test with exercising or inhalation of dry, cold air

(fall in FEV1 greater than 10% is positive and greater than 15% is diagnostic)

882
Q

A pt presenting with dyspnea, persistent cough, facial fullness and neck pain progressing into hoarseness, dysphagia, chest pain and syncope with associated findings of edema and erythema of the neck and dilated veins of the arm and neckespecially in pt with high risk of malignancy most likely suffers from …

A

Superior Vena Cava Syndrome

dx: contrast CT of chest and neck

883
Q

What is the best step in management of a pt with a cat or dog bite? (4)

A
  1. local wound care
  2. rabies prophylaxis (if animal status unknown)
  3. tetanus prophylaxis (tetanus toxoid and IG if no recent tetanus vaccination within 5 or 10 years depending on wound dirtiness)
  4. amoxicillin/ clavulanate
884
Q

What is the next best step in management for a pt with syphilis who has a penicillin allergy?

A

penicillin desensitization

885
Q

What is the next best step in a pt suffering from benzodiazepine withdrawal (restlessness, tremors, autonomic instability) while hospitalized?

A

IV benzodiazepines (lorazepam, diazepam) with gradual tapering

886
Q

What is the best initial treatmnet for heat stroke (hyperthermia, tachycardia, flushed non sweating skin, headache, altered mental status)?

A

augmentation of evaporative cooling

naked pt sprayed with water mist or covered with wet sheets while fans circulate air

887
Q

What conditions are at an increase risk for women on hormone replacement therapy (with combine estrogen-progestin therapy)? (4)

A
  1. breast cancer
  2. venous thromboembolism (DVTs and PEs)
  3. non fatal myocardial infarction
  4. dementia

(benefits lipids, osteoporesis, and menopausal symptoms)

888
Q

A pt presenting with knee pain and joint effusion after injury with associated tendereness along medial joint line, pain occurs with external rotation of right tibia and moderate laxity with valgus (abducted) stress on knee most likely suffers form ..

A

Medial Collateral Ligament Injury

due to valgus/ abductor stresson knee

889
Q

An infant with an adducted and internally rotated arm with forearm pronation and flexed wrist and asymmetric Moro reflex most likely suffers from …

A

Erb’s Palsy

(injury to brachial plexus- upper roots consisting of C5, C6, C7)

(progrnosis: full or near full recovery by age 1)

890
Q

A pt presenting with lower-extremity claudication (intermittent butt/ thigh/leg pain with exertion), absent/ diminished femoral pulses and erectile dysfunction most likely suffers from …

A

Leriche syndrome (due to peripheral artery disease)

dx: ankle-brachial index less than 0.9
(tx: phosphodiesterase-5 (PDE-5) inhibitors- “afils”)

891
Q

A multipareous women with history of prior pelvic surgery/ chronic constipation or straining presenting with abdominal discomfort, straining to defecate/ fecal incontinence with mucus, defecation requiring digital maneuvers as disease progresses, and thick protruding rectal mucosal mass with bluish discoloration thru anus with concentric rings of rectum most likely suffers from …

A

Rectal Prolapse

(tx surgically if full thickness prolapse, sensation of prolapse or prolapse with fecal incontinence/ constipation)
(tx medically with fiber & fluid intake and pelvic floor muscle exercises, if non full thickness)

892
Q

A pt with a loud, harsh holosystolic murmu heard best in the 4th left intercostal space near sternal border with an associated palpable thrill most likely suffers from …

A

Ventricular Septal Defect

893
Q

A pt with back pain, low phosphorous level, high alkaline phosphatase, and elevated PTH in the setting of bariatric surgery/ malabsorption disorder most likely suffers from …

A

Vitamin D Deficiency

dx: 25- OH vitamin D level less than 20 ng/mL
(tx: vitamin D3/ cholecalciferol to maintain vitamin D level of 30-50)

894
Q

A pt presenting with malodorous frothy yellow-greeen vaginal discharge, vulvar pruritus and dyspareunia most likely suffers from …

A

Trichomonas vaginalis

(tx: metronidazole 2 grams PO for 1 dose)

(discontinue breastfeeding for 12-24 hours b/c enters breast milk)

895
Q

What is the major problem that leads to difficulties finding cross-matched blood in patients with history of multiple transfusions (i.e. sickle cell anemia or myelodysplasia patients)?

A

alloantibodies formation

especially to E, L, and K

896
Q

What are the five steps of root cause analysis (quality improvement tool used to identify what, how, and why an undesirable outcome occured)?

A
  1. collect data
  2. create causal factor flow chart
  3. identify root causes
  4. generate recommendations and implementation
  5. measure success of changes implemented
897
Q

A pt presenting with double vision and ptosis that worsens at the end of the day along with dysphagia. dysarthria, and proximal muscle weakness that worsen throughout the day and improves with restmost likely suffers from …

A

Myasthenia Gravis

a neuromuscular junction disease due to autoantibodies against ACh receptor

898
Q

What is a better measure of central tendency in a set of data that is strongly skewed distribution?

A

median

899
Q

What is the best next step in management of a severely malnourished child (lethargic, hypoactive, severe wasting, symmetrical lower extremity edema) if the child is vomiting, has impaired consciousness or painful oral ulcers?

A

insert dobhoff tube and rehydrate/ feed

(placed beyond pylorus to reduce aspiration)

(use oral rehydration if possible)

900
Q

A young, female healthcare professional (nurse) who presents with repeated bacteremia and sepsis with multiple different organisms most likely suffers from…

A

Factitious Disorder

(pt feigns illness to be pt and to assume sick role)
(pt is injecting exogenous material parenterally or ingesting drugs)

901
Q

A young female presenting with occasional urine leakage aggravated by coughing and straining in the setting of having genitourinary surgery/ multiple pregnancies and leakage occurs when pt is asked to relax and give single vigorous cough most likely suffers from ….

A

Stress Urinary Incontinence

initial tx: pelvic floor exercises
(last resort tx: surgical correcttion)

902
Q

What anti-hypertensive medication is associated with photosensitivity reaction (erythematous rash on exposed parts of body)?

A

Thiazides diuretics

hydrochlorothiazide

903
Q

What is the next best step in management of a pt on pentamidine therapy who develops generalized seizure/ CNS symptoms?

A

finger stick blood test

pentamidine associated with hypo/ hyper-glycemia, hyper/ hypo-kalemia, hypocalcemia

904
Q

What is the criteria for delayed puberty in a boy?

A
  1. no testicular enlargement by 14 years old
  2. testicles are 2.5 cm or smaller in diameter
  3. delay in development for 5 years or more from onset of genitalia enlargement
905
Q

What is the next best step in management of a pt with delay puberty?

A

X-ray imaging to assess bone age

906
Q

What is the next best step in management of a pt with subclinical thyrotoxicosis (suppressed TSH and normal thyroid hormone levels)?

A

recheck thyroid function test in 6-8 weeks

if no symptoms, normal heart rhythm, normal bone denisty

907
Q

What is the best initial test for suspected pneumothorax?

A

upright chest X-ray with posteroanterior view

(gas separating white visceral pleural line from parietal pleural line with no vascular markings beyond visceral pleural line)

908
Q

What are the indications for implantable cardioverter-defibrillator placement in hypertrophic obstructive cardiomyopathy as primary prevention? (50

A
  1. family hx of sudden cardiac death
  2. syncope (recurrent or associated with exertion)
  3. nonsustained VT on Holter monitoring
  4. hypotensive BP response to exercise
  5. extreme LV hypertophy (greater than 3 cm)

(used as secondary prevention if prior hx of cardiac arrest or sustained, spontaneous ventricular tachycardia)

909
Q

What is the next best step in management of a male pt presenting with fever, dysuria, lower abdominal discomfort, inability to void, palpable bladder, and anterior rectal wall tenderness?

A

suprapubic catheterization

(relieve pt’s urinary obstruction that is worsen acute prostatitis)

(avoid foley catheter and prostatic massage as can result in bacteremia)

910
Q

What is the next best step in management of a cirrhotic pt that is hospitalized for variceal bleeding to prevent assocaited complications?

A

fluoroquinolone (cipro/o/nor-floxacin) for 7-days

prevent infection such as spontaneous bacterial peritonitis

911
Q

What three type of patients should be treated for asymptomatic bacteriuria (bacteria greater than 10^5 colony forming units in pt without signs or symptoms of UTI)?

A
  1. pregnant pt
  2. urologic intervention pt
  3. hip arthroplasty pt

(not treating others prevents drug resistance)

912
Q

What is the most common complication of cat-scratch disease due to Bartonella henselae?

A

suppration of lymph nodes

other complications are visual loss due to neuroretinitis, encephalopathy, fever, hepatosplenomegaly

913
Q

What is the next best step in management of a pubertal male who develops gynecomastia?

A

reassure and follow-up

normal due to excess production of estrogens from testes during puberty; regresses in 18-24 months

914
Q

A middle aged woman presenting with elevated alkaline phosphatase levels and intense pruritus (excoriations from scratching) and may also have jaundice/ steatorrhea/ fatigue/ hyperlipidemia with xantomas/bone disease/ hyperpigmentation of exposde areas most likely suffers from …

A

Primary Biliary Cirrhosis

progressive autoimmune destruction of liver

915
Q

What is the best initial diagnostic test for suspected primary biliary cirrhosis?

A

antimitochondrial antibody level

definitive dx is with liver biopsy

916
Q

What treatment can be used to slow the progression of primary biliary cirrhosis?

A

ursodeoxycholic acid

definitive tx with liver transplant

917
Q

What is a common complication of primary biliary cirrhosis disease that should be routinely screened for?

A

Osteoporesis

screen with DEXA scan, prevent with calcium and vitamin D supplementation along with bisphosphonates if needed

918
Q

What are significant predictors of adverse cardiovascular outcomes in pts and are considered coronary heart disease risk equivalents?

A
  1. diabetes
  2. chronic kidney disease
  3. noncoronary atherosclerotic disease (carotid, peripheral artery, abdominal aortic aneurysm)
919
Q

What diagnostic test can be performed to distinguish between a true positive tuberculin skin test and induration due to prior BCG vaccination (usualy never exceeds 15 mm)?

A

interferon gamma release assay

920
Q

What is the next best step in management of a pt with positive tuberculin skin test and signs of latent tuberculosis on CXR (fibrosis)?

A

Isoniazid (INH) for 9 months (can be 6 months if adherence difficulties) with vitamin B6 (to prevent peripheral neuropathy)

(alternative options: Rifampin for 4 months in adults/ 6 months in kids, INH and rifampin for 4 months, or INH and rifapentine weekly for 3 months- not in HIV pts)

(to prevent conversion into active TB)
(can use rifampin for 4 months as alternative)

921
Q

What is used to determine whether a patient with atrial fibrillation needs anticoagulation?

A

CHADS2 score

C: CHF (1 point)
H: hypertension (1 point)
A: age older than 75 (1 point)
D: diabetes (1 point)
S: stroke/ transient ischemic attack (2 points)

If score 2 or more, need anticoagulation
if 1, can give aspirin or anticoagulation

922
Q

What is the next best step in management of a infant with unilateral/ bilateral palpable firm disc-like tissues under the areolae with/without galactorrhea?

A

Reassurance

(isolated premature thelarache due to maternal estrogen withdrawal after birth elading to prolactin production resolves within 6 months)

923
Q

What is the most common cause of sudden death due to steering wheel injuries?

A

aortic injury (aortic rupture)

924
Q

A female who develops significant postpartum hemorrhage shortly after birth in the setting of a soft uterine fundus at the level of the umbilicus most likely suffers from …

A

Uterine Atony

tx: fundal massage and oxytocin infusion
(maintain BP with IV fluids and blood transfusion)

925
Q

What is the next best step in management of a pt with pneumonia in the same location as recently treated pneumonia (recurrent pneumonia in same location)?

A

chest CT scan

(to assess for possible obstructing endobronchial neoplasma if smoker, abscess, empyema)

(if not helpful, then bronchoscopy)

926
Q

An HIV pt who develops worsening of a pre-existing infection after starting treatment with HAART therapy and antibiotics most likely suffers from …

A

Immune reconstitution inflammatory syndrome (IRIS)

(due to rapid improvement of immune function allowed by HAART therapy and subsequent inflammatory reaction at site of pre-existing infection

(reassure pt and continue treatment)

927
Q

What is the next best step in management of an elderly pt with squamous cell carcinoma who refuses surgical excision?

A

radiation therapy (requires multiple visits and risk of future malignancies)

(other options are cryotherapy and elexctrosurgery but dont provide tissue for histologic exam)

928
Q

What anti-hypertensive medications are associated with the side effect of peripheral edema?

A

calcium channel blockers (amlodipine and nifedipine more so than verapmil and diltiazem)
(use of ACE inhibitors with calcium channel blockers reduces risk of peripheral edema)

929
Q

A pt presenting with a prodrome of itching and urticaria followed by outbreak of erythematous rash with tense bullae and blistering most likely suffers from …

A

bullous pemphigoid

dx: skin biopsy from edge of intact blister with assay for basement membrane antibodies
(tx: glucocorticoids)

930
Q

A pt presenting with cirrhosis/ liver dysfunction, diabetes mellitus, bronzed skin (golden brown skin), hook like osteophytes of the 2nd and 3rd metacarpophalangeal joints, and central hypogonadism (low LH and FSH) most likely suffers from ..

A

Hemochromatosis

excessive iron accumulation in parenchymal organs
(tx: therapetic phlebotomy- remove 1 unit of blood per week)

931
Q

What are the common side effects of insulin secretagogues (sulfonylureas- glyburide; meglitinides- nateglinide/ repaglinide) used for type 2 diabetes? (2)

A
  1. hypoglycemia

2. weight gain

932
Q

What are the common side effects of thiazolidinediones (“azones”) used for type 2 diabetes? (3)

A
  1. hepatoxictiy
  2. fluid retention/ heart failure/ pulmonary edema
  3. bone fractures
933
Q

What are the common side effects of metformin used for type 2 diabetes? (3)

A
  1. GI upset
  2. lactic acidosis
  3. decreased vitamin B12 absorption
934
Q

What are the common side effects of DDP inhibitors (“gliptins”) used for type 2 diabetes? (2)

A
  1. headache

2. nasopharyngitis

935
Q

What are the common side effects of alpha-glucosidase inhibitors (acarbose, miglitol) used for type 2 diabetes? (2)

A
  1. diarrhea

2. flatulence

936
Q

What are the common side effects of SGLT2 inhibitors (“gliflozins”)used for type 2 diabetes? (3)

A
  1. polyuria
  2. UTIs
  3. hypotension
937
Q

A newborn male born with bladder distention, bladder wall thickening, palpable bladder and weak urinary stream most likely suffers from …

A

Posterior Urethral Valves

leads to vesicoureteral reflux and hydronephrosis
(dx: voiding cystourethrogram (VCUG))

938
Q

An HIV pt presenting with hemiparesis and disturbance in speech, vision and gait along with multiple demyelinating, non-enhancing lesions with no mass effect on brain MRI most likely suffers from …

A

Progressive Multifocal Leukoencephalopathy

(due to JC virus)

(tx: optimize HAART therapy)
(dx: confirmed by brain MRI)

939
Q

A young healthy pt presenting with acute non-traumatic mono or oligoarthritis along with a history of unprotected sex most, tenosynovitis (painful tendonsat wrist/ fingers/ ankle/ toes), and transient pustular or vesiculo-pustular skin rash likely suffers from …

A

Gonococcal arthritis

dx: culture joint fluid, rectum, urethra, and oral cavity

940
Q

… is defined as a misclassification of events that can result from knowing the exposure status of a patient during a study, and it can be reduced by …

A

Observer’s bias; blinding

941
Q

What is the next best step in the management of a pt with deep venous thrombosis formation secondary to reversible/ time limited risk factor (i.e. surgery, pregnancy, OCP use, trauma)?

A

warfarin anticoagulation for 3-6 months and then discontinue

if DVTs develops idiopathically, then warfarin anticoagulation for 6 months then re-evalaute for long-term need

942
Q

What is the dietary recommendation fro a child with acute diarrhea and no signs of dehydration?

A

normal, age appropriate diet with limited fats and sugars

fats and sugars can increase osmolarity of stool and delay gastric emptying which worsens diarrhea

943
Q

What is the next best step in management of a pt with acute cholangitis and signs of suppurative cholangitis whose abdominal pain, hypotension, fever, and mental confusion persist despite aggressive fluid resuscitation and antibiotics?

A

Biliary Drainage (endoscopic or percutaneous)

944
Q

What is the causes of Cushing’s syndrome (weight gain, fatigue, proximal muscle weakness, truncal obesity, buffalo hump, striae) in a pt who has increased urinary cortisol level, nonsupressible high dose dexamethasone suppression test and undecteable ACTH level?

A

Adrenal adenoma

dx: CT scan of adrenals

945
Q

A pt presenting with vesiculation and faint erythema after topical antibiotic use for a burn most likely suffers from…

A

Acute Allergic Contact Dermatitis

tx: avoid offending agent; steroids for itching and discomfort

946
Q

What is the next best step of management for a pt with type 2 diabetes being treated with metformin who is scheduled for a cardiac cathertization?

A

discontinue metformin on day of procedure and restart 2 days after procedure

(to avoid lactic acidosis from metformin use with large dose IV iodine contrast)

947
Q

True or False. Most men with prostate cancer die from other causes rather than from prostate cancer.

A

True

prostate cancer is progresses slowly

948
Q

What is the next best step in mangement of a hemodynamically stable pt with EKG findings of supraventricular tachycardia (regular and narrow QRS tachycardia)?

A

identify type of SVT by using IV adenosine or vagal maneuvers

(slows tachycardia so can unmask hidden EKG findings)

949
Q

A pt who develops chest pain that is worse with positional changes and deep respiration along with scratchy sound heard during ventricular systole over left sternal border, EKG with diffuse ST segment elevation with PR depression and JVD 1-4 days after a myocardial infarction most likely suffers from ..

A

Acute Pericarditis (infarction Pericarditis)

tx: aspirin for pain control and close observation
(usualy transient)

950
Q

A pt who develops fever, leukocytosis, pleuritic chest pain (worse with position and deep respiration), and pericardial rub weeks to months after and acute myocardial infarction most likely suffers from …

A

Dressler’s syndrome

951
Q

An uncircumsized male pt presenting with small, flesh colored dome-topped or filiform papules positioned circumferentially around the corona or sulcus of the glans penis most likely suffers from…

A

Normal Variant

not sexually transmitted, not malignant, no treatment needed

952
Q

A pt on long standing NSAID use who presents with unilateral flank pain radiating to groin, hematuria and unilateral CVA tenderness but CT of the abdomen shows mild dilation of the pelvicalyceal system without an obvious stone/ obstruction most likely has renal dysfunction (elevated creatinine) due to …

A

Analgesic (NSAID induced) nephropathy

953
Q

What are the two most important predictors of survival for a pt with chronic obstructive pulmonary disease (COPD)?

A
  1. FEV1 (
954
Q

… occurs when a screening test detects the disease at an earlier point in time (making it look like the survival rate increased), but the associated prognosis of the disease does not actually change

A

Lead time bias

955
Q

A pt presenting with stearatorrhea, vitamin D deficiency, iron deficiency anemia, vitligo, dermatitis herpetiformis, peripheral neuropathy and bone pain most likely suffers from …

A

Celiac Disease

dx: anti-endomysial antibody and anti-tissue transglutaminase antibody
(small bowel biopsy is gold standard)

956
Q

An alcoholic who is being hospitalized and treated with fluids and glucose and then develops muscle weakness and other signs of rhabdomyolysis most likely suffers from …

A

Hypophosphatemia

(due to insulin increased shifting phosphate intracellularly and unmasking compensated phosphate depletion due to poor feeding)

957
Q

A pt older than 50 presenting with subacute diffuse muscle pain that is bilateral and associated with morning stiffness for more than 1 month and involving neck/torso/ shoulders/proximal arms/ proximal thigh/ hip and may have constitutional symptoms (fever, malaise, weight loss), on physical exam have decreased active range of motion in shoulders, hips and neck most likely suffers from …

A

Polymyalgia Rheumatica

elevated ESR and CRP
(tx: low dose glucocrticoids/ prednisone)

958
Q

What is the next best step in management of a female pt aged 21-24 years old with atypical squamous cells of undtermined significance (ASCUS) or low-grade squamous intraepithelial lesion (LSIL) on Pap smear?

A

repeat Pap smear in 1 year

colposcopy if pt demonstrates ASCUS or LSIL on 3 consecutive pap smear

959
Q

What is the next best step in management of a female pt aged older than 25 with atypical squamous cells of undtermined significance (ASCUS) or low-grade squamous intraepithelial lesion (LSIL) on Pap smear?

A

HPV DNA testing

if negative, perform HPV DNA testing with Pap smear in 3 years

960
Q

What is the best initial treatment for a preschool aged child (3-5 years old) who has attention deficit hyperactivity disorder (ADHD)?

A

parent-child behavioral therapy

if fails, then stimulant medication (methylphenidate or amphetamines)

961
Q

What is the best initial step in management of a pt with attention deficit hyperactivity disorder before starting pt on stimulant medication (methylphenidate or amphetamines)?

A

detailed cardiac and physcial examination

atomoxetin- NE reuptake inhibitor and alpha-2 agonist are non-stimulant options for ADHD

962
Q

What is the next best step in management of a hospitalized pt who is found to have decreased total T3 with normal total T4 and TSH on thyroid function studies?

A

repeat thyroid function studies in 8 weeks

most likely pt has euthyroid sick syndrome- TSH increases above normal when recovering from illness

963
Q

A pt with fever, nasal congestion/ obstruction, prurulent nasal discharge, maxillary tooth discomfort, facial pain/ pressure that is worse with bending forward as well as symptoms persist for 10 days or more without improvement/ severe symptoms with high fever (greater than 39), purulent nasal discharge or facial pain for more than 3 days/ worsening symptoms more than 5 days after an initial improving viral upper respirtaory infection most likely suffers from …

A

Acute Bacterial Rhinosinusitis

tx: amoxicillin-clavulanate

964
Q

What is the best initial treatment for an infant aged 28 days or less than presents with sepsis?

A

ampicillin with gentamicin or cefotaxime

E. Coli and Group B strep, as well as Listeria

965
Q

What is the best initial treatment for an infant aged older than 28 days presenting with sepsis?

A

ceftriaxone or cefotaxime within or without vancomycin

Step pneumoniae and Neisseria meningitidis

966
Q

A pt presenting with lower leg pain and tibial tenderness in the setting of vigorous running regimen and bothersome shoes, associated with local pain that increases with jumping/running and local swelling and point tenderness most likely suffers from …

A

Tibial Stress Fracture

(no bone abnormalities on initial X-rays for up to 4 weeks)
(tx: pneumatic splinting and reduced weightbearing with graduated exercise program)

967
Q

A pt with delayed puberty, retarded bone age (bone age less than chronological age) and a positive family history without any evidence of systemic disorder most likely suffers from …

A

Constitutional Pubertal Delay

tx: reassurance, psychology referral for psychosocial issues and follow-up in 3-6 months

968
Q

What is the best initial treatment for benign paroxysmal positional vertigo (vertigo triggered by certain provocative positions)?

A

canalith repositioning procedure

to move particles out of posterior semicircular canal

969
Q

What is the most common cause of viral meningitis or encephalitis in the pediatric popultion?

A

enterovirus and arbovirus

zoonosis transmitted via mosquitos

970
Q

What is the best initial treatment for community acquired pneumonia for a pt being hospitalized for the pneumonia (due to CURB 65)?

A

IV ceftriaxone and azithromycin/ clarithromycin

Strep pneumoniae and Legionella

971
Q

A boy (toddler age) presenting with speech delay, bilateral lower extremity weakness, increase in calf diameter, and family history of similar symptoms who demonstrates difficulty walking/ running/ jumping/ climbing stairs and must push their arms on their thighs to transition from sitting to standing most likely suffers from …

A

Duchenne muscular dystrophy

X-linked defect of dystrophin gene on X-chromosome p21

972
Q

What is the initial step in management of a pt with suspected Duchenne muscular dystrophy?

A

obtain serum creatine phsophokinase (CPK) level

markedly elevated by age 2 and decreases with advancing disease

973
Q

An HIV pt with CD4 count less than 200 cells/ cubic mm presenting with low grade fever, cough, dyspnea, tachypnea and with CXR findings of diffuse bilateral ground glass opacities most likely suffers from …

A

Pneumocystis pneumonia (PCP)

tx: IV Bactrim/ trimethoprim-sulfamethoxazole
(dx: fiberoptic bronchoscopy with bronchoalveolar lavage)

974
Q

An HIV pt presenting with painful swallowing (odynophagia), burning chest pain and giant ulcers with no virus found on biopsy and failure to respond to anti-fungal medicationmost likely suffers from …

A

Aphthous ulcers

tx: prednisone

975
Q

What is the best initial step in management of a pt with anaphylaxis (respiratory distress, hypotension, itching, hives, swelling of lips/ tongue)?

A

assess airway, breathing and circulation and administer IM epinephrine

(place pt prone with lower extremities elevated)
(followed by fluid resuscitation, bronchodilators, antihistamines, steroids)

976
Q

A child with a history of anemia who presents with Salmonella osteomyelitis (long bones have multiple foci on bone scan) most likely suffers from ..

A

Sickle Cell Disease

dx: hemoglobin electrophoresis

977
Q

What is the next step in management for a pregnant pt who has second stage arrest of labor (lack of progress toward delivery after 3 hours or more without epidural/ 4 hours or more with epidural in a nulliparous women; or 1 hour less for multiparous women)?

A

Cesarean Delivery

978
Q

What are the 4 indications for lipid-lowering therapy with statin?

A
  1. clinically significant atherosclerotic disease (ACS, MI, angina, coronary/ arterial revascularization, stroke, TIA, PAD) (if aged 75 or older, high intensity statin)
  2. LDL greater than or equal to 190 (high intensity statin)
  3. age 40-75 with diabetes (if 10 year risk 7.5% or greater than high intensity statin)
  4. 10 year risk is 7.5% or greater
979
Q

What is the best initial treatment for symptomatic peripheral arterial disease?

A

supervised exercise program (30-45 minutes 3x a week for 3 months) with antiplatelet therapy (aspirin/ clopidogrel)

(if fails, then cilostazol)
(if cilostazol fails or limb threatening disease, then revascularization)

980
Q

What is the next best step in management of cocaine-induced myocardial infarction that is not responding to nitrates, aspirin and benzos?

A

immediate coronary angiography

to assess for coronary thrombus

981
Q

A pt presenting with flushing followed by cyanosis, altered mental status, metabolic acidosis (from lactic acidosis), and GI distress in the setting of renal failure and nitroprusside use most likely suffers from …

A

Cyanide Toxicity

(due to nitroprusside is metabolized to cyanide leads to cyanide accumulation)
(tx: stop nitroprusside and give sodium thiosulfate)

982
Q

What non-pharmacological measure would be most effective in decreasing a patient’s blood pressure?

A

10% weight loss (reduce BMI to less than 25)

983
Q

What is the best diagnostic test for suspected gout?

A

arthrocentesis (synovial fluid contains monosodium urate crystals that are negatively birefringent, needle shaped crystals under polarizing light)

984
Q

What vaccines that are not recommended for the general population should be given to HIV patients?

A
  1. pneumococcus (PCV13 followed by PPSV23 8 weeks later and every 5 years)
  2. Hep A and Hep B (if no documented immunity)
985
Q

What vaccines are contraindicated in HIV patients with CD4 cell count less than 200?

A

live attenuated vaccines

MMR, Varicella, Zoster, live attenuated influenza

986
Q

A pt older than 50 presenting with new onset localized headache, fever, visual disturbances, elevated ESR (greater than 50), and tenderness/ decreased pulse of temporal artery most likely suffers from …

A

Giant Cell Arteritis

(dx: temporal artery biopsy- necrotizing arteritis with mainly mononuclear cells)
(tx: high dose steroids- initiate prior to biopsy)
(associated with polymyalgia rheumatica)

987
Q

What is the next best step in management for a pt with subclinical hypothyroidism (mild elevated TSH with normal free T4)?

A

measure antithyroid peroxidase (anti-TPO) antibodies

(treat if antithyroidantibodies present, an abnormal lipid profile, symptoms of hypothyroidism, or ovulatory/ menstrual dysfunction)

988
Q

What is the best treatment for the managing the TCA (tricyclic antidepressant)- induced cardio toxic effects (of QRS widening or ventricular arrhthymia)?

A

Soidium Bicarbonate

989
Q

A 2 year old child presenting with painless hematochezia (bloody stools), abdominal distension and a technetium-99 nuclear scan showing increased uptake in the right lower quadrant most likely suffers from …

A

Meckel’s diverticulum

(ectopic gastric mucosa located within 2 feet of ileocecal valve)

(dx: technetium-99 nuclear scan/ Meckel’s scan)
(tx: surgery if symptomatic)

990
Q

What is the next best step in management of a pt who develops diarrhea after cholecystectomy/ ileal resection or if have history of short bowel syndrome?

A

Cholestyramine (bile salt binding resin)

diarrhea is due to excess secondary bile acids stimulating colon

991
Q

What is the next best step in management of a female pt who has had more than 2 UTIs in six months or more than 3 UTIs in a year (in the absence of nephrolithiasis or obstruction)?

A

antibiotic prophylaxis (continuously or just after sex)

992
Q

What is the next best step in management of a female pt planning to become pregnant?

A

folic acid for at least 1 month prior to conception through first trimester (to avoid neural tube defects)

(0.4 mg/day for general population)
(4 mg/day for women on anticonvulsants or have hx of prior child with neural tube defect)

993
Q

What is the next best step in management of a pt with dyspepsia (epigastric discomfort/pain that is worse after eating, postprandial fullness, early satiety, heartburn)?

A

typical GERD (heartburn): acid suppression (PPI)

NSAID use: discontinue NSAID and acid suppression (PPI)

alarming symptoms (weight loss, worsening dysphagia/odynophagia, hematemesis, anemia, early satiety, older than 55): upper GI endoscopy

high prevalence region (Mexico, Asia, Eastern Europe, Latin and South America): H pylori testing

994
Q

How long should a pt remain on antidepressant medication for a single episode of depression once the symptoms are in remission?

A

at least 6 months to 1 years (to avoid relapse)

995
Q

What is the best initial form of contraception for patients with sickle cell disease?

A

progestin-releasing intrauterine device

IUD most effective form; copper associated with heavy menses which can lead to anemia

996
Q

An intoxicated pt presenting with disorientation, restlessness, bilateral vertical nystagmus, as well as hypertension and tachycardia most likely is intoxicated with …

A

Phencyclidine (PCP)

keep pt in low-stimulation environment

997
Q

A pt with hypothermia/ renal failure/ hepatic failure/ shock who develops muscle spasms in the face and upper extremities, bilateral hand contractures, and generalized tonic-colonic sezirues after blood transfusion most likely suffers from ….

A

Hypocalcemia

(due to inability to metabolize citrate that is in blood transfusion; citrate binds calcium leading to low calcium)

(tx: calcium replacement)
(prevent: give 10 cc of 10% calcium gluconate for every 500 ml packed RBCs transfused)

998
Q

What is the best treatment for stage 1 renal cell carcinoma (renal mass confined withing renal capsule)?

A

partial nephrectomy

999
Q

What is the best treatment for stage 2 renal cell carcinoma (extends through renal capsule but nor beyond Gerota’s fascia)?

A

radical nephrectomy

1000
Q

…. study is a study in which individuals are assessed at specific point in time to determine whether or not they have a certain risk factor and a certain disease of interest

A

Cross-sectional study

1001
Q

What are the 5 characteristics of a skin lesions that suggest malignancy?

A
  1. asymmetry (A)
  2. border irregularity (B)
  3. color variation (C)
  4. diameter greater than 6 mm (D)
  5. evolving (lesion changing in size, shape, color or new lesion) (E)
1002
Q

What is the next best step in management of a pt with a skin lesion suspicious of being malignant (ABCDE positive)?

A

excisional biopsy with elliptical/ punch biopsy that includes entire lesion with 1-3 mm margins

1003
Q

What is the next best step in management of a hypothyroid pt on levothyroxine who starts estrogen (combined OCPs; hormone replacement therapy)/ tamoxifen/ raloxifene/ methadone/ heroin?

A

recheck TSH in 12 weeks and adjust levothyroxine dose

most likely will need to increased due to drugs causing increase thyroxine-binding globulin which decreases free T4

1004
Q

What is the next best step in management of a hypothyroid pt on levothyroxine who starts androgens/ danazol/ anabolic steroids/ glucocorticoids?

A

recheck TSH in 12 weeks and adjust levothyroxine dose

most likely will need to decrease due to drugs causing decrease thryoxin- binding globulin which increases free T4

1005
Q

What are the recommendations for tetanus prophylaxis after a wound?

A

clean/ minor wound:
if more than 3 prior toxoid doses: tetanus toxoid vaccine (if last booster given more than 10 years ago)
if unimmunized/ uncertain/ less than 3 prior toxoid doses: tetanus toxoid vaccine

dirty/ severe wound:
if more than 3 prior toxoid doses: tetanus toxoid vaccine (if last booster given more than 5 years ago)
if unimmunized/ uncertain/ less than 3 prior toxoid doses: tetanus toxoid vaccine with tetanus IG

1006
Q

What is the next best step in management of a pt who develops statin-induced myopathy following vigorous exercise?

A

discontinue statin followed by rechecking serum creatine kinase level and then restarting the statin if level normalized

1007
Q

A pt presenting with erythema in the central face associated with flushing, telangiectasias, and pustules and flushing is triggered by hot/ spicy foods/ alcohol/ temperature extremes/ emotional distress most likely suffers from …

A

Rosacea

tx: papulopustular- topical metronidazole; erythema and telangiectasias only- topical brimonidine and avoid triggers

1008
Q

What is th emost common complication of rosacea?

A

ocular manifestations so should have ophthalmology consult

burning/ foreign body sensation, blepharitis, keratitis, conjunctivitis, corneal ulcers, recurrent chalazion

1009
Q

A pt presenting with a painless pea-sized nodule within the eyelid most likely suffers from …

A

Chalazion

high recurrence rate in pts with rosacea

1010
Q

A pt with macrocytic anemia, normal folate level, depressed serum cobalamin level and gastrointestinal symptoms most likely suffers from …

A

Pernicious Anemia

dx: anti- intrinsic factor antibodies; can also have anti-parietal antibodies

1011
Q

What are the gastric endoscopic findings associated with pernicious anemia?

A

glandular atrophy, intestinal metaplasia and inflammation; abent rugae in the fundus and gastric body

1012
Q

What is the best initial treatment for metoclopramide induced acute dystonia (deviation of head and eyes, involuntary tongue movements)?

A

IV diphenhydramine

alternatives: anticholinergives- benzotropine, trihexyphenidyl

1013
Q

What is the cause of secondary amenorrhea in female atheletes?

A

estrogen deficiency (due to low LH and GnRH)

associated with infertility, vaginal and breast atrophy, osteopenia/ osteoporesis, mild hypercholesterolemia

1014
Q

What is the next best step in management for persistent (more than 3-4 weeks)and localized abnormal lymp nodes?

A

lymph node biopsy (to assess for presence of lymphoma)

1015
Q

A pt with a history of IV drug use who presents with sudden onset shortness of breath, high fever and dry cough along with multiple, round lesions in lung bilaterally on CXR most likely suffers from …

A

Septic Pulmonary Embolism

(due to septic thrombophlebitis- painful subcutaneous mass in cubital area or due to tricuspid endocarditis)

(next best step: blood cultures and empiric antibiotics)

1016
Q

A pt found to be HBsAg positive who presents with fever, skin rash and arthralgias along with elevated transaminases most likely suffers from …

A

Serum Sickness-like syndrome

due to circulating immune complexes

1017
Q

A pt who presents with irregularly shaped hyperpigmentated macules on sun exposed areas of the face during pregnancy most likely suffers from ..

A

Melasma

tx: use sunscreen, avoid sun exposure
(usually resolves after delivery)

1018
Q

What is the next best step in management for a pt with chronic myeloid leukemia (fatigue, poor exercise tolerance, weight loss, splenomegaly, leukocytosis, bcr/abl translocation- Philadelphia chromosome consisting of chromosomes 9 and 22)?

A

Tyrosine Kinase Inhibitor (imatinib)

1019
Q

A pt presenting with diarrhea, mucus discharge, tenesmus (ineffectual/ painful straining on defecation), rectal bleeding and colonoscopy findings of pallor, friability, telangiectasias, and mucosal hemorrhage after treatment of cancer most likely suffers from …

A

Radiation Proctitis

tx with supportive measures like fluid and anti-dirrheal; if chronic use sucralfate or steroid enemas

1020
Q

What is considered arrest of labor in the first stage of labor?

A

6 cm or greater cervical dilation, ruptured membranes and either:

  • no cervical change for 4 hours of more despite adequate contractions (200-250 MVUs or more)
  • no cervical change for 6 hours or more with inadequate contraction
1021
Q

A male pt presenting with difficulty maintaining and achieving erections but has preserved nocturnal and early morning erections most likely suffers from …

A

Psychological erectile dysfunction

due to performance anxiety, depression, relationship conflict, loss of attraction

1022
Q

What is the best initial treatment for squamous cell carcinoma of the glottis that is confined to one vocal cord without metastasis?

A

laser excision of the lesion or radiotherapy

1023
Q

A child age less than 2 years old presents with nasal congestion, rhinorrhea and cough that progressed to respiratory distress (tachypnea, retractions, nasal flaring, apnea, cyanosis) and has bilateral crackles and wheezing especially in winter months on exam most likely suffers from …

A

Respiratory Syncytial Virus Bronchiolitis

(prevent: Palivizumab in infants who were preterm/ born before 29 weeks, have chronic lung disease of prematurity, hemodynamically significant congenital heart disease)
(tx: supportive care and respiratory isolation)

1024
Q

What are the indications for slow tapering of glucocorticoids when they are no longer needed for treatment?

A
  1. used more than 3 weeks (moderate to high dose of 20 mg prednisone or higher)
  2. used evening dosing
  3. cushingoid signs or symptoms

(if unsure, test for HPA suppression with low dose ACTH stimulation test; if dont have increased cortisol then need taper)

1025
Q

When should hep B serology testing be done in an infant born to a woman with Hep B infection (assuming given appropriate prophylaxis once born)?

A

between 9-15 months old

1026
Q

When can a noncontact athlete return to usual activities after suffering from a clavicular fracture?

A

4-6 weeks after injury in young pt

must have painless, full active ROM; near normal strength; and evidence of bridging callus

1027
Q

A pt presents with asymptomatic petechiae and ecchymosis, as well as mucocutaneous bleeding (epitaxis, hematuria, GI bleed) following a viral infection and is found to have isolated thrombocytopenia (less than 100,000) and megakaryocytes on peripheral smear most likely suffers from …

A

Immune Thrombocytopenic Purpura (ITP)

due to platelet destruction by antiplatelet antibodies directed at GPII/IIIa

1028
Q

What is the treatment for immune/idiopathic thrombocytopenic purpura (ITP) in kids?

A

just skin manifestation: observe

active bleeding: glucocorticoids (first line) or IVIG

1029
Q

What is the treatment for immune thrombocytopenic purpura (ITP) in adults?

A

platelets 30,000 or greater without bleeding: observe

platelets less than 30,000 or active bleeding: glucocorticoids (first line) or IVIG

1030
Q

A pt presenting with tachypnea, tacycardia, shallow breathing, anterior chest bruises and signs of inadequate ventilation (cyanosis) after an accident most likely suffers from …

A

Flail Chest

(due to double rib fractures in multiple sites)
(associated with paradoxical/ segmental chest wall movement)
(tx: Oxygen, pain control, positive pressure venitlation, surgical stabilization)

1031
Q

A pt presenting with fever, severe focal spinal pain and neurologic deficits (areflexia, motor and sensory deficits, bowel/ bladder dysfunction) especially after spinal trauma most likely suffers from …

A

Spinal Epidural Abscess

(dx: MRI of spine with gadolinum followed by CT guided aspiration and culture)
(obtain CBC, ESR, CRP, blood cultures)
(tx: immediate surgical decompression and antibiotics)

1032
Q

What is the next best step in management of a child (younger than adolescent) who presents with sexually transmitted disease (trichomonas, chlamydia, gonorrhea, HIV)?

A

inform child protective services about possible sexual abuse and separate child from possible abuser

1033
Q

What is the mahjor side effect associated with clozapine use (usualy for schizophrenia)?

A

agranulocytosis

monitor WBC count weekly for first 6 months, then can decrease to bimonthly and then to monthly

1034
Q

A pt with cirrhosis who presents with altered mental status, shifting dullness/ asctite without peripheral edema, and new increase in creatinine with a BUN: creatinine ratio of 20:1, renal ultrasound shows no evidence of obstruction, and peritoneal fluid has less than 250 cells/ mm3 most likely suffers from …

A

Hepatorenal syndrome

dx: volume challange- pt fails to respond so not due to intravascular volume depletion
(tx: octreotide and midodrine)

1035
Q

A pt with a history of Hasimoto’s thyroiditis (elevated TSH, low T4 and antithyroid peroxidase antibodies present) who presents with rapidly enlarging thryoid gland most likely suffers from …

A

Thyroid Lymphoma

1036
Q

What is the Pemberton’s test and what does it signify?

A

have pt raise arms over head for 60 seconds; if facial plethora or engorgement of neck veins occurs, then thyroid is source of pt’s esophageal obstruction (dysphagia- difficulty swallowing)

1037
Q

What is the next step in management for a child less than 24 months of age who presents with a UTI other than antibiotics?

A

renal and bladder ultrasound

(to assess for anatomic abnormalities: hydronephrosis, renal scarring, high grade vesicoureteral reflux, obstructive uropathy)
(if abnormal ultrasound or recurrent UTIs, then perform voiding cystourethrogram)

1038
Q

What diagnostic studies are used to assess the etiology of a patient’s stroke?

A
  1. head CT- intracranial hemorrhage
  2. MRA/CTA (evaluate intracranial vasculature)
  3. carotid duplex (arotids)
  4. EKG (MI/ arrhythmia leading to intracardiac thrombus)
  5. ECHO (intracardiac thrombus presence)
1039
Q

A pt presenting with well-demarcated circular , non-scarred patch of complete hair loss with the presence of hairs that are tapered near the insertion into the scalp (exclamation point hairs) near periphery of patch, along with nail pitting most likely suffers from …

A

Alopecia Areata

(tx: topical or intralesional steroids)
(autoimmune process)

(prognosis: chance of recurrence after successful tx)

1040
Q

A woman presenting with unilateral lower abdominal pain, ipsilateral adnexal tenderness, amenorrhea, vaginal bleeding and positive urine beta HCG most likely suffers from …

A

Ectopic Pregnancy

(dx: transvaginal ultrasound not showing intrauterine pregnancy with hcg greater than 1500, but showing adnexal mass)

(if no adnexal mass or intrauterine pregnancy on transvaginal ultrasound, repeat HCG and transvaginal ulatrasound in 48-72 hours)

1041
Q

What is the best treatment for tinea pedis (pruritus of the feet, erythema and scaling between toes, can extend to soles and sides of feet with sharp border)?

A

mild: topical terbinafine/ miconazole/ clotrimazole

if extensive/ fail topical: oral terbinafine/ miconazole/ clotrimazole

1042
Q

What is the best prevention regimen for prolonged sun exposure or UV radiation-induced skin damage?

A
  1. use sunscreen (SPF 15 for regular use, SPF 30 for outdoor work/ reaction) and protective clothing
  2. maintain adequate hydration
1043
Q

What is the best treatment for displaced clavicular fractures?

A

open reduction and internal fixation

use conservative therapy for non-displaced clavicular fractures

1044
Q

A pt with prior history of surgery who presents with nausea, vomiting, soft distended abdomen, decreased bowel sounds, and gaseous distension of small and large bowel (gas pattern in colon and rectum) most likely suffers from ….

A

Hypokalemia induced ileus

especially if on loop diruetic
(tx: potassium replacement)

1045
Q

What type of cancer has been associated to be at increased risk for patients who are being treated with tamoxifen (used for estrogen receptor positive breast cancer)?

A

endometrial/ uterine cancer

(perform routine GYN and Pap smear exams; only perform transvaginal ultrasound or endometrial biopsy if postmenopausal and symptomatic with bleeding or vaginal symptoms)

1046
Q

What is the best initial approach to smoking cessation in a pregnant pt?

A

phsycian directed counseling intervention (5 A’s and behavioral therapy)

1047
Q

What is the best treatment for a hospitalized pt suffering from opioid withdrawal (lacrimation, yawning, mydriasis, abdominal cramps, rhinorrhea, body aches, piloerection)?

A

methadone or buprenorphine

can also use clonidine if not willing to be in supervised detox setting

1048
Q

What is the next best step in management of a pt with Whipples triad (hypoglycemia, symptoms of hypoglycemia, and resolution of symptoms with glucose) who are found to have elevated insulin, C-peptide and proinsulin? (suggesting endogenous insulin secretion)

A

serum assay for oral hypoglycemic agents (sulfonylurea or meglitinide)

(differentiates between insulinoma and surreptitious use of oral hypoglycemic meds)

1049
Q

A child presenting with fever, drooling, sore throat, decreased appetite and painful vesicles and ulcers on the posterior oropharynx most likely suffers from ..

A

Herpangina

due to Cosackie A virus
(tx: supportive with saline gargles, analgesics and antipyretics)

1050
Q

A pt presenting with recurrent ulcers on anterior oral mucosa (lips, cheeks, mouth floor, ventrum of tongue) most likely suffers from …

A

Aphthous stomatitis (canker sores)

1051
Q

What is the best treatment for chronic bacterial prostatitis?

A

Ciprofloxacin/ levofloxacin for 6-12 weeks

alternative is trimethoprim- sulfamethoxazole

1052
Q

What fasting blood glucose level is considered impaired glucose tolerance?

A

100 to 125 mg/dl

increased risk of coronary artery disease and progression to diabetes

1053
Q

What is the best intiial treatment for dysfunctional uterine bleeding (heavy vaginal bleeding, irregular menses, negative pregnancy test)?

A

combination oral contraceptive pills with high dose estrogen

1054
Q

What is the diagnostic test of choice for a pregnant pt with suspected renal colic/ kidney stone (flank pain sudden, severe and paroxysmal, hx of previous stones)?

A

Renal and Pelvic ultrasound

if negative, use transvaginal ultrasound
(usually use helical CT scan in non-pregnant pt)

1055
Q

What is the best initial diagnostic test for suspected hemochromatosis?

A

fasting transferrin and ferritin level
(fasting transferrin greater than 50%)
(gold standard- liver biopsy)

1056
Q

What is the best treatment for superior saggital sinus thrombosis (acute progressive headache, vomiting, contralateral hemiparesis, asymmetric deep tendon reflexes, unilateral hemorrhage in white matter, seizure, papilledema, thrombosis seen on MRI or MR venography)?

A

heparin

hemorrhage is secondary to venous hypertension so ok to anticoagulate

1057
Q

What are the three most common causes of fever of unknown origin (fever above 38.3 C on multiple occasions over a time span of 3 weeks or more in the setting of negative basic evaluation)?

A
  1. connective tissue disease (22%)
  2. infection (16%)
  3. malignancy (7%)
1058
Q

What is the most characteristic biochemical pattern associated with alcoholic liver disease?

A

AST/ALT ratio of 2.5

AST elevates disproportionately to ALT

1059
Q

A postpartum pt presenting with sweating, heat intolerance, SOB, tachycardia and has low TSH with high T4 with a low radioactive iodine uptake study result, elevated thyroglobulin, positive anti-TPO and painless enlarged thyroid most likely suffers from …

A

Postpartum Thyroiditis

(thyroglobulin released and low RAIU with destruction of thyroid gland that occurs in thyroditis)
(starts as hyperthyroid then transitions to hypothyroid before recovery)

1060
Q

What are the three major indications for inferior vena cava (IVC) placement for thromboembolism?

A
  1. complications of anticoagulation (substantial bleeding)
  2. contraindications to anticoagulation
  3. failure of anticoagulation in the setting of known DVT/ PE
1061
Q

What is the next best step in management of a pt with small bowel obstruction (crampy abdominal pain, abdominal distension, no bowel movements, vomiting, multiple air fluid levels in small intestine)?

A

observation and supportive treatment (V hydration, NG suctioning, correction electrolyte abnormalities)

(if no improvement after 12-24 hours, then surgical intervention)

1062
Q

What is the sequence of medical tratment for heart failure?

4

A
  1. ACE inhibitor/ ARB (for class 1- no limits with physical activity)
  2. Diuretic, beta blocker (if EF 40% or less), spironolactone (if EF less than 30% with stable renal function and K) (for class 2- oridanary acitivty causes fatigue/ dyspnea/ palpitations)
  3. isosorbide dintrate/ hydralazine (if black), digoxin (if symptomatic with spironolactone), cardiac resynchronization (if QRS greater than 150 msec) (class 2-3)
  4. transplant/ ventricular assisted device (class 4- not able to do physical activity without symptoms/ symptoms at rest)
1063
Q

What are the glycemic goals in the hospital setting (pre-meal, post-meal/ random, on insulin infusion)?

A

pre-meal: 100-140 mg/dl

post-meal/ random: less than 180 mg/dl

insulin infusion: 140-180 mg/dl

1064
Q

What is the best initial treatment for locally advanced head and neck cancer?

A

combined radiotherapy and chemotherapy (chemoradiotherpy)

can render some inoperable head and neck cancers operable after treatment

1065
Q

What is the best management for abdominal aortic aneurysm?

A
  1. smoking cessation
  2. aspirin and statin therapy
  3. elective repair (if large- 5.5 cm or greater, rapidly enlarging- 0.5 cm/ 6months or faster, if associated with peripheral artery disease)
1066
Q

How often should you perform a follow-up nultrasound for an abdominal aortic aneursym>

A

medium (4-5.4 cm): every 6-12 months

smaller (less than 4 cm): every 2-3 years

1067
Q

What intervention decreases the likelihood of expansion of an abdominal aortic aneurysm the most?

A

smoking cessation

1068
Q

What is the best treatment for pancreatogenic diabetes (hyperglycemia secondary to chronic pancreatitis damaging beta cells)?

A

mild: metformin
severe: insulin

1069
Q

A child aged 6 months to 3 years presents with fever, harsh/ hoarse/ barking cough, hoarseness and inspiratory stridor in fall/ early winter with steeple sign (subglottic narrowing on X-ray) most likely suffers from …

A

Croup (laryngotracheitis due to parainfluenza)

1070
Q

What is the best treatment for croup (laryngotracheitis)?

A

mild (no stridor at rest): humidified air with/ without steroids

moderate/ severe (stridor at rest): steroids and nebulized epinephrine

1071
Q

A pt presenting with fever, malaise, myalgia, arthralgia/ arthritis, serositis (pleuritis, pericarditis) and/or hepatomegaly/ splenomegaly while being treated with procainamide/ hydralazine/ minocycline most likely suffers from …

A

Drug-Induced Lupus

dx: antinuclear antibodies (ANA) and anti-histone antibodies
(tx: discontinue use of offending drug)

1072
Q

What is the prognosis for absence seizures (staring spells associated with generalized 3 Hz spike and wave activity on EEG)?

A

episodes diminish with age and treatment is usually effective (ethosuximide)

(if generalized tonic-clonic seizures absent)

1073
Q

A pt presenting with sudden onset severe periumbilical pain out of proportion to initial examine without signs of peritoneal signs (guarding, rebound tenderness), nausea, vomiting, metabolic acidosis/ lactic acidosis, leukocytosis, and hemoconcentration most likely suffers from …

A

Acute Mesenteric Ischemia

(dx: CT angiogram; CT may show focal/ segmental bowel wall thickening, intestinal pneumatosis with portal vein gas, dialteral bowel, mesenteric stranding)

1074
Q

A pt presents with eye pain and redness, photphobia, visual loss, unlateral constricted and irregular pupil, hazy flar, redness at the juntction between the cornea and sclera, and leukocytes in the anterior segment of the eye most likely suffers from ….

A

Anterior Uveitis (iritis)

tx: antimicrobials if due to virus/ bacteria; steroids if non-infectious

1075
Q

A pt with hypertension after kidney transplant that develops acute kidney injury (increase in creatinine) after the initiation of an ACE inhibitor most likely has hypertension due to …

A

Renal Artery Stenosis

(ACE inhibitor decreases renin-angiotensin- aldosterone stimulation that occured secondary to renal hypoperfusion from renal artery stenosis; thereby decreasing the hypertension but allowing hypoperfusion of kidney)

1076
Q

A neonate who is found to have asymmetric folds in one thigh, ipsilateral leg shortening when supine with knees flexed and feet resting on flat surface, and a clunking sound when hip dislocation is attempted (Barlow test) and is able to be relocatable and may have history of breech delivery most likely suffers from …

A

Developmental dysplasia of the Hip

tx: Pavlik harness- maintain hip in flex, abducted position for 1-2 months, managed by orthopedic surgeon

1077
Q

A pt presents with abdominal pain, diarrhea, postprandial nausea and vomiting, dizziness, sweating, and shortness of breathe after undergoing a gastrectomy procedure most likely suffers from …

A

Dumping Syndrome

(due to liquid and food passing thru stomach into jejunum faster)
(tx: high protein diet and smaller, more frequent meals)

1078
Q

When is reconstruction surgery for cleft lip with or without cleft palate ususaly performed?

A

rule of 10: 10 pounds of weight, 10 weeks of age, 10g of hemoglobin

(about 3 months old)
(recurrence in following pregnancies is about 50%, not commonly associated with cardiac and renal abnormalities)

1079
Q

What is the best next step in management of a pregnant pt in her first trimester on levothyroxine for hypothyroidism?

A

increase dose of levothyroxine

due to decreased free T4/T3 levels b/c of increased thyroxine-binding globulin during pregnancy

1080
Q

A pt presenting with preoccupation with multiple unexplained medical symptoms for 6 months or more leading to significant social and occupational impairment most likely suffers from …

A

Somatic Symptom Disorder (Somatization Disorder)

(misinterpret normal bodily sensations)
(tx: schedule regular visits with same provider, limit unnecessary workup and referrals, assure that serious illness ruled out, functional improvement goal, refer to psychiatrist/ therapist)

1081
Q

What is the best long-term treatment for a pt with atrial fibrillation who is unable to achieve adequate heart rate control, has recurrent symptomatic episodes, or has heart failure symptoms in setting of underlying left ventriuclar systolic dysfunction?

A

antiarrhythmic (rhythm control with amiodarone or dronedarone)

(dronedarone- coronary artery disease without CHF)
(amiodarone- heart failure, LV hypertrophy)

1082
Q

A pt who is found to have warty projections in the anogenital region in which the lesions turn white with acetic acid is applied most likely suffers from …

A

Condylomata acuminata (anogenital warts due to HPV)

1083
Q

What is the best initial treatment for condylomata acuminata (anogenital warts)?

A

chemical destruction with trichloroacetic acid

podophyllin application is similiar but not indicated for internal use/ mucosal surfaces or in pregnant pts

1084
Q

A pt with a history of smoking, thin, mild respiratory symptoms (coughing, wheezing), weight loss, lung mass on CXR and hyponatremia in setting of concentrated urine most likely suffers from …

A

Syndrome of inappropriate antidiuretic hormone (SIADH) secondary to small cell carcinoma

1085
Q

What is the best treatment for hyponatremia secondary to syndrome of inappropriate antidiruteic hormone (SIADH)?

A

water restriction

as long as there are no neurologic symptoms from the hyponatremia

1086
Q

What is the best initial diagnostic test for a non-pregnant pt with suspected nephrolithiasis (severe unilateral abdominal/ flank pain, radiates to groin, sharp in nature, associated with nausea, hematuria)?

A

non-contrast helical CT scan

(intravenous pyelogram is second line)
(use ultrasonogram for pregnant pts or pts who you want to avoid radiation exposure in)

1087
Q

A pt with unilateral shoulder pain that starts off being worse at night but then gradually progressive to stiffness that limits active and passive range of motion in all directions (must be in at least 2 or more planes) most likely suffers from …

A

Adhesive Capsulitis (Frozen Shoulder syndrome)

(glenohumeral joint lsoes normal distensibility due to chronic inflammation, fibrosis and contracture of the joint capsule)

(tx: rest and gentle ROM exercise; if fail, then steroid injection)

1088
Q

A pt with history of burning abdominal pain presenting with sudden onset severe upper-abdominal pain that radiates to back and is exacerbated by any movement and is found to lie flat and motionless in bed to prevent pain with significant abdominal tenderness and guarding on palpation most likely suffers from ….

A

Bowel Perforation with peritonitis (secondary to perforated peptic ulcer)

(dx: upright CXR showing pneumoperitoneum; if unclear on CRX, then abdominal CT)

1089
Q

What is the best next step in management of a pt with peptic ulcer perforation?

A

IV fluids and antibiotics along with IV proton pump inhibitor with emergent surgery

1090
Q

What is the next best step in management of a pt with acute hepatitis B infection?

A

mild/ uncomplicated: supportive measures and close follow-up

severe/ fulminant heptatic failure, immunosuppressed pt, or concurrent hep C: lamivudine

1091
Q

What is the risk of progression from acute hepatitis B infection to chronic hepatitis B infection?

A

perinatally acquired: 90%

pt aged 1-5 years old : 20-50%

adult: 5% or less

1092
Q

What is the best prognostic indicator for patients with acute hepatitis B?

A

prothrombin time (PT)

if consistently normal, then likely infection will resolve without significnat sequalae

1093
Q

A pt presenting with non-throbbing headaches with bilateral localization occurring mainly on weekdays and no associated with nausea/ vision problems/ neuro deficits most likely suffers from ….

A

Tension headaches

1094
Q

What is the most common source of bleeding in a pt with diverticulosis?

A

erosion of the artery (due to fecalith in the diverticular sac)

1095
Q

What is a common side effect of idinavir therapy for HIV pts?

A

crystal-induced nephrotpathy and nephrotoxicity

1096
Q

What is a common life-threatening side effect of didanosine used for HIV therapy?

A

pancreatitis

1097
Q

What is a common life threatening side effect of nevirapine used for HIV therapy?

A

liver failure

1098
Q

A young athlete who experiences exertional chest pain followed by presyncope/ syncope during exertion and then has sudden cardiac death with a normal EKG/ ECHO and without a family history of sudden death most likely suffers from ..

A

Anomalous origin of the coronary artery

1099
Q

A pt presenting with a faint skin rash over the upper body that worsened in the summer time and is associated with mild pruritis most likely suffers from ..

A

Tinea Versicolor

tx: topical anti-fungal- selenium/ sulfide/ terbinafine/ clotrimazole/ ketoconazole

1100
Q

What is the Centor criteria and when does it suggest further diagnostic testing?

A
  1. tonsillar exudate
  2. tender anterior cervical lymphadenoapthy
  3. fever
  4. absence of cough

(use to determine likelihood of streptococcal pharyngitis; if two or more criteria then rapid strep test)

1101
Q

A pt presenting wtih dull back pain, urinary retention, decreased sensation, progressive weakness of the lower extremities with associated muscle flaccidity and hyporeflexia that then can progress to spasticity and hyperreflexia following a upper respiratory infection most likely suffers from …

A

Transverse Myelitis

1102
Q

What is the standard caloric intake recommendation for enteral feeding (via gastrostomy tube)?

A

30 kcal/ kg/ day with 1 g/kg/day of protein

if pt is severely malnourished, then lower rate to prevent refeeding syndrome

1103
Q

What is the next best step in management of a pt who is found to have asymptomatic congestive heart failure (i.e cardiac dilatation and low ejection fraction) on ECHO?

A

perform cardiac stress test

to assess for likelihood of coronary artery disease since most common cause of heart failure

1104
Q

What is the next best step in management of a pt who develops angioedema (facial swelling with prominent lip and tongue swelling) while on ACE inhibitor treatment?

A

switch to Angiotension Receptor Blocker (ARBs)

(ACE inhibitors prevent breakdown of bradykinin leading to angioedema; ARBs do not affect bradykinin metabolism)

(avoid ACE inhibitor use in future if ever had angioedema effect)

1105
Q

What is the most common electrolyte disturbance associated with TURP (transurethral resection of prostate) for benign prostate hypertrophy?

A

hyponatremia (due to use of isosomtic flushing solutions lacking sodium and containing glycine, sorbitol and mannitol)

1106
Q

What three drugs an interfere with folate metabolism and lead to folic acid deficiency macrocytic anemia?

A
  1. methotrexate
  2. phenytoin
  3. trimethoprim
1107
Q

What is the best treatment for methotrexate induced folate deficiency macrocytic anemia?

A

Folinic acid

more potent, bypasses the blocked dihydrofolate reductase

1108
Q

What is the strongest risk factor for pelvic inflammatory disease (PID- fever, abdominal pain, cervical discharge, cervical motion and adnexal tenderness, presence of gonorrhea/ chlamydia)?

A

Multiple sexual partners

1109
Q

A obese female pt presents with menstrual irregularity, hirsuitism, increased ovariam volume measured by ultrasonography, and problems conceiving most likely suffers from ..

A

Polycystic Ovarian Syndrome (Stein-Leventhal syndrome)

tx: weigh loss; if fails, then try clomiphene citrate if want to get pregnant

1110
Q

What is the best treatment regimen for metastatic prostate cancer (new onset or worsening bone pain in setting of hx of prostate cancer, osteoblastic lesins seen on radionuclide bone scan)?

A

flutamide (anti-androgen) for 1 week followed by Leuprolide (LH-releasing hormone)

(flutamide blocks the effects of the initial testosterone surge that occurs with LHRH)

1111
Q

What is the next best step in management of a pt presenting with a hernia (dull sensation of heaviness/ discomfort that is more pronounced with standing/ straining and relieved by laying flat)?

A

Surgical hernia repair

if left untreated, increase risk of incarceration and strangulation of bowel

1112
Q

What is the next best step in management of a pt with status epilepticus (single unremitting seizure lasting longer than 5-10 minutes or multiple seizures without interictal return to baseline)?

A

IV benzo (lorazepam or diazepam) after ABC’s

second line: barbituates in kids; phenytoin/ fosphenytoin in kids and adults

1113
Q

A child presenting with refusal to move arm and arm is extended and pronated, attempts to supinate arm cause child to cry and there is no swelling/ deformity/ focal tenderness after a child is pulled/lifted or swung by arm most likely suffers from …

A

Radial Head Subluxation (nursemaid’s elbow/ annular ligament displacement)

(tx: hyperpronation of forearm; supination of forearm and flexion or elb- second line)

1114
Q

A pt presenting with depressed respiratory rate (usually less than 10) with shallow breaths and crackles (from basal atelectasis), small pupils (miosis), and drowsiness (decreased mental status) along with decreased bowel sounds, bradycardia and respiratory acidosis with a history of chronic pain (along with renal/liver injury) most likely suffers from …

A

Opioid Intoxication

tx: naloxone- repeated doses; airway managment and ventilation, cardiac monitoring

1115
Q

What is the next step in management of a neonate who is found to have low total T4 and elevated TSH on neonatal screening for hypothyroidism using heel pad?

A

Repeat free T4 and TSH levels from regular blood draw

1116
Q

What is the best step in management of renal stone measuring less than 10 mm without signs of urosepsis/ acute renal failure or complete obstruction?

A

medical management with IV hydration, pain control, alpha blocker, and discharge pt home once symptoms controlled

(if uncontrolled pain, no stone passage in 4-6 weeks or stone size of 10 mm or more, then urology consult for stone removal)

1117
Q

When can a diabetic pt being treated for diabetic ketoacidosis or hyperglycemic hyperosmolar nonketotic state be switched to subcutaneous (basal bolus ) insulin?

A

if able to eat, glucose less than 200 mg/dl, anion gap less than 12, and serum HCO3 greater than or equal to 15

(overlap subcutaneous and IV insulin by 1-2 hours)

1118
Q

A pt presenting with widespread musculoskeletal pain, cognitive/ mood disturbances and fatigue for 3 months or more, and symmetrical tender points over muscles and bony prominences with normal inflammatory markers (ESR, CRP) most likely suffers from …

A

FIbromyalgia

1119
Q

What diagnostic tests should be performed in a pt with suspected fibromyalgia? (4)

A
  1. CBC (rule out anemia)
  2. inflammatory markers (rule out inflammatory arthropathy)
  3. TSH (rule out hypothyroidism)
  4. can also do creatine kinase (rule out myosisitis)
1120
Q

What is the next best step in management of anterior epitaxis (nosebleeding invoving the Kiesselbach’s plexus) after bleeding fails to cease with nostril pinching of nasal alae?

A

Topical vasoconstricor (oxymetazoline in kids) on cotton pledget or using squirt bottle placed on nasal septum followed by nostril pinching for another 5-10 minutes

1121
Q

An immigrant (especially from Africa) presents with persistent urinary symptoms (painful urination and increased urinary frequency) along with anemia and microhematuria most likely suffers from …

A

Schistosomaiasis

dx: urine microscopy showing parasite eggs

1122
Q

What is the next best step in management for a woman with negative/ normal cytology on Pap smear with no endocervical/ metaplastic cells seen on smear if pt is aged 21-29? and aged 30 or older?

A

aged 21-29: routine screening (pap smear in 3 years)

aged 30 or older: HPV testing

1123
Q

What is the treatment for acne vulgaris that is comedonal acne (closed or open comedones on forehead, nose and chin)?

A

water-based cosmetics, pH neutral detergent cleansers, topical retinoids

(alternative: topical salicylic/ azelaic/ glycolic acid)

1124
Q

What is the treatment for acne vulgaris that is inflammatory acne (inflamed papules less than 5 mm in size and pustules with erythema)?

A

mild: topical retinoids and benzoyl peroxide
moderate: add topical antibiotics (erythromycin/ clindamycin)
severe: add oral antibiotics

1125
Q

What is the treatment for acne vulgaris that is nodular/ cystic acne (large/ greater than 5 mm nodules that appear cystic and may merge to form sinus tracts with scarring)?

A

moderate: topical retinoid plus benzoyl peroxide plus topical antibiotic
severe: add oral antibiotic

unresponsive severe: oral isotretinoin

1126
Q

When should a child be screened for hypercholestrolemia and what is the screening process?

A

after child turns 2 years old, if have family member with high blood cholesterol or premature coronary artery disease

  1. if family member has total cholesterol greater than 240 mg/dl, then perform random cholesterol (if less than 170, repeat in 5 years; if greater than 200, order fasting lipid panel)
  2. if family member has hx of premature coronary artery disease, order fasting lipid panel
1127
Q

If the relative risk of an outcome in group A as compared to group B is x, then the relative risk in group B as compared to group A is …

A

1/x

inverse of relative risk of group A compared to group B

1128
Q

An IV drug user or pt with multiple unprotected sexual encounters who presents with fever, fatigue, maculopapular rash on the face, trunk, and extremities, headaches, lymphadenopathy, pharyngitis, myalgias, arthralgias, GI symptoms, night sweats, and/or oral thrush most likely suffers from …

A

Primary HIV infection (acute retroviral syndrome)

dx: HIV RNA or p24 antigen levels; ELISA may be negative initially

1129
Q

What is the best initial treatment for a pt with benign prostatic hyperplasia (BPH- intermittent urinary frequency and hesitancy, diffuse smooth and enlarged prostate)?

A

alpha blocker (terazosin, tamsulosin, doxazosin)

side effect: orthostatic hypotension dizziness

1130
Q

A pt presenting with hyperthyroid symptoms (palpitations, restlessness, menstrual irregularities, irregular tachycardia, tremor,enlarged thyroid gland) with an elevated TSH, elevated T3 and T4 and elevated alpha subunit most likely suffers from …

A

TSH secreting pituitary adenoma

if alpha subunit was low and show signs of hypothyroid, then thyroid hormone resistance syndrome

1131
Q

A pt presenting with erythematous papules and vesicles that are distributed in a linear fashion and intensely pruritic after being outside most likely suffers from …

A

Poison Ivy Dermatitis

1132
Q

What is the next best step in management of a postpartum pt who is known to be Rh negative (D negative) after a positive rosette test (qualitative test that determines presence of feto-maternal hemorrhage)?

A

Kleihauer-Betke stain or fetal red cell stain using flow cytometry

(to determine amount of hemorrhage that occured so dose of anti-D immun globulin can be corrected)

1133
Q

Applying population level infroamtion to an individual level may lead to …

A

Ecologic fallacy

can not make individual-level conclusions from an ecologic/ correlational study- population level information

1134
Q

What modifiable intervention would most benefit in decreasing the risk of developing osteoporosis?

A

smoking cessation

1135
Q

What is the most common complication associated with anal abscesses?

A

fistula formation

(anal abscess that persists after incision and drainage or pustule like lesion in perianal/ ischiorectal area that continues to drain)

(usually treat surgically)

1136
Q

What is the initial step in management of a pt with suspected delayed gastric emptying (i.e. diabetic with gastroparesis)?

A

Upper GI endoscopy (to rule out mechanical obstrcution)

then perform scintigraphic gastric emptying study/ gastroduodenal manometry

1137
Q

What is the most likely complication of untreated varicocele?

A
Testicular atrophy  
(dilation of pampiniform plexus leads to rise in intrascrotal temperature that causes seminiferous tubules in testicle to atrophy)
1138
Q

What is the next best step in management for a male pt who has bilateral varicocele, right varicocele or varicocele that does not disappear in the supine position?

A

Abdominal CT (to assess for inferior vena cava obstruction)

1139
Q

What is the best initial step in management of an elderly pt who develops delirium?

A

recheck vital signs and pulse oximetry

(to assess for early signs of infection or fluid and electrolyte disturbances)

(followed by urinalysis, CMP, liver function tests, calcium, CBC)

1140
Q

What is the best management of choking due to complete obstruction (hands clutched at throat, inability to talk, difficulty breathing, unable to cough, cyanosis) in a child less than 1 years old?

A

turn the child face down/ head down and administer 5 back blows; then turn child face up and administer 5 chest thrusts

1141
Q

What is the best management of choking due to complete obstruction (hands clutched at throat, inability to talk, difficulty breathing, unable to cough, cyanosis) in a child 1 year or older?

A

lean child forward and perform abdominal thrusts or alternate between 5 back blows and 5 abdominal thrusts

1142
Q

What screening tests are used to diagnose diabetes mellitus? (4)

A
  1. Hemoglobin A1C greater than or equal to 6.5%
  2. fasting blood glucose greater than or equal to 126 (no caloric intake for 8 hours prior to test)
  3. random glucose of 200 mg/dl or greater with symptoms (polyuria, polydipsia, polyphagia)
  4. oral glucose tolerance test of 200 mg/dl or greater
1143
Q

A pt with the appearance of a female who presents with primary amenorrhea, bilateral inguinal masses, breast development without pubic/axillary hair, lacks a uterus and fallopian tubes most likely suffers from ..

A

Androgen Insensitivity

46 XY male with femlae phenotype

1144
Q

What is a major long term complication of chronic proton pump inhibitor use?

A

increase risk of osteoporosis and hip fractures

1145
Q

A pt presenting confused and disoriented with generalized abdominal pain, increase thirst, ketonuria/ ketonemia, anion gap metabolic acidosisand mildly elevated glucose levels most likely suffers from …

A

Alcoholic Ketoacidosis

tx: IV dextrose containing normal saline and thiamine and hospitalize

1146
Q

What are common complications of respiratory syncytial virus bronchiolitis? (2)

A
  1. concurrent acute otitis media

2. future development of reactive airway disease (recurrent wheezing)

1147
Q

What is the best treatment for multiple brain metastases?

A

Whole brain radiation therapy

improves survival to 3-6 months

1148
Q

What is the best next step in management of a pt with a lung mass determined to be lung cancer after cytology is performed?

A

chest CT scan (to accurately stage cancer before determining most appropriate treatment)

1149
Q

What is the most common and earliest complication associated with sickle cell disease in young children?

A

splenic sequestration

presents as splenomegaly and acute worsening of anemia

1150
Q

What are the two best contraceptive options for women with cyanotic heart disease (i.e. Eisenmenger syndrome)?

A
  1. hysteroscopic sterilization (avoid need for general anesthesia)
  2. progestin only contraception (avoid combined OCPs due to risk of thromboembolism)
1151
Q

What is the next best step in management of a pt being treated for diabetic ketoacidosis whose blood glucose drops below 200 md/dl but continues to have a persistently elevated anion gap (greater than 13)?

A

decrease insulin infusion by half and add 5% dextrose to IV fluids (to prevent hypoglycemia)

(use 0.9% NS if corrected serum Na less than 135; use 1/2 NS if corrected serum Na is 135 or greater)

1152
Q

What is the calculation for corrected serum sodium in the setting of diabetic ketoacidosis?

A

corrected serum Na = measured Na ( measure Na + 0.016 {measured glucose -100])

(use 0.9% NS if corrected serum Na less than 135; use 1/2 NS if corrected serum Na is 135 or greater)

1153
Q

A pt who develops respiratory insufficiency (tachypnea, low oxygen saturation, SOB), neurological impairment (confusion), and petechial rash on trunk 24-72 hours after a severe trauma resulting in fractures most likely suffers from …

A

Fat Embolism

prevent: early immobilization and operative fixation of fracture
(tx: supportive care)

1154
Q

… hernia is related to failure of the processus vaginalis to obliterate ; whereas, … hernia is due to muscular weakness of the abdominal wall

A

indirect; direct

1155
Q

What is the best management of asystole and PEA (pulseless electrical activity- organized rhythm on cardiac monitoring without a measurable BP or palpable pulse)?

A

CPR (for 2 minutes then recheck for pulse/ rhythm) with epinephrine every 3-5 minutes

(repeat as long as continues to be asystole/ PEA and attempt to treat reversible causes)

1156
Q

An immigrant child presents with low grade fever, conjunctivityis (red eyes), coryza (inflammation of mucoas), posterior cervical and occipital lymphadenopathy, petechiae on soft palate (Forschheimer spots), and blanching erythematous maculopapular rash that starts on head and spreads downward sparing the palms and soles most likely suffers from …

A

Rubella (German Measles)

(dx: PCR; anti-rubella IgM and IgG)

(adolescents/ adults can also have arthalgia/ arthritis)

1157
Q

An infant with sensorineural hearing loss, intellectual disability, cardiac abnormalities, cataracts and glaucoma msot likely suffers from …

A

Congenital Rubella

1158
Q

What is the most common cardiac abnormality associated with Down’s syndrome?

A

Endocardial cushion defect

also associated with duodenal atresia, Hirschsprung’s disease, atlanto-axial instability, hypothyroidism

1159
Q

What is a pt with Down’s syndrome likely to develop later in life? (2)

A
  1. Acute leukemia

2. Alzheimer’s like dementia

1160
Q

A pt presenting with a history of recurrent episodes of bronchitis with productive sputum, upper respiratory infections, sinus infections, wheezing, diarrhea and steatorrhea in the setting of a positive family history most likely suffers from …

A

Cystic Fibrosis

dx: sweat chloride test- if concentration greater than 60 mEq/L

1161
Q

What is the next best step in management of a pt presenting with hematuria after extensive exercise (marathon, contact sport) in the absence of red blood cell casts (only presence of intact RBCs)?

A

repeat urinalysis in 1 week (to assure it has resolved)

likely due to exercise- induced hematuria

1162
Q

What is the next best step in management of a pt with suspected catheter-associated infection leading to severe sepsis with organ hypoperfusion/ hemodynamic instability/ endocarditis, suppurative thrombophlebitis?

A

catheter removal and start antibiotics (vancomycin and cefepime) after starting IV fluids and taking blood cultures

1163
Q

A pt who develops low back pain, local tenderness to spinal percussion, reduced back mobility, and spasm of nearby muscles while being treated for Staph aureus infection of blood most likely suffers from …

A

Vertebral Osteomyelitis

dx: MRI of spine

1164
Q

A pt presenting with painless rectal bleeding and is found to have aortic stenosis (ejection systolic murmur radiating to carotids) on physical exam or end stage renal disease most likely suffers from …

A

Angiodysplasia (vascular ectasia/ angioectasia)

1165
Q

An obeses pt with history of smoking presents with a painful nodule in an intertriginous area (axilla, inguinal, genital, perianal, perineal) that can then progress to an abscess with purulent or serosanguineous drainage and over time multiple nodules can lead to sinus tracts, comedones and scarring most likely suffers from …

A

Hidradenitis Supprative

(tx: weight loss, smoking cessation, skin cleansing daily)
(stage 1/ mild: topical clindamycin, intralesional steroids)
(stage 2/ inflammatory nodules, sinus tracts, scarring: oral tetracycline antibiotic)
(Stage 3/ severe and refractory: TNF alpha inhibitors and/or oral retinoids, surgical excision)

1166
Q

What is the next best step in management of a pt with symptomatic (syncope, dizziness) Mobitz type 1 second degree AV block (progressive prolongation of PR interval leads to non-conducting P wave- group beating) or symptomatic/ non-symptomatic Mobitz type 2 second degree AV block (PR interval remains constant with intermittent nonconducted P waves)?

A

permanent pacemaker placement

use temporary tranvenous/ transcutaenous external pacemaker initially if severe symptoms or hemodynamically unstable

1167
Q

A pt who presents with motor tics (facial grimacing/ blinking/ shoulder shrugging/ tongue protrusion/ sniffing/ head or neck jerking) and vocal tics (grunts, snorts, throat clearing, barking, yelling, coprolalia- yelling obscenities) that presented before the age of 18 years old, with the tics occurring multiple times a day for at least 12 months most likely suffers from ..

A

Tourette’s Syndrome

(strong association with obsessive compulsive disorder/ OCD and attention deficit hyperactivity disorder)

(tx: antipsychotics, alpha agonists- clonidine/ guanfacine, tetrabenazine- dopamine depleter, habit reversal training- behavioral therapy)

1168
Q

What is the next best step in management of a woman aged 25 years old or less or already pregnant who is found to have cervical intraepithelial neoplasia grade 2 (CIN 2) confirmed by colposcopy with biopsy?

A

observation and expectant management (cytology and colposcopy every 6-12 months for 1 year)

(CIN2 is more likely to regress in pt less than 25 and high risk of cervical insufficiency/ stenosis with treatment of ablation/ excision of transformation zone)

1169
Q

What is the black box warning issued by the FDA for atypical antipsychotics (olanzapine)?

A

Elderl pts with dementia related psychosis treated with antipsychotic drugs are at an increased risk of death (cardiovascular and infection)

(however its use may provide more benefits than harm)

1170
Q

An immigrant pt from mexico/ central or south america with history of recent fever and myalgias presenting with palpitations, lightheadness and found to have left ventricular apical aneursym without coronary artery disease, biventricular heart failure (right worse then left) with cardiomegaly, mural thrombosis with embolic complications, conduction abnormalities and progressive dilation of esophagus and colon most likely suffers from ….

A

Chagas Disease (due to Trypanosoma cruzi- protozoa)

most common cause of dilated cardiomyopathy in central and south america

1171
Q

What are the common side effects of combined oral contraceptives? (2)

A
  1. break through bleeding
  2. breast tenderness

(ring causes vaginal irritation and discharge; patch causes skin irritation and rash)

1172
Q

What are common side effects of depot medroxyprogesteone acetate (depot)? (4)

A
  1. initial irregular bleeding
  2. short term reversible bone mineral density loss
  3. possible delayed return to fertility
  4. possible weight gain
1173
Q

What are three ECHO findings used to diagnose early cardiac tamponade and the need for immediate pericadiocentesis?

A
  1. right atrial and ventricular collapse during diastole
  2. exaggerated respiratory variation of cardiac and venous flows
  3. IVC plethora (dilation and less than 50% decrease IVC size during inspiration)
1174
Q

A pt presenting with jugular venous distension, hypotension, diminished heart sounds, pulsus paradoxus (large inspiratory decrease in sBP of greter than 10 mmHg), low voltage QRS complexes and electrical alternans on EKG, and enlarged cardiac silhouette, water bottle shaped heart and clear lung fields on CXR most likely suffers from …

A

Cardiac Tamponade

(tx: immediate pericardiocentesis)

(can be due to cancer, radiation therapy, infection (TB/ viral/ HIV), connective tissue disorder (lupus/ RA), post cardiac surgery, uremia)

1175
Q

A … study is a type of experimental study design that utilizes 2 or more interventions and all combinations of these interventions

A

factorial (fully crossed)

1176
Q

A pt presenting with hypopigmented spots on their body with a personal or family history of bilateral deafness most likely suffers from …

A

Neurofibromatosis type 2

1177
Q

What is the best diagnostic tool to assess the cause of an upper GI bleed (hematemesi, melena, hemodynamically unstable, orthostasis, BUN to creatinine ratio greater than 20:)?

A

Upper GI endoscopy

diagnostic and therapeutic with photocoagulation and local injection of vasoconstrictor agent

1178
Q

A pt with history of atherosclerotic disease presenting with abdominal pain followed by bloody diarrhea and associated elevated white cell count in which X-rays and sigmoidoscopy shows mucosal edema and mucosal ulcerations most likely suffers from …

A

Acute Ischemic Colitis

usually affects splenic flexure and rectosigmoid junction

1179
Q

A pt presenting with fever, malaise and abrupt onset of facial rash described as red, painful, edematous and elevated with sharp demarcation from uninvolved skin present on the cheeks and bridge of the nose most likely suffers from …

A

Erysipelas

due to group A step mostly

1180
Q

What is the next best step in management for a pt with a solitary pulmonary nodule on CXR after comparing to previous CXR?

A

Chest CT (if no previous imaging, new mass, or growing mass)

no further testing if stable lesion over 2-3 years when compared to previous imaging

1181
Q

What are the criteria for high risk of malignancy for a solitary pulmonary nodule and what is the next best step after chest CT?

A
  1. nodule size 2.0 cm or greater
  2. age older than 60
  3. current smoker
  4. smoking cessation less than 5 years ago
  5. corona radiata or spiculated nodule margins

surgical excision (video assisted thoracoscopic surgery)

1182
Q

What is the best diagnostic tool to assess the cause of a lower GI bleed (hematochezia- bright red blood per rectum, hemodynamically stable, normal BUN to creatinine ratio)?

A

Colonoscopy

1183
Q

What is the next best step in management of diabetic pt on oral hypoglycemics who develops renal failure?

A

stop the use of metformin and sulfonylureas; and start insulin

(can continue “azones”, acarbose, “glinides”)

1184
Q

What is the best treatment for post-herpetic neuralgia (pain or other sensory symptoms lasting more than 3 months after resolution of herpes zoster skin lesions including allodynia- pain with non-painful stimuli)?

A

Tricyclic antidepressants (TCAs), topical capsaicin cream, gabapentin, long acting oxycodone

1185
Q

When are corticosteroids indicated for the treatment of pneumocystis pneumonia (PCP) along with trimethoprim-sulfamethoxazole?

A

alveolar to arterial oxygen gradient of more than 35 mmHg on room air

1186
Q

What is the next best step in management of a pregnant pt with a blood glucose of 140 mg/dl or more 1 hour after administration of 50 gram oral glucose load (routinely performed at 24-28 weeks gestation)?

A

administer 100 gram oral glucose load while checking fasting serum glucose and glucose level each hour after glucose load for 3 horus

(if 2 abnormal values, then gestational diabetes)
(abnormals: fasting- 105 mg/dl or more, 1 hour- 190 mg/dl or more, 2 hour- 165 mg/dl or more, 3 hour- 145 mg/dl or more)

1187
Q

What are the target blood glucose levels for gestational diabetes mellitus?

A

fasting: 95 mg/dl or less
1 hour postprandial: 140 mg/dl or less
2-hour postprandial: 120 mg/dl or less

(tx: dietary modification first then insulin and oral agents- metformin/ glyburide)

1188
Q

What is the next best step in management if a pt with persistently elevated calcitonin levels after total thryoidectomy for medullary thyroid cancer?

A

CT scan of neck and chest (to detect residual metastatic thyroid cancer)

1189
Q

A pt presents with fever, chills, malaise and no sign of hemolysis within 1-6 hours of a blood transfusion most likely suffers from …

A

Febrile nonhemolytic transfusion reaction

(prevent: leukoreduction)
(tx: stop transfusion, give antipyretics, use leukoreduced blood products for future transfusions)

1190
Q

A pt presenting with elevated serum calcium, elevated 24 hour urine calcium, elevated PTH, gastric ulcers with significant bleeding, and possibly signs of pituitary tumor (prolactinoma) in the setting of a positive family history for similar symptoms most likely suffers from …

A

Multiple Endocrine Neoplasia (MEN) type 1

(hyperparathyroidism, Zollinger-Ellison gastrin secreting pancreatic tumor, and pituitary tumor usually prolactinoma)
(tx: removal of 3.5 glands or total parathyroidectomy with autotransplantation- helps with hyperparathyroidism and zollinger-ellison)

1191
Q

What is the best step in management of a person who was exposed to blood from a pt with Hep B infection if the person’s HBs antibody titer is less then 10 mlU/ml?

A

give Hep B immunoglobulin and revaccinate pt with Hep B vaccine

(since low titers suggest inappropriate response to vaccination)

1192
Q

What are the indications for oseltamivir treatment for influenza? (2)

A
  1. low risk pts with mild disease presenting within 48 hours

2. high risk pt (older than 65, pregnant, pulmonary/ cardiac disease, hospitalized, lower respiratory tract invovlement)

1193
Q

What is the best initial treatment for fat malabsorption secondary to chronic pancreatitis?

A

diet modification with smaller meal that are low in fat

second line: pancreatic enzymes, opioids

1194
Q

What is a focal submandibular mass in a pt with Sjogren’s syndrome most likely due to?

A

B cell non Hodgkin’s lymphoma

due to polyclonal B cell activation and infilitration of salivary glands

1195
Q

What is the usual partition of Hb A and Hb S in a pt with sickle cell trait?

A

60% Hb A; 40% Hb S

1196
Q

A young pt presenting with unilateral weakness, hyperreflexia and MRI showing stroke in the setting of a genital ulcerative lesion and lack of family history of cerebrovascular disease or personal history of cardiovascular disease most likely suffers from …

A

Meningovascular Syphilis

dx: serum RPR and CSF VDRL
(tx: IV penicillin)

1197
Q

What is the most common complication of compartment syndrome other than possible loss of affected extremity?

A

acute renal failure (secondary to rhabdomyolysis from muscle necrosis)

1198
Q

… refers to the proportion of people with a particular condition who end up dying from the condition; whereas, … refers to the probability of dying from a particular disease in the general population

A

Case fatality rate; Mortality rate

1199
Q

A child/ adolescent is found to have a round back and on exam the postural round back is corrected by voluntary hyperextension and C-ray shows convex alignment of the thoracic spine between 20-40 degrees most likely suffers from ..

A

Flexible Kyphosis

no adverse physical effects

1200
Q

A child/ adolescent presents with aching spinal pain that is provoked by physical activity and prolonged standing, found to have thoracic curving of the pain that is not corrected with voluntary hyperextension of the spine, and there is sharp angularity with bending forward, and X-ray shows convex alignment of thoracic spine greater than 40 degrees most likely suffers from …

A

Scheuermann disease (structural kyphosis)

(mild tx- less than 70-80 degrees: Milwaukee brace and spinal muscle strengthening exercises)

(severe tx- more than 80 degrees: surgical correction)

1201
Q

What is the best treatment of pelvic inflammatory disease (abdominal pain, tenderness of adnexa on palpation, tenderness on lateralization of cervix, vaginal discharge)?

A

severe (fever, leukocytosis with bands, N/V): IV cefoxitin/ ceftriaxone with IV doxycyline

mild: IM ceftriaxone and oral doxycycline/ azithromycin

1202
Q

What is the most common microorganism isolated from a pt with necrotizing fasciitis?

A

Group A streptococcus

(tx: pipperacillin/tazobactum or carbapenem, vancomycin, and clindamycin with urgent, agressive surgical exploration)

(if crepitus, then likely clostridium perfringens)

1203
Q

What is the next best step in management for a pt with suspected pulmonary embolism (sudden onset SOB, tachycardia, hypoxia in setting of immobilization/ DVT) if Wells score is greater than 4?

A

CT pulmonary angiogram

(Wells score: clinical signs of DVT/ alternate diagnosis less likely- 3 points, previous PE or DVT/ heart rate greater than 100/ recent surgery or immobilization- 1.5 points, hemoptysis/ cancer- 1 point)

(if wells score 4 or less than, D-dimer greater than 500 suggests performing CT angiogram)

1204
Q

What abnormality is associated with a poor prognosis for a pt with a pulmonary embolism?

A

elevated cardiac troponin (if elevated past 2 days suggest MI secondary to embolism that leads to death)

1205
Q

What is the best initial step in management of a pt with confirmed pulmonary embolism?

A

low molecular weight or unfractionated heparin followed by early initiation of warfarin

(use IV heparin infusion if renal failure or possibility of bleeding)

(use fibrinolytics if persistent hypotension; use IVC filter if contraindications to anticoagulation)

1206
Q

What is the next best step in management for a child with delayed speech?

A

hearing test (audiologic assessment for possible hearing loss)

1207
Q

A pt presenting with symmetric proximal muscle weakness (difficulty rising from chair/ climbing stairs/ combing hair) in upper and lower extremities, elevated muscle enzymes (creatine kinase, aldolase, LDH), violaceous papules and plaques on dorsum of hand (grotten’s papules) and heliotrope rash on face most likely suffers from ..

A

Dermatomyositis

(associated with interstitial lung disease, dysphagia, myocarditis)

(dx: ANA, anti-ribonucleoprotein (RNP), anti-Jo1, anti Mi2; skin or muscle biopsy)
(tx: high dose steroids with steroid sparing agent; screen for cancer)

1208
Q

What are the four medications that are associated with digoxin toxicity when they are added to a pt’s medication regimen?

A
  1. verapamil
  2. quinidine
  3. amiodarone
  4. spironolactone
1209
Q

A pt presenting with monocular visual loss accompanied by eye pain that worsens with eye movement (optic neuritis), upper motor neuron signs and sensory loss below level of spinal involvement (transverse myelitis), cerebellar ataxia (dizziness, intention tremor), heat sensitivity (Uhthoff’s phenomenon), electrical sensation that runs down the back and into the limbs when the head is bent forward (Lhermitte’s sign), and minimal adduction of eye with conjugate lateral gaze (internuclear ophthalmoplegia) presenting at different times most likely suffers from …

A

Multiple Sclerosis

(dx: MRI- hyperintense lesions in brain/ spinal cord, ovoid shaped periventriuclar whhite matter lesions; CSF- oligoclonal bands and elevated IgG

(tx of exacerbation: steroids- IV if optic neuritis; plasmapheresis if steroid refractory pt)

(long term tx for relapse remitting MS: beta interferon, glatiramer acetate)

(pregnancy: minimal exacerbations, increased MS in child, increase chance of assisted delivery)

1210
Q

What is the best initial treatment for muscle spasticity associated with multiple sclerosis?

A

baclofen or tizanidine (physical therapy, stretching, massage therapy)

1211
Q

What is the best initial treatment for fatigue associated with mulitple sclerosis?

A

sleep hygiene, regular exercise, amantadine

1212
Q

What is the best treatment for postpartum endometritis (fever, uterine tenderness, foul smelling vaginal discharge, leukocytosis)?

A

clindamycin and gentamicin

(can use metronidazole with ceftriaxone/ levofloxacin but not if breastfeeding)

(most important risk factor is route of delivery- high incidence with c-sections)

1213
Q

What is the breast cancer screen guideline?

A

start mammography every 2 years at age 40-50 years old

(genetic testing if 2 first degree relative with breast cancer including 1 before age 50, 3 first or second degree affected relative; 1st or 2nd degree relative with breast and ovarian cancer; 1st degree relative with bilateral breast cancer; breast cancer in male)

1214
Q

What is the best initial treatment for active tuberculosis (TB) in pregnant woman?

A

Isoniazid (INH), rifampin, and ethambutol for 9 months

add pyrazinamide if known multi drug resistant strain

1215
Q

What is the next best step in management of a near miss event (type of medical error in which as a result of timely intervention or simply circumstance did not reach the patient and no harm occurred)?

A

report near miss to hospital administration (to prevent more dangerous outcomes in future) and can disclose to pt if there is potential for further harm or pt is aware of incident

1216
Q

What is the genetic information nondiscrimination act?

A

prohibits discrimination by health insurers and employers on the basis of genetic information (individuals genetic test, genetic tests of family members, knowledge that a family member has a genetic disorder)

1217
Q

A pt presents with acute onset of severe vomiting, watery diarrhea, fever and muscle aches while on a cruise/ in a nursing home or health care facility most likely suffers from …

A

Norovirus

dx: clincially, PCR or nucleic acid testing
(tx: supportive, resolves after 24-48 hours)

1218
Q

A pt presenting with fatigue, cold intolerance, constipation, dry skin, amenorrhea, muscle cramps., bradycardia, enlarged thryoid, decreased relaxation of ankle/ knee jerks, carpal tunnel symptoms and signs, periorbital puffiness and elevated TSH, low T4 and antimicrosomal (anti-TPO) antibodies most likely suffers from …

A

Hashimoto’s Thyroiditis (causing hypothyroidism)

(dx: antimicrosomal (anti-TPO) antibodies)

(associated with hyperlipidemia- elevated LDL and possibly elevated triglycerides; treatment with levothyroxine can help if TSH above 10)

1219
Q

A pt presenting with clinical features of central hypothyroidism (hyponatremia, borderline low TSH and low free T4, fatigue, constipation) along with borderline low 8:00 am cortisol and glucose levels most likely suffers from ..

A

Central adrenal insufficiency (along with hypothyroidism)

(dx: ACTH level and ACTH/ cosyntropin stimulation test- suboptimal increase in cortisol and low ACH level)

(central hypothyroidism should prompt assessment f other pituitary hormones as well as pituitary imaging)

1220
Q

What is the next best step in management of a pt who becomes angry and verbally abusive during interview/ discussion?

A

acknowledge the anger and express empathy (to maintain physician patient relationship)

1221
Q

A pt presenting with hypocalcemia (and associated symptoms like seizures, muscle pain/ cramps), bilateral cataracts, hyporeflexia, hyperphosphatemia, elevated PTH, and calcification of basal ganglia (Fahr’s syndrome) most likely suffers from…

A

Pseudohypoparathyroidism

(type 1A: have Albright hereditary osteodystrophy- short stature, round facies, short 4th and 5th metacarpals, sort neck)
(type 1B: lacks albright hereditary osteodystrophy)

1222
Q

A pt with a history of inflammatory disease (rheumatoid arthritis) presenting with signs and symptoms of anemia (fatigue, pallor) i which iron studies show low serum iron, elevated ferritin levels and normal to low transferrin saturation most likely suffers from …

A

Anemia of CHronic Disease

(tx: treat underlying disease)

(add erythropoietin if epo level is low or inappropriately normal; epo level should be high)

1223
Q

An infant born to a mother who ate raw/ uncooked meat durnig pregnancy and is found to have microcephaly, hydrocephalus, corioretinitis, mental retardation, deafness and seizures most likely suffers from …

A

Congenital Toxoplasmosis

also if mother in contact with cat feces

1224
Q

A pt with history of eczema/ healthcare worker with frequent glove use presenting with rash of the skin where it is thinnest (dorsum of hands, fingertips, finger webs) that is pruritic, erythematous and scaling, which then can progress to hyperkaratosis and painful fissuring most likely suffers from …

A

Irritant Contact Dermatitis

(tx: avoid offending agent, wash hands with lukewarm water and mild cleansers, use emollients frequently, topical steroids)

1225
Q

A pt with severe hypertension which is resistant to 2 medications and has acutely worsened from previously stable state or with malignant hypertension in the setting of atherosclerotic disease (PAD, carotid artery, CAD) most likely suffers from …

A

Renovascular hypertension

(dx: doppler ultrasound of renal arteries, MR angiogrpahy, CT angiography)

(associated with elevated creatinine, unilateral renal atrophy, recurrent flash pulmonary edema, abdominal bruit)

1226
Q

What is the next best step in management for a pt who is undergoing an orthopedic procedure (hip replacement or knee replacement) in the setting of risk factors such as old age, history of DVT/ malignancy, obesity, inherited hypercoaguable state?

A

anticoagulation for DVT prophylaxis starting before surgery and continuing until pt ambulatory

(low molecular weight heparin is best)

1227
Q

… occurs when the survival benefits of a screening test are overstated due to detection of a disproportionate number of slowly progressive, benign cases

A

Length time bias

1228
Q

An unimmized pt presents with fever, difficulty breathing, sore throat, difficulty swallowing, has a muffled voice, toxic appearance while leaning forward and moving around restlessly with his mouth open and tongue protruding most likely suffers from …

A

Epiglottis (due to H. influenza type B)

tx: transfer to hospital via ambulance and intubate; give antibiotics

1229
Q

An infant presents with a rash that developed shortly after a mild diarrheal illness and is a erythematous, peeling rash that involves the gluteal region and perineum most likely suffers from …

A

Diaper Dermatitis (Diaper rash- irritant contact dermatitis)

(tx: frequent diaper changes, avoid tight fitting diapers, expose skin to air, use diapers with super absorbent surfaces, apply barrier cream- zinc oxide or pertrolatum)

1230
Q

What are the major side effects associated with the use of amiodarone (which require monitoring)?

A
  1. thyroid dysfunction (TSH, T4, T3)
  2. hepatotoxicity (liver function test)
  3. pulmonary toxicity (PFTs; most serious; deaths)

(also leads to photosensitivity, skin discoloration, bone marrow suppression)

1231
Q

A pt presents with nonproductive cough, fever, pleuritic chest pain, weight loss, dyspnea on exertion, and focal/ diffuse interstitial opacity on CXR in the setting of chronic (months to years) use of amiodarone at high doses (more than 400 mg/day) most likely suffers from ..

A

Chronic interstitial pneumonia

tx: discontinue amiodarone therapy, steroids if severe or life threatening

1232
Q

A pregnant pt or postpartum pt who presents with hypertension, hemolytic anemia (shistocytes on smear), elevated liver transaminases, hepatomegaly, thrombocytopenia, and proteinuria most likely suffers from…

A

HELLP syndrome (hemolytic anemia elevated liver enzymes, low platelets)

(tx: magnesium sulfate- to reduce risk of seizures; deliver baby if at term)

1233
Q

What is the best treatment for gastoenteritis secondary to Giardia lamblia?

A

Metronidazole

alternatives: nitazzoxanide, tinidazole
(only for symptomatic pts; not carriers)

1234
Q

What is the next best step in management of an adult pt presenting with history of recurrent sinopulmonary and gastrointestinal infections?

A

quantitative measurement of serum immunoglobulin levels

to assess for immunodeficiency of humoral immunity

1235
Q

What is the best definitive therapy for severe Grave’s disease in a pt from united states?

A

radioactive iodine ablation with prednisone

(prednisone prevents worsening of ophthalmopathy)

(use total thyroidectomy if large obstructive goiter, possible thyroid cancer, co-exisiting primary hyperparathyroidism, pregnant pts who cant tolerate meds)

1236
Q

What is the next best step in management of a pt who underwent radioactive iodine ablation with prednisone for hyperthyroidism?

A

assess thyroid function with total T3 and free T4 levels

1237
Q

What are the indications for hospitalization for treatment of anorexia nervosa (BMI less than 18.5, intense fear of weight gain, distorted views of body weight and shape)? (5)

A
  1. dehydration
  2. arrhythmis/ severe bradycardia
  3. acute food refusal
  4. severe malnutrition
  5. suicidal ideation/ psychosis
1238
Q

A pt who is restarted on feeding after an extended period of poor oral intake develops hypophosphatemia, hypokalemia, hypomagnesemia, heart failure (pulmonary edema, peripheral edema, orthopnea, progressive dyspnea), arrhythmia, seizures, respiratory failure and wernicke encephalopathy (thiamine deficiency) most likely suffers from ..

A

Refeeding syndrome

(due to sudden fluid and electrolyte shifts)

(tx: phospahte, potassium and magnesium replacement; monitor electrolytes while increasing caloric intake slowly; give thiamine before refeeding starts)

1239
Q

An HIV pt with CD4 cell count less than 50 presenting with confusion, lethargy, decreased strength who is found to have a well-defined focal lesion in the brain most likely suffers from …

A

Primary CNS Lymphoma

(tx: HAART therapy, steroids and radiation therapy)

(prognostic factor: increasing CD4 count)

1240
Q

What diagnostic tool is used to determine the etiology of a pt’s acute decompensated heart failure?

A

ECHO (to assess for ventricular dysfunction and valvular disease)

(most common cause of acute decompensated heart failure is LV systolic or diastolic dysfunction)

1241
Q

What is the next best step in management of a pt with left ventricular dysfunction (i.e LV dilation with low ejection fraction) who is asymptomatic?

A

ACE inhibitor (to slow progression into symptomatic heart failure and prolong survivial)

(start beta blocker as well)

1242
Q

What is the next best step in management of a health care worker who experiences occupational exposure (needlestick) while caring for a pt with HIV?

A

post-exposure prophylaxis (involving immediate skin washing and starting antiretroviral therapy)

(if pt has low viral load / asymptomatic: 2 nucleoside reverse transcriptase inhibitors for 4 weeks)
(if pt has high viral load: 2 nucleoside reverse transcriptase inhibitors with a protease inhibitor for 4 weeks)

1243
Q

What are the three prophylactic treatments that HIV patients should receive and when are they indicated?

A
  1. Hep B vaccine (all HIV pts)
  2. PCP prophylaxis with TMP/SMX (if CD4 less then 200)
  3. MAC porphylaxis with azithromycin/ clarithromycin (if CD4 less than 50)
1244
Q

When is complementary and alternative medicine tolerated in a pt? (2)

A
  1. not inherently harmful

2. will not displace effective conventional care

1245
Q

A pt develops tongue deviation to one side following a carotid endartectomy most likely suffers from …

A

Hypoglossal nerve damage

(change in voice quality suggests recurrent laryngeal nerve damage)
(assymetric smile suggest mandibular branch of facial nerve damage)

1246
Q

… is when the exposure-disease relationship is obscured by the effect of an extraneous factor that is associated with both the exposure and disease

A

Confounding bias

(statistical significance when not taking into account confouding factor, but loss of significance when confounding is taken into account)

1247
Q

What is the next best step in management of a pt who was treated for a molar pregnancy with suction evacuation?

A
  1. contraception (to prevent pregnancy for 6-12 months)

2. weekly quantitative beta HCG levels (to assess for effective treatment)

1248
Q

What is the next best step in management of a hyperthyroid pt with symptomatic tachycardia/ palpitations/ atrial fibrillation?

A

beta blockers (to decrease sympathetic activity associated with elevated thyroid hormones) until pt become euthyroid with anti-thyroid meds or radioiodine

1249
Q

What is the next best step in management of a pt with pulmonary embolism who develops fever, leukocytosis and has streaky opacity on CXR?

A

obtain blood cultures (wait to start antibiotics b/c fever and leukocytosis could be due to body’s reaction to PE)

1250
Q

What are the recommended routine vacciantions for adults (65 years old or older)?

A
  1. annual influenza vaccine
  2. tetanus-diphtheria (td) booster every 10 years with one being Tdap (tetanus-diphtheria-pertussis)
  3. 13- valent pneumococcal conjugate vaccine (PCV13) followed by 23- valent pneumococcal polysaccharide vaccine (PPSV23) 6-12 months later
1251
Q

A pt presents with high fever that rapidly resolves and is followed by eruption of rosy non-pruritic rash originating on the trunk and speading to the extremities most likely suffers from …

A

Roseola

1252
Q

An unimmunized pt presents with cough, conjunctivitis, coryza (inflammation of nasal mucosa), fever, photophobia, blue-white Koplik spots on buccal mucosa followed by maculopapular rash that starts on the face and spreads downward most likely suffers from …

A

Rubeola (Measles)

1253
Q

What is the next best step in management of a pt who has recent significant exposure to tuberculosis pt?

A

tuberculin skin test

if initial test is negative, repeat test 10 weeks after last known exposure

1254
Q

Other then routine medications given to pts with chest pain, what is the next best step in management of a pt with cocaine related chest pain?

A

benzos (IV lorazepam to decrease anxiety and agitation that can promote cardiac complications)

1255
Q

What is the next best step in management of persistent hypertension after cocaine use if hypertension does not respond to benzos?

A

phentolamine (alpha agonist)

alternatives: nitroglycerine, nitroprusside

1256
Q

What are adolescent pregnant women at high risk of? (3)

A
  1. perinatal mortality
  2. preterm delivery
  3. premature and low birth weight infants

(no increase risk of congenital malformations)

1257
Q

A pt presenting with medullary thyroid cancer (elevated calcitonin), pheochromocytoma (eleavted plasma free metanephrine), and hyperparathyroidism (due to parathyroid hyperplasia) with associated mutation in RET proto-oncogene most likely suffers from …

A

Multiple Endocrine Neoplasia type 2A

1258
Q

A pt presents with chest pain and dyspnea on exertion, hypertension, abdominal pain, anorexia, hepatomegaly, prominent S2 on auscultation, prominent pulmonary arteries on CXR, and right axis deviation on EKG most likely suffers from ..

A

Pulmonary artery hypertension

(dx: ECHO- showing RV hypertrophy and failure, valvular incompetence, right atrial size, increased right sided pressures)

(evaluate for causes such as left side heart failure, pulmonary disease, chronic thromboembolism disease, obstructive sleep apnea)

1259
Q

What is the next best step in management of a pt with idiopathic pulmonary artery hypertension (not due to left heart failure, pulmonary disease, thromboembolism, obstructive sleep apnea)?

A

vasoreactivity test

(measure pulmonary artery pressure in response to vasodilator with right heart catheter)
(if respond: tx with calcium channel blocker)

(if no response: tx with epoprostenol- prostanoid, bosentan- endothelin receptor antagonist, or phosphodiesterase-5 inhibitor- sidenafil)

1260
Q

A neonate is found to have a bruise on his head shortly after birth characterized by diffuse swelling that crosses the suture lines and has dark bluish color and is associated with molding of the head and overriding of parietal bones most likely suffers from …

A

Caput Succedaneum

resolves in first few weeks of life; only tx if associated with hyperbilirubinemia

1261
Q

A neonate is found to have a bruise on the head that becomes visible hours after birth characterized by not crossing the suture lines and no discoloration of scalp and is associated with skull fracture most likely suffers from …

A

Cephalohemaotma

resolves by 2 weeks to 3 months of life; tx only if associated with hyperbilirubinemia

1262
Q

A pt presenting with hyperthyroid symptoms, low TSH, elevated T3 and T4, painful/ tender enlarged thyroid, fever, decreased radioactive iodine uptake, elevated ESR and CRP following a viral illness most likely suffers from …

A

Subacute Thyroiditis (de Quervain’s thyroiditis)

tx: NSAIDs for pain, beta blocker for hyperthyroid symptoms

1263
Q

what should be done for a child who is HIV positive and attending school?

A

can attend school without limitations (can paly sports, comingle with other kids, use standard precautions with gloves when contacting blood/ mucous membranes)

(no required disclosure to school about HIV status)

1264
Q

A pt with mild elevations of liver enzymes (AST/ALT ratio is less than 1) and alkaline phosphatase in the setting of no alcohol use or IV drug use history most likely suffers from …

A

NASH (non-alcoholic steatohepatitis)

use metformin and “azones” for diabetic control bc also helps with NASH

1265
Q

What is the next best step in the management of an infant 4 weeks old whose bowel movements have decreased from 6-8 times a day to 1-2 bowel movements a day?

A

Observation (normal change in bowel movements around 4 weeks to decreased frequency)

1266
Q

A pt develops eye rolling to back of head and neck twisting to one side after antipsychotic treatment most likely suffers from …

A

Acute Dystonic Reaction

tx: IM/ IV diphenhydramine
(alternatives: anticholinergives- benzotropine, trihexyphenidyl)

1267
Q

A presenting with acute onset flushing, throbbing headache, palpitations, abdominal cramps, diarrhea, oral burning shortly after eating fish most likely suffers from…

A

Scombroid poisoning

self limited

1268
Q

What is the likely cause of subarachnoid hemorrhage in a pt with associated findings of ptosis and aniscoria (unequal pupil size) suggesting cranial nerve 3 involvement?

A

posterior communicating artery aneurysm

1269
Q

A pt presents with small skin colored papules with central indentation occurring anywhere except the palms and soles and can have associated pruritis and erythema most likely suffers form …

A

Molluscum contagiosum

self limited; spread via direct skin to skin contact
(consider HIV testing if lesions widespread and invovle face)

1270
Q

What is the prognosis for patients who suffer from trastuzumab- associated cardiotoxicity (decline in left ventricular ejection fraction, heart failure)?

A

complete recovery of heart function after discontinue trastuzumab

1271
Q

What are the common conditions which involve the use of drugs that can causes pancreatitis? (6)

A
  1. diuretics (furosemide, thiazides)
  2. inflammatory bowel disease (sulfasalazine, 5-ASA)
  3. immunosuppressive agents (azathioprine, L-asparaginase)
  4. hx of seizures/ bipolar disorder (valproic acid)
  5. AIDS (didanosine, pentamidine)
  6. antibiotics (metronidazole, tetracycline)
1272
Q

What is the next best step in management of a blunt testicular trauma (i.e. kicked in scrotum)?

A

mild (minimal pain/ swelling and normal exam): oral analgesics, rest, follow up in 48 hours

moderate (pain and swelling on exam): ultrasound

1273
Q

A pt presenting with erythematous plaques and erosions with associated satellite papules in intertriginous areas (inguinal, perineal, genital, axillar, intergluteal, inframammary) most likely suffers from …

A

Candidal Intertrigo (due to candida albicans)

tx: topical antifungals- miconazole/ nystatin/ terbinafine

1274
Q

What is the next best step in management of a symptomatic (hypotension, dizziness, confusion, syncope, worsening of ischemia) pt with bradycardia after an inferior wall MI (leads II, III, avf)?

A

IV atropine sulfate

give IV fluid if hypotension persists

1275
Q

what is the best next step in management of a pt with priapism (undesired, painful erection lasting longer than 3 hours)?

A

ice pack application and injection with phenylephrine/ epinephrine

1276
Q

What are the four indications for antibiotic use for bacterial gastroenteritis (fever, crampy abdominal pain, bloody diarrhea)?

A
  1. invasive (w/ sepsis, osteomyelitis, meningitis)
  2. immunocompromised pt
  3. infant less than 3 months of age
  4. cholera

(otherwise treat with rehydration)

1277
Q

A pt presenting with history of periodic fever, chills, headaches, and myalgias with associated abdominal discomfort, pharyngeal erythema without exudates or adenopathy, splenomegaly, and thrombocytopenia after traveling to Africa most likely suffers from ..

A

Malaria

(prevent: prophylactic antibiotics)

(primaquine for non- P. falciparum areas; chloroquine/ hydroxychloroquine for cholorquine sensitive P. falciparum; atovaquone-proguanil/ doxycyline/ mefloquine for chloroquine resistent P. falciparum)

1278
Q

A pt presenting with a midline neck mass that moves with protrusion of the tongue most likely suffers from …

A

Thyroglossal duct cyst

(prior to surgery, imaging for normal thyroid is required b/c at times ectopic thyroid tissue in thyroglossal duct cyst is only functioning thyroid tissue)

1279
Q

What is the next best step in management of a pt with an incidental finding of an adrenal mass during abdominal imaging?

A

hormone hypersecretion and malignancy workup

(serum electrolytes, dexamethasone suppression test, 24 hour urine catecholamine, metanephrines, vanillylmandelic acid, 17-ketosteroid levels)

(surgically remove if functional, evidence of malignancy, or larger than 4 cm)

1280
Q

What is the formula for the NNH (number needed to harm; how many pts need to be exposed before a harmful even occurs in 1 pt)?

A

NNH = 1/ absolute risk increase (ARI/ attributable risk)

NNH= 1/ (incidence rate in exposed - incidence rate in non-exposed)

1281
Q

What is the best initial treatment for obsessive- compulsive disorder (obsession- recurrent intrusive anxiety provoking thoughts/ urges/ images; compulsions- response to obsessive thoughts with repeated behaviors/ mental acts with intent on reducing anxiety/ avoid dreaded outcome; cosuming more than 1 hour of day causing significant distress and interfering with daily routine/ social functioning)?

A
  1. Cognitive behavioral therapy with exposure and response prevention
  2. SSRIs (fluoxetine, sertraline, paroxetine, fluvoxamine, citalopram, escitalopram)

(alternatives: clomipramine/ antipsychotic augmentation; deep brain steimulation if severe or refractory)

1282
Q

A pt presenting with shoulder pain that is mainly lateral shoulder pain/ deltoid pain aggravated by reaching or lifting arms up with associated weakness during external rotation or abduction of shoulder most likely suffers from..

A

Rotator Cuff Tendonitis/ Tear

1283
Q

What is the next best step of management for a pt with chronic liver disease who has a recurrent upper GI bleed after an initial upper GI endoscopy with banding/ sclerotherapy?

A

repeat upper GI endoscopy with banding/ sclerotherapy

if bleeding persists, consider portal shunting with surgical shunt or TIPS procedure

1284
Q

What is the best medical treatment to reduce the risk of rebleeding in pt with esophageal varices secondary to portal hypertension (i.e. chronic liver disease)?

A

non-selective beta blocker (propanolol, nadolol)

1285
Q

what is the next best step in management of an intoxicated pt who exhibits behavior that puts him in danger of harming himself or others (demanding to be discharged despite needing medical attention)?

A

physically restrain pt and treat under implied consent doctrine

1286
Q

What is the next best step in management of a newly pregnant pt on anti-epileptic drug?

A

continue current effective anti-epileptic drug and offer early detection of major fetal anomalies (alpha fetoprotein screening, amniocentesis, ulatrasound)

(if possible switch from valproate 6 months before pregnancy; give high dose folic acid if on valproate or carbamazepine)

1287
Q

What is the next best step in management of a newborn born to a group B positive mother who did not receive antibiotic prophylaxis 4 or more hours before birth and is less than 37 weeks gestation at time of delivery or ruptured membranes longer than 18 hours?

A

obtain CBC with diff and blood culture and observe for 48 hours

1288
Q

What area of the brain is most likely affected in a pt presenting with construction apraxia (inability to copy a picture) and dressing apraxia (difficulty wearing clothes, struggle to get into coat/ pants)?

A

Nondominant parietal lobe

(dominant parietal lobe damage reesults in Gerstmann syndrome- difficulty with simple math (acalculia), inability to name individual fingers (finger agnosia), imparied writing (agraphia), and right/ left confusion (difficulty identifying/ distinguishing right or left side))

1289
Q

What is the best step in management of pain for a pt who suffers severe traumatic injury and has a history of opiate addiction?

A

narcotics (IV morphine)

(can use alternatives such as acupuncture, cold packs, hypnotherapy, trans cutaneous electrical neural stimulation and NSAIDs for mild to moderate pain)

1290
Q

What is the best initial treatment for a pt with a nonfunctioning pituitary adenoma (gonadotropin secreting cells producing low LH and low FSH but high alpha subunit with associated amenorrhea and hypogonadism)?

A

trans-sphenoidal pituitary surgery

dopamingergic agonist dont have effect on gonadotropin secreting adenomas

1291
Q

What is the next best step in management of a pt with acute ischemic stroke during the first 24 hours after negative CT imaging for hemorrhage and tPa administration to restore blood flow to ischemic area?

A

strict blood pressure control to maintain BP above 140/90 but below 185/105

(use IV labealol, nicardipine, sodium nitroprusside)

(if no tPA administered, BP can be allowed to be as high as 220/120)

1292
Q

A pt presents with atrial fibrillation, palpitations, dizziness, SOB, tachycardia, other signs of hyperthyroid, palpable thyroid nodule, low TSH, elevated free T4 and increased radioactive iodine uptake at the location of the nodule and decreased uptake in the rest of the thyroid gland most likely suffers from …

A

Toxic Nodule

1293
Q

… is defined as the applicability of a study’s results beyond the group that was initially assessed

A

External validity (generalizability)

1294
Q

What are the two most common side effects of isoniazid (INH) therapy?

A
  1. hepatotoxicity (discontinue if transaminase elevation 5 or more times the upper limit of normal or symptoms with elevation 3 or more times the upper limit of normal)
  2. peripheral neuropathy (prevent with treatment of viatmin B6/ pyroxidine)
1295
Q

What is the next best step in management of a pt with a ureteral stone reslting in proximal ureteral obstruction, hydronephrosis, acute kidney injury, superimposed infection and hemodynamic instability?

A

percutaneous nephrostomy or ureteral stent placement

1296
Q

What is the next best step in management of a pt with high grade squamous intraepithelial lesion (HGSIL) on Pap smear?

A

colposcopy or diagnostic excisional procedure (conization or loop electrosurgical excision)

1297
Q

A newborn is found to have thickened injected conjunctivae (chemosis) with watery, mucopurulent or blood stained discharge, eyelid swellings and conjunctival pseudomembrane at 5-14 days of life most likely suffers from ..

A

Chlamydial conjunctivitis

(due to direct vaginal contact during delivery)

(tx: oral erythromycin for 14 days)

1298
Q

A pregnant pt presents with fever, chills, lower abdominal pain, bloody/ purulent vaginal discharge along with a boggy and tender uterus with dilated cervix and ultrasound shows retained products of conception/ increased vascularity/ echogenic material in cavity/ thick endometrial stripe most likely suffers from …

A

Septic Abortion

(tx: blood and endometrial cx, IV fluids anf IV antibiotics, surgical evacuation)
(complication: salpingitis, peritonitis, septic shock)

1299
Q

A pt presenting with sudden onset transient visual loss in one eye most likely suffers from … and is correlated with ….

A

Amaurosis Fugax (ischemic event of retinal artery); carotid artery atherosclerosis (carotid artery bruit)

1300
Q

What is the next best step in management for a pt found to have polycythemia (elevated hemoglobin/ hematocrit; Hg greater than 16.5 in women and greater than 18.5 in men)?

A

serum erythropoietin level

(if elevated, due to renal cell carcinoma or chronic hypoxia secondary to cardiac/ pulmonary disease)

(if low, due to polycythemia vera)

1301
Q

What is the next best step in management of an HIV pt with asymptomatic thrombocytopenia and splenomegaly?

A

HAART therapy

1302
Q

what is virologic failure?

A

failure to achieve viral load less than 200 copies/mL within 24 weeks (6 months) of antiretroviral therapy in HIV pt

(suggests drug resistance or noncompliance)
(expected decrease is less than 5000 by 4 weeks, less then 500 by 8-16 weeks, and less then 50 by 16-24 weeks)

1303
Q

True or False. Physicians lecturing at conferences are allowed to accept honorarium and reimbursement for travel expenses from pharmacetical companies.

A

True

(if physician is just attending conference, not allowed to accept)
(lecturers should retain full control over the presentation content)

1304
Q

What is the next best step in management of a pt who is symptomatic (difficulty/ inability to swallow oral secretions, painful swallowing, feeling of fullness, discomfort) after accident foreign body ingestion or history of sharp object ingestion?

A

urgent endoscopy to remove object

1305
Q

What is the next best step in management of a pregnant woman with adnexal cyst measuring larger than 5 cm?

A

surgical removal in the 2nd trimester

(not prior to 1st trimester to decrease fetal complications)

(should remove bc increase chance of rupture, hemorrhage, and torsion which leads to preterm delivery)

1306
Q

What is the next best step in management of a child who is suffering from failure to thrive (weighs less than 5th percentile or slowing of linear growth such that cross 2 growth curve lines) but is otherwise well-appearing?

A

dietary modification

(proper formula preparation, amount of feedings, introduction of table food at appropriate time)

(newborn needs 110 kcal/kg/day, child up to 12 months needs 100 kcal/kg/day)

1307
Q

A pt presenting with hematemesis following a bout of retching/ vomiting and is found to have a single longitudinal tear at the gastro-esophageal junction most likely suffers from …

A

Mallory Weiss tear

(tx: observation and supportive care if not actively bleeding)

(associated with hiatal hernia- predisposes pts to mallory weiss syndrome)`

1308
Q

What is a common complication of Paget’s disease of bone?

A

hearing loss

best treament unknown

1309
Q

A pt presents with multiple small, vascular bright red dome shaped papules that blanch with pressure unless fibrotic and may bleed if disturbed most likely suffers from …

A

Cherry Angioma

benign vascular tumor; no tx needed

1310
Q

A pt with difficulty sleeping, morning headaches, shortness of breath that is worse when in supine position and confirmed with decreased forced vital capacity when lying supine, along with paradoxical abdominal wall retraction during inspiration when lying supine most likely suffers from …

A

Diaphragmatic Paralysis

(dx: sniff test using fluoroscopy)

(associated with amyotrophic lateral sclerosis if bilateral)

1311
Q

What are the two true contraindications to diphtheria-tetanus-pertusis (DTaP) vaccination?

A
  1. anaphylaxis within 7 days of administration of previous DTP or DTaP vaccine
  2. encephalopathy within 7 days of administration of previous DTP or DTaP vaccine
1312
Q

What are the common causes of metabolic acidosis with increased anion gap?

A
M- methanol intoxication
U- uremia
D- diabetic ketoacidosis
P- propylene glycol/ paraldhyde
I- isoniazid/ iron toxicity
L- lactic acidosis
E- ethylene glycol (antifreeze)
S- salicylates (aspirin)
1313
Q

A pt with a history of ulcerative colitis/ other inflammatory bowel disease who persents with toxic appearance, fever, weakness, generalized abdominal pain, bloody stools, abdominal tenderness and distension and tympanitic on percussion most likely suffers from ….

A

Toxic Megacolon

(dx: abdominal X-ray showing prominent dilation of colonic segments, multiple air-fluid levels, mucosal ulcerations or abdominal CT scan)
(tx: bowel rest with NG tube, steroids if not due to C. difficile, fluid resuscitation, avoid opioids and 5-ASA)

1314
Q

What is a common side effect associated with carbamazepine use?

A

neutropenia/ bone marrow suppression

assess for fever, mouth ulcers, petechiae, easy bruising

1315
Q

What does pregnancy category C mean in reference to a medication?

A

studies showed fetal risk in animals but without any human studies or no studies have been done in animals or pregnant women

(use if potential benefit outweighs risk)

1316
Q

What disorder is associated with shuffling gait (decreased speed and amplitued of leg movement during walking and small steps almost never raising feet from floor)?

A

Parkinson’s disease

1317
Q

What disorder is associated with steppage gait?

A

Distal lower motor neuron compromise

1318
Q

What disorder is associated with scissoring gait (drag leg forward with each step)?

A

Spastic paraparesis

1319
Q

What disorder is associated with drunken sailor gait (jerky hesitant steps, walks in zig zag)?

A

Cerebellar Ataxia

1320
Q

What disorder is associated with a walking on ice gait?

A

Senile Gait

1321
Q

A boy aged 3-10 years old presents with fever and pain in the hip/knee/thigh following a respiratory infection most likely usffers from ….

A

Transient Synovitis (toxic synovitis)

dx: ultrasound is most accurate

1322
Q

What is the next best step in management of a women 30 years old or older than presents with a new palpable breast mass?

A

mammogram and ultrasound

if complex cyst/ solid mass, image guided core biopsy
(if simple cyst, needle aspiration)

1323
Q

What is the next best step in management of a pt with exposure to possibly rabid animal (bat) but has history of prior rabies vaccination?

A

rabies vaccine on day 0 and 3

1324
Q

What is the next best step in management of a pt with exposure to possibly rabid animal (bat) without prior vaccination?

A

rabies vaccine on day 0,3,7,14 and rabies immunoglobulin on day 0

1325
Q

A pt presenting with weight loss, abdominal pain asthenia (physical weakness/ lack of energy), amenorrhea, fatigue, weakness, poor appetite, muscle tenderness, decreased axillary and pubic hair, increased pigmentation, hyponatremia, hyperkalemia, and hyperchloremic metabolic acidosis most likely suffers from …

A

Addison’s disease (Adrenal Insufficiency)

dx: cortisol and ACTH level, cosyntropin level

1326
Q

What two antibodies are associated with systemic erythematous lupus (SLE)?

A

antinuclear antibodies (ANA) and anti-double stranded DNA (dsDNA)

1327
Q

What is the best initial treatment for systemic lupus erythematous (SLE)?

A

low dose prednisone (5-15 mg.day) and hydroxychloroquine

1328
Q

What is the nleading cause of death/ mortality in pts with systemic lupus erythematous (SLE)?

A

cardiovascular events (especially accelerated/ premature coronary atherosclerosis)

1329
Q

A pt presenting with worsening dull epigastric pain after eating in the setting of a history of severe atherosclerotic disease (coronary artery disease, carotid bruits, peripheral vascular disease) most likely suffers from ….

A

chronic mesenteric ischemia

dx: abdominal angiography is gold standard

1330
Q

A pt presenting with green frothy dvaginal discharge, vaginal pruritis, dysuria, urinary frequency, pear shaped motile organism and strawberry cervix (vaignal and cervical petechiae) most likely suffers from …

A

Trichomonas Vaginalis

tx: oral metronidazole

1331
Q

What is the next best step in management for tuberculosis meningitis/ miliary TB/ TB osteomyelitis?

A

Isoniazid (INH), rifampin and pyrazinamide for 2 months followed by INH and rifampin for remaining months

(for totoal treatment time of 12 months)

1332
Q

When are pts with pulmonary/ laryngeal tuberculosis considered non-infectious?

A

clinically improving and have 3 negative sputum smears

1333
Q

What are the absolute contraindications to measles- mumps- rubella (MMR) vaccine? (4)

A
  1. anaphylaxis to neomycin
  2. anaphylaxis to gelatin
  3. pregnancy
  4. immunocompromised

(delay 3-11 months after immunoglobulin treatment, avoid 2nd dose if develop thrombocytopenia with first dose)

1334
Q

What is the next best step in managaement of a pt with hypoxemia due to acute asthma exacerbation?

A

oxygen followed by beta 2 agonist (albuterol)

1335
Q

What is the next best step in management of a pt with active hemorrhage while on warfarin therapy?

A

fresh frozen plasma infusion

rapidly provide clotting factors

1336
Q

What is the best next step in management of chronic constipation in a pt who did not respond to increased dietary fiber intake and lacks a secondary / medication cause?

A

bulk forming laxative (psyllium, methylcellulose, polycarbophi)

1337
Q

A pt presenting with acute florid nephrotic range proteinuria (more than 2.5 grams/ day) while treating chronic pain most likely suffers from …

A

analgesic induced nephropathy (NSAIDs)

1338
Q

A pt presenting with acute episodes of vertigo, dizziness, dysarthria, diplopia, and numbness in the setting of having hypertension/ diabetes/ coronary artery disease/ smoking most likely suffers from …

A

Vertebrobasilar insufficiency

1339
Q

A pt presenting with fever, headache, and neck stiffness andhas a CSF finding of high open pressure, neutrophilic leukocytosis, high protein, and low glucose most likely suffers from …

A

Bacterial meningitis

1340
Q

A pt presenting with fever, abdominal pain that started in epigastrium and then localized to right lower quadrant, nausea, tenderness of RLQ on palpation mcburney’s point), RLQ pain elicited with leg elevation (iliopsoas sign), and RLQ pain elicited with RLQ palpation (Rovsing’s sign) most likely suffers from …

A

Appendicitis

1341
Q

What complication occurs in pregnant pts with appendicitis when it is not adequately treated?

A

Pylephlebitis

infection of thrombosis in portal vein

1342
Q

What is the next best step in management of a pt with type 1 diabetes who experiences hypoglycemia during increased physical activity/ exercise?

A

decrease insulin dose

1343
Q

What is the next best step in management of a pt with a type 3 pressure ulcer (tissue damage extends into subcutaneous tissue but not thru underlying fascia)?

A

covered with occlusive dressing or losoely packed wtih saline moistened gauze

(moist wound environment while keeping viable surrounding skin dry)

1344
Q

Can a pt with a life limiting illness and estimated prognosis of 6 months or less that wants to forgive life-prolonging treatment enter hospice if they are full code?

A

Yes

1345
Q

What is the most likely cause of step up in oxygen saturation between the right atrium and the right ventricle?

A

Ventricular Septal Defect (VSD)

harsh holosystolic murmur heard best at left 3rd and 4th intercoastal spaces with associated palpable thrill

1346
Q

What is the most likely cause of step up in oxygen saturation between the right ventricle and the pulmonary artery?

A

patent ductus arteriosus (PDA)

continuous machine like murmur heard best at left infraclavicular area

1347
Q

What is the Model for End Stage Liver DIsease (MELD) calculation and what is it used for?

A

MELD = 3.8 [Ln serum bilirubin] + 11.2 [Ln INR] + 9.6 [Ln serum creatinine] + 6.4

(bilirubin, INR, and creatinine are best prognostic indicators for chronic liver disease)

1348
Q

… is conducted by pooling the data from severeal studies to increase the statistical power (ability to detect difference in outcome between groups)

A

Meta-Analysis

1349
Q

A pt presenting with recent onset diabetes, bronzed skin, hepatomegaly, weakness, decreased libido, along with birefringent rhomboid crystals in synovial joints affected by arthritis (usually 2nd and 3rd metacarpophalangeal) most likely suffers from …

A

Hemochromatosis

(iron overload; associated with calcium pyrophospahte dihydrate crystals)

(dx: elevated ferritin and serum iron, transferrin saturation greater than 50%

1350
Q

What is the best initial step in management for a pt with gross hematuria in the absence of trauma or suspected nephrolithiasis (renal stones)?

A

urinalysis and urine culture

1351
Q

What is the next best step in management for an elderly pt who is unable to perform the basic activities of daily living and unable to take care of themselves but is ready to be discharged from the hospital and has no one at home to care for the pt and guarentee adherence to treatment and medical follow-up?

A

Nursing Home

1352
Q

What is the leading cause of mortality in pts with acromegaly?

A

cardiovascular disease

LV dysfunction, coronary atherosclerosis, asymmetrical septal hypertrophy, conduction defect

1353
Q

What are the indications for proton pump inhibitor prophylaxis for stress ulcers? (4)

A
  1. have coagulopathy
  2. hx of GI bleed in last year
  3. mechanicl ventilation for mmore than 48 hours
  4. 2 of the following (sepsis, ICU for more than 1 week, occult GI bleeding for more than 6 days, steroid therapy)